You are on page 1of 137

 Clozapine is an atypical antipsychotic medication used to treat schizophrenia that has not responded to

standard, more traditional treatment.  Clozapine is associated with a risk for agranulocytosis and is
therefore used only in clients with treatment-resistant schizophrenia. must have their WBC and ANC
monitored regularly throughout the course of therapy (initially once every week). the health care
provider (HCP) immediately if fever or a sore throat develops, as this may indicate an underlying
infection from neutropenia. also cause metabolic syndrome (weight gain, hyperlipidemia, insulin
resistance/diabetes) and seizures.

 Weight gain—a baseline height, weight, and waist circumference should be obtained, and a BMI can be
calculated

 Hyperglycemia—symptoms of hyperglycemia (eg, increased thirst and urination, weakness, increased


blood glucose) should be monitored

 Dyslipidemia—a lipid profile should be obtained

A hemorrhagic stroke occurs when a blood vessel ruptures in the brain and causes bleeding into the brain
tissue or subarachnoid space.  Seizure activity may occur due to increased intracranial pressure (ICP)
(Option 3).  During the acute phase, a client may develop dysphagia.  To prevent aspiration, the client must
remain NPO until a swallow function screen reveals no deficits (Option 4).  The nurse should perform
neurological assessments (eg, level of consciousness, pupillary response) at regular intervals and report any
acute changes (Option 5).
Preventing activities that increase ICP or blood pressure will minimize further bleeding.  The nurse should:
 Reduce stimulation, maintain a quiet and dimly lit environment, limit visitors
 Administer stool softeners to reduce strain during bowel movements (Option 1)
 Reduce exertion, maintain strict bed rest, assist with activities of daily living
 Maintain head in midline position to improve jugular venous return to the heart

Sulfonylureas (eg, glyburide) stimulate insulin release via the pancreas and carry a risk for severe and
prolonged hypoglycemia in the geriatric population due to potential delayed elimination.  Avoidance of these
drugs is recommended by the Beers Criteria.  Instead, other medications that are at lower risk for hypoglycemia
should be used (eg, metformin)

Nephrotic syndrome, an autoimmune disease, affects children age 2-7 and is characterized by increased
permeability of the glomerulus to proteins (eg, albumin, immunoglobulins, natural anticoagulants).  Loss of
albumin in urine leads to hypoalbuminemia; this causes decreased plasma oncotic pressure, which allows fluid
to leak out of the vascular spaces.  Reduced plasma volume (hypovolemia) activates kidneys to retain salt and
water (renin-angiontensin-aldosterone system).  Clients will have generalized edema, weight gain, loss of
appetite (from ascites), and decreased urine output.  Loss of immunoglobulins makes children susceptible to
infection.  Treatment typically includes:
 Corticosteroids and other immunosuppressants (eg, cyclosporine)
 Loss of appetite management by making foods fun and attractive
 Infection prevention (eg, limiting social interaction until the child is better)

Nephrotic syndrome is a collection of symptoms resulting from various causes of glomerular injury.  Below
are the 4 classic manifestations of nephrotic syndrome:

 Massive proteinuria – caused by increased glomerular permeability


 Hypoalbuminemia – resulting from excess protein loss in the urine
 Edema – specifically periorbital and peripheral edema and ascites; caused by low serum protein and
albumin as fluid is pulled into interstitial spaces and body cavities
 Hyperlipidemia – related to increased compensatory protein and lipid production by the liver

Additional symptoms include decreased urine output, fatigue, pallor, and weight gain.

The most common cause of nephrotic syndrome in children is minimal change nephrotic syndrome, which is
generally considered idiopathic.  Less common secondary causes may be related to systemic disease or
infection, such as glomerulonephritis, drug toxicity, or acquired immunodeficiency syndrome.

Pica is the abnormal, compulsive craving for and consumption of substances normally not considered nutritionally
valuable or edible.  Common substances include ice, cornstarch, chalk, clay, dirt, and paper.  Although the condition is
not exclusive to pregnancy, many women only have pica when they are pregnant.  Pica is often accompanied by iron
deficiency anemia due to insufficient nutritional intake or impaired iron absorption.  However, the exact relationship
between pica and anemia is not fully understood.  The health care provider would likely order hemoglobin and
hematocrit levels to screen for the presence of anemia.

Immediate postoperative nursing care focuses on management of the airway, breathing, circulation, bleeding, and pain. 
Although antiemetic medications are typically administered immediately after surgery to control nausea and vomiting,
nausea is still a common complication caused by anesthetic side effects and decreased gastrointestinal motility.  Clients
are at high risk for aspiration (and possible asphyxiation) due to their altered level of consciousness, which is caused by
anesthesia.  Clients reporting nausea should be placed immediately on their side to prevent aspiration of vomit.
Postoperative clients are at an increased risk for vomit aspiration due to nausea and an altered level of consciousness
(caused by anesthesia).  These clients should be placed on their side and should receive antiemetics to prevent potential
airway and breathing complications.

The needle is inserted between ribs 6 and 7 or 8 and 9 while the client lies supine with the right arm over the
head and holding the breath.  A full bladder is a concern with paracentesis when a trocar needle is inserted into
the abdomen to drain ascites.  An empty bladder may aid comfort, but it is not essential for safety.

The client must lie on the right side for a minimum of 2-4 hours to splint the incision site.  The liver is a
"heavy" organ and can "fall on itself" to tamponade any bleeding.  The client stays on bed rest for 12-14 hours.
Essential nursing actions related to a needle liver biopsy include checking coagulation, blood type, and crossmatch
beforehand, positioning the client on the right side for hours afterward, and monitoring vital signs and for potential signs
of shock.

Albuterol (Proventil) is a short-acting beta-2 agonist that produces immediate bronchodilation by relaxing smooth
muscles.  Bronchodilation decreases airway resistance, facilitates mucus drainage, decreases the work of breathing, and
increases oxygenation.  Peak flow will improve.  The most frequent side effects are tremor, tachycardia, restlessness,
and hypokalemia.

Following open radical prostatectomy, any rectal interventions such as suppositories or enemas must be avoided to
prevent stress on the suture lines and problems with healing in the surgical area.  The client should not strain when
having a bowel movement for these reasons.

Central chemoreceptors located in the respiratory center of the brain (medulla) respond to changes in blood
carbon dioxide and hydrogen ions by either increasing or decreasing ventilation to normalize the pH.  When the
receptors sense a low pH (acidosis), ventilation increases to rid the body of excess carbon dioxide; when the
receptors sense a high pH (alkalosis), ventilation decreases to retain carbon dioxide.  Peripheral chemoreceptors
located in the carotid and aortic bodies respond to low levels of oxygen and stimulate the respiratory center to
increase ventilation.

Many clients with COPD breathe because their oxygen levels are low rather than because carbon dioxide levels
are high.  This is commonly referred to as the hypoxemic drive.  If they receive too high a level of inspired
oxygen, this drive can be blunted.  It is therefore important for these clients to receive a "guaranteed" amount of
oxygen as an increase in inspired oxygen can decrease the drive to breathe.

To promote adequate gas exchange, the nurse should use a high-flow Venturi mask to deliver a specified,
guaranteed amount of oxygen.  Because this device has a mechanism that controls the mixture of room air, the
inspired oxygen concentration remains constant despite changes in respiratory rate, depth, or tidal volume.  It is
the most appropriate intervention to promote adequate gas exchange.

Amniotomy refers to the artificial rupture of membranes (AROM) and may be performed by the health care
provider to augment or induce labor.  After AROM, there is a risk of umbilical cord prolapse if the fetal head is
not applied firmly to the cervix.  A prolapsed cord can cause fetal bradycardia due to cord compression.  The
nurse should assess the fetal heart rate before and after the procedure (Option 1).

The nurse should note the amniotic fluid color, amount, and odor.  Amniotic fluid should be clear/colorless and
without a foul odor.  Yellowish-green fluid can indicate the fetal passage of meconium in utero, and a strong,
foul odor may indicate infection (Option 5).  Once the membranes are ruptured, there is an increased risk for
infection.  The nurse should monitor the client's temperature at least every 2 hours after AROM (Option 2).

(Option 3)  As with any vaginal examination, the client may feel some pressure and discomfort during an
amniotomy.  However, the actual AROM procedure, or "breaking the bag of water," is painless.
(Option 4)  Supine positioning decreases uteroplacental blood flow and fetal oxygenation.  The client should be
assisted to upright positions after AROM to allow for drainage of amniotic fluid and to encourage the fetal head
to remain firmly applied to the cervix.

Clients with schizophrenia have difficulty initiating and maintaining social interactions with other people.  The
nurse can facilitate interpersonal functioning by providing one-on-one interaction in which the client can
practice basic social skills in a non-threatening way.  Once the client feels more comfortable, the nurse can
encourage participation in activities that require some interaction with others.

Impaired social interaction is one of the negative symptoms of schizophrenia; others include the following:

 Inappropriate, flat, or bland affect, and apathy


 Emotional ambivalence, disheveled appearance
 Inability to establish and move toward goal accomplishment
 Lack of energy, pacing and rocking, odd posturing
 Regressive behavior, inability to experience pleasure
 Seeming lack of interest in the world and people

It is the negative symptoms of schizophrenia that affect a client's ability to establish personal relationships and
manage day-to-day social interactions.  The positive symptoms of schizophrenia (hallucinations, delusions,
thought impairment) often improve with psychotropic medications; negative symptoms tend to persist even with
medication.  Psychosocial and supportive treatment, including psychotherapy, education, behavioral training,
cognitive therapy, and social skills therapy, may be beneficial in improving the quality of life for clients with
schizophrenia.

Ankylosing spondylitis (AS) is an inflammatory disease affecting the spine that has no known cause or cure. 
AS is characterized by stiffness and fusion of the axial joints (eg, spine, sacroiliac), leading to restricted spinal
mobility.  Low back pain and morning stiffness that improve with activity are the classic findings.  Involvement
of the thoracic spine (costovertebral) and costosternal junctions can limit chest wall expansion, leading to
hypoventilation.

The client with AS should do the following:

 Promote extension of the spine with proper posture, daily stretching, and swimming or racquet sports
(Option 2)
 Stop smoking and practice breathing exercises to increase chest expansion and reduce lung
complications (Option 3)
 Manage pain with moist heat and nonsteroidal anti-inflammatory drugs (NSAIDs)
 Take immunosuppressant and anti-inflammatory medications as prescribed to reduce inflammation
and increase mobility

(Option 1)  It is best to rest during flare-ups.  The client should wait to exercise until the pain and
inflammation are under control.

(Option 4)  Clients with AS are encouraged to sleep on their backs on a firm mattress to prevent spinal
flexion and the resulting deformity.

(Option 5)  Ibuprofen and other NSAIDs should be taken with a meal or snack to avoid gastric upset.

Influenza is a respiratory illness common during the cooler months of the year.  Each year, a new influenza vaccine is
created to help protect against specific viral strains.  The Centers for Disease Control and Prevention and Public Health
Agency of Canada recommend that all clients age ≥6 months receive the influenza vaccine annually unless the client has
a life-threatening allergy to the vaccine or one of its ingredients.

Immune thrombocytopenic purpura (ITP) is an autoimmune condition in which antibodies bind to and cause
destruction of platelets.  Clients with ITP have a platelet count <150,000/mm3 (150 x 109/L) and are at increased
risk of bleeding.

Key teaching to reduce the client's risk of bleeding includes:

 Use soft-bristle toothbrushes, gentle flossing, and nonalcoholic mouthwashes.  These prevent
periodontal disease and gingival bleeding (Option 1).
 Avoid activities that may cause trauma (eg, high-intensity sports).  Appropriate exercise includes low-
impact activity (eg, walking) while wearing nonskid footwear to help prevent falls (Option 2).
 Take prescribed stool softeners and laxatives as needed.  These medications prevent hard stools and
straining, which can cause anorectal fissuring, bleeding, and hemorrhoids (Option 4).

(Option 3)  Clients with ITP should use electric razors instead of safety or straight razors.  Electric razors have
a more complete guard, reducing the risk of accidentally nicking the skin.

(Option 5)  Clients with ITP should avoid nonsteroidal anti-inflammatory drugs (eg, aspirin, ibuprofen,
ketorolac), which further impair platelet function.  Acetaminophen and opiates are better options for pain
management.

Respiratory distress is a life-threatening complication of thyroid surgery that occurs when swelling in theAirway swelling
is a life-threatening complication of thyroid surgery.  Signs of respiratory distress such as stridor and dyspnea require
rapid intervention. surgical area at the base of the neck compresses the airway.  Stridor and/or difficulty breathing in
the client who has had thyroid surgery should be reported immediately to the registered nurse, and a rapid response
should be activated.

Metformin is an oral antidiabetic medication used to manage hyperglycemia in clients with type 2 diabetes.  Metformin
increases the sensitivity of insulin receptors in cells and reduces glucose production by the liver.  These actions increase
the efficacy of insulin present in the body and prevent large rises in blood glucose after meals.  Because metformin does
not stimulate insulin secretion by the pancreas, the risk of hypoglycemia is minimal (Option 3).  Although skipping meals
would cause a drop in blood glucose, metformin would not cause further hypoglycemia.

Clients with MVP may have palpitations, dizziness, and lightheadedness.  Chest pain can occur but its
etiology is unknown in this client population.  It may be a result of abnormal tension on the papillary muscles. 
Chest pain that occurs in MVP does not typically respond to antianginal treatment such as nitrates.  Beta
blockers may be prescribed for palpitations and chest pain.

Client teaching for MVP includes the following:

 Adopt healthy eating habits and avoid caffeine as it is a stimulant and may exacerbate symptoms
(Option 3)
 Check ingredients of over-the-counter medications or diet pills for stimulants such as caffeine or
ephedrine as they can exacerbate symptoms
 Reduce stress and avoid alcohol use
 Obstructive sleep apnea (OSA) is a chronic condition that involves the relaxation of pharyngeal
muscles during sleep.  The resulting upper airway obstruction with multiple events of apnea and
shallow breathing (hypopnea) leads to hypoxemia and hypercapnia.  CPAP is an effective treatment for
OSA; it involves using a nasal or full face mask that delivers positive pressure to the upper airway to
keep it open during sleep.
 In this case, the nurse's first action should be to check the tightness of the straps that hold the mask in
place.  The full face mask must fit snugly over the client's nose and mouth without air leakage to
maintain the positive airway pressure and prevent obstruction of upper airway airflow.  Readjustment of
the head straps may be necessary
 Buerger's disease (thromboangiitis obliterans) is a nonatherosclerotic vasculitis involving the arteries
and veins of the lower and upper extremities.  It occurs most often in young men (age <45) with a long
history of tobacco or marijuana use and chronic periodontal infection, but no other cardiovascular risk
factors.
 Clients experience thrombus formation, resulting in distal extremity ischemia, ischemic digit ulcers, or
digit gangrene.  They often have intermittent claudication of the feet and hands.  Over time, rest pain
and ischemic ulcerations may occur.  Many clients also develop secondary Raynaud phenomenon (cold
sensitivity).
 The mainstay treatment of Buerger's disease is the cessation of all tobacco and marijuana use in any
form.  Nicotine replacement products (eg, nicotine patch) are contraindicated.  However, bupropion
and varenicline can be used for smoking cessation.  Clients may have to choose between continued use
of tobacco and marijuana and their affected limbs.  Conservative management includes avoidance of
cold exposure to affected limbs, a walking program, antibiotics for any infected ulcers, analgesics for
ischemic pain, and avoidance of trauma to the extremities.
 (Option 1)  Clients should avoid exposure to cold (not warm) weather to prevent vasoconstriction and
worsening of symptoms.
 (Option 2)  Statins do not provide much benefit as this is a vasculitis and not an atherosclerotic
condition.
 (Option 3)  Warfarin is an anticoagulant and is not indicated in the treatment of Buerger's disease. 
Calcium channel blockers, cilostazol, and sildenafil have been used, but there is insufficient evidence to
support their effectiveness.  Intravenous iloprost has been shown to improve rest pain, promote healing
of ulcers, and decrease the need for amputation.
 Educational objective:
Buerger's disease is a nonatherosclerotic vasculitis involving small to medium arteries and veins of the
upper and lower extremities.  Young male smokers are typically affected.  Clients should avoid exposure
to cold weather and cease using tobacco and marijuana in all forms.  Smoking cessation can be achieved
with bupropion or varenicline but not with nicotine replacement products.

Major predisposing factors for the development of delirium in hospitalized clients include:

1. Advanced age
2. Underlying neurodegenerative disease (stroke, dementia)
3. Polypharmacy
4. Coexisting medical conditions (eg, infection)
5. Acid-base/arterial blood gas imbalances (eg, acidosis, hypercarbia, hypoxemia)
6. Metabolic and electrolyte disturbances
7. Impaired mobility - early ambulation prevents delirium
8. Surgery (postoperative setting)
9. Untreated pain and inadequate analgesia

Client 4 has 4 predisposing risk factors: advanced age, acidosis and hypoxemia associated with chronic
respiratory failure, and sepsis.  This client is at greatest risk for developing delirium.

The following should be taught to clients taking tetracyclines (eg, tetracycline, doxycycline, minocycline):
1. Take on an empty stomach – for optimum absorption, tetracyclines should be taken 1 hour before or 2
hours after meals (Option 3)
2. Avoid antacids or dairy products – tetracyclines should not be taken with iron supplements, antacids,
or dairy products as they bind with the drug and decrease its absorption (Option 1)
3. Take with a full glass of water – tetracyclines can cause pill-induced esophagitis and gastritis; the risk
can be reduced by taking with a full glass of water and remaining upright after pill ingestion
4. Photosensitivity – severe sunburn can occur with tetracycline.  The client should use sunblock (Option
5).

Medications such as tetracycline and rifampin can decrease the effectiveness of oral contraceptives; additional
contraceptive techniques will be needed

People with latex allergy usually have a cross-allergy to foods such as bananas, kiwis, avocados, tomatoes,
peaches, and grapes because some proteins in rubber are similar to food proteins.  Latex sensitivity increases
with exposure and should be suspected in the following situations:

1. Allergic contact dermatitis (rash, itching, vesicles) developing 3–4 days after exposure to a rubber
latex product.  This is a type IV hypersensitivity reaction (delayed onset).
2. Anaphylaxis - many cases of anaphylaxis have been reported in both medical and non-medical settings. 
These represent a type I hypersensitivity reaction and should be treated with intramuscular epinephrine
injections.  Some common settings include:
o Glove use
o Procedures involving balloon-tipped catheters (eg, arterial catheterization)
o Blowing up toy balloons
o Use of bottle nipples, pacifiers
o Use of condoms or diaphragms during sex

Clients with severe allergies should wear a Medic Alert bracelet and carry an injectable epinephrine pen due to
cross-sensitivity with many food and industrial products that can be impossible to avoid.

Peripherally acting calcium channel blockers (eg, nifedipine, amlodipine, felodipine) cause vasodilation, and clients may
develop peripheral edema.

Necrotizing enterocolitis occurs predominantly in preterm infants secondary to gastrointestinal and


immunologic immaturity.  On initiation of enteral feeding, bacteria can be introduced into the bowel, where
they can proliferate excessively due to compromised immune clearance.  This results in inflammation and
ischemic necrosis of the intestine.  As the disease progresses, the bowel becomes congested and gangrenous
with gas collections forming inside the bowel wall. Measuring the client's abdominal girth daily is an
important nursing intervention to note any worsening intestinal gas-associated swelling.  Clients are made NPO
and receive nasogastric suction to decompress the stomach and intestines.  Parenteral hydration and nutrition
and IV antibiotics are given. To avoid pressure on the abdomen and facilitate observation for a distended abdomen,
clients are placed supine and undiapered. Rectal temperatures should be avoided due to the risk of perforation.

Postpartum psychosis is a rare but serious perinatal mood disorder.  Research suggests a multifactorial
etiology, including genetic predisposition and hormone fluctuation after birth.  Risk factors include history of
bipolar disorder and previous discontinuation of mood-stabilizing medications (eg, lithium).

Signs appear within 2 weeks after birth and include hallucinations, delusions, paranoia, severe mood changes,
delirium, and feelings that someone will harm the baby (Option 2).  Postpartum psychosis is a psychiatric
emergency requiring hospitalization, pharmacologic intervention, and long-term supportive care.  Women
exhibiting signs of postpartum psychosis are at increased risk of suicide and infanticide, and their assessment
should take priority to ensure the safety of mother and baby.

Cirrhosis, the end stage of many chronic liver diseases, is characterized by diffuse hepatic fibrosis with
replacement of the normal architecture by regenerative nodules.  The resulting structural changes alter blood
flow through the liver and decrease the liver's functionality.

Elevated bilirubin (jaundice) results from functional derangement of liver cells and compression of bile ducts by
nodules.  The liver has a decreased ability to conjugate and excrete bilirubin (Option 3).

Most coagulation factors are produced in the liver.  A cirrhotic liver cannot produce the factors essential for
blood clotting.  As a result, coagulation studies (prothrombin time [PT]/International Normalized Ratio [INR]
and activated partial thromboplastin time [aPTT]) are usually elevated (Option 4).

Ammonia from intestinal deamination of amino acids normally goes to the liver and is converted to urea and
excreted by the kidney.  This does not happen in cirrhosis.  Instead, the ammonia level rises as the cirrhosis
progresses; ammonia crosses the blood-brain barrier and results in hepatic encephalopathy (Option 2).

(Options 1 and 5)  Albumin holds water inside the blood vessels.  In cirrhosis, the liver is unable to synthesize
albumin (protein), so hypoalbuminemia would be expected.  This is the primary reason that fluid leaks out of
vascular spaces into interstitial spaces (eg, edema, ascites).  The kidneys perceive this as low perfusion and try
to reabsorb (conserve) both sodium and water.  The large amount of water in the body results in a dilutional
effect (low sodium).

Educational objective:
The chronic, progressive destruction characteristic of cirrhosis causes bilirubin, ammonia, and coagulation
studies (PT/INR and aPTT) to become elevated.  Hyponatremia and hypoalbuminemia are to be expected.

Systemic inflammatory response syndrome (SIRS) occurs when the body undergoes a major insult (eg,
trauma, infection, burns, hemorrhage, multiple transfusions).  Stimulation of the immune response leads to
activation of white blood cells (WBCs), release of inflammatory mediators, increased capillary permeability,
and inflammation of organs.  The sepsis continuum progresses in severity from sepsis, to severe sepsis, to septic
shock, to multiple organ dysfunction (MODS).

Sepsis is an exaggerated systemic inflammatory response associated with a documented or suspected infection. 
Severe sepsis is sepsis complicated by organ dysfunction.  Septic shock is severe sepsis with hypotension
despite fluid resuscitation.  MODS occurs in relation to decreased perfusion and is the end point of the sepsis
continuum.  It is important for the nurse to recognize manifestations of SIRS to promote early recognition,
prevention, and treatment of infection and to limit its progression to MODS.

Diagnostic criteria for SIRS include 2 or more of the following manifestations:

 Hyperthermia (temperature >100.4 F [38 C]) or hypothermia (temperature <96.8 F [36 C])
 Heart rate >90/min
 Respiratory rate >20/min or alkalosis (PaCO2 <32 mm Hg [4.3 kPa])
 Leukocytosis (WBC count >12,000/mm3 [12.0 x 109/L] or 10% immature neutrophils [bands])

The heart rate, respiratory rate, and temperature are elevated, and the WBC count is increased; these findings
indicate the presence of SIRS.

(Option 1)  CVP (normal 2-8 mm Hg) indicates circulating volume.  It is decreased, not increased, in septic
shock due to massive vasodilation and maldistribution of blood flow.  An abnormal finding is not associated
with SIRS and would not be expected in this client.

(Option 2)  Sepsis with hypotension and decreased perfusion despite fluid resuscitation is a characteristic
finding in septic shock.  An MAP of 80 mm Hg is within the normal range (70-105 mm Hg).  It is not
associated with SIRS and is not an expected finding in this client.

Educational objective:
Temperature (hyper- or hypothermia), respirations >20/min, heart rate >90/min, and WBC count >12,000/mm3
(12.0 x 109/L) are assessed to document SIRS.  The presence of 2 or more of these findings indicates the
syndrome.

Systemic inflammatory response syndrome (SIRS) occurs due to trauma, ischemia, infection (ie, sepsis), or
other distributive shock processes that trigger systemic inflammation remote from the primary source.  This
overwhelming inflammatory response can rapidly progress to hemodynamic instability, respiratory failure,
and end-organ dysfunction.

This client has fever and decreased arterial pCO2.  Due to the high morbidity and mortality of clients with
SIRS, early therapy with aggressive fluid resuscitation and other indicated treatments (eg, antibiotics) based
on cause is crucial, so this client is the highest priority

Hemophilia is a hereditary bleeding disorder caused by a deficiency in coagulation proteins.  Treatment


consists of replacing the missing clotting factor and teaching the client about injury prevention, including:

 Avoid medications such as ibuprofen and aspirin that have platelet inhibition properties (Option 4).
 Avoid intramuscular injections; subcutaneous injections are preferred.
 Avoid contact sports and safety hazards; noncontact activities (eg, swimming, jogging, tennis) and use
of protective equipment (eg, helmets, padding) are encouraged (Option 5).
 Dental hygiene is necessary to prevent gum bleeding, and soft toothbrushes should be used.
 MedicAlert bracelets should be worn at all times (Option 3).

Diabetic neuropathy is caused by nerve damage as a result of the metabolic disturbances associated with diabetes
mellitus.  Autonomic neuropathy is nerve damage to the autonomic nervous system, the system responsible for
involuntary body functions such as blood pressure, heart rate, perspiration, sexual function, and digestion.  Impairment
to the autonomic nervous system caused by neuropathy can cause symptoms such as postural hypotension, tachycardia,
painless myocardial infarction, bowel incontinence, diarrhea, urinary retention, and hypoglycemic unawareness.  The
client with postural hypotension is also at risk for falls and should be taught to get up from a lying or sitting position
slowly.

An external fixator is a metallic device composed of metal pins (screws) placed into the bone to stabilize it;
these are positioned above and below the fracture through small incisions in the skin and muscle.  After the pins
are placed, they are attached to an adjustable external rod or frame outside the skin.

Infection of the pin tract is a major complication associated with the device.  The nurse should notify the HCP
immediately if there are signs or symptoms of infection (eg, drainage, pain, erythema, swelling, fever, pin
looseness) at the pin sites.  Prompt treatment with antibiotic therapy is necessary as a localized pin tract
infection can progress to osteomyelitis, an infection of the bone (Option 3).

Infection can also cause the pins to loosen, and this can lead to bone displacement.  Therefore, the nurse should
perform meticulous sterile pin care with 1/2-strength hydrogen peroxide and NSS or chlorhexidine solution,
or as directed by institution policy and procedure (Option 5).

Regular neurovascular assessment is important after fixator placement as inadvertent pin placement can
compromise the integrity of nerves and vessels (Option 4).

(Option 1)  Loosening of the pins can compromise bone alignment and healing.  The nurse should assess the
pins regularly and notify the HCP if they are loose but should not turn the bolts to tighten.

(Option 2)  An external fixator device allows for early ambulation with the device in place, increases
independence while maintaining bone immobilization, and prevents immobility hazards.  If used long-term (>4
weeks), the fixator is removed when the bone is healed.

Educational objective:
Nursing interventions to prevent common complications (eg, infection, loosening of pins) associated with an
external fixator include meticulous sterile pin care with an antimicrobial solution, regular assessment of pin
tightness, and immediate HCP notification if pins are loose or there are signs of infection.

Toddlers (age 1-3) display an egocentric approach as they strive for autonomy.  They attempt to control their
experiences through intense emotional displays, such as temper tantrums or forceful negative responses (eg,
"no!").  Hospitalization results in loss of a toddler's usual routines and rituals, often resulting in regressive
behavior.  The toddler may also be frequently separated from the parents, leading to separation anxiety.

Nursing care activities should be similar to home routines, such as providing preferred snacks and anticipating
nap time.  The toddler should be given options rather than asked yes/no questions to limit the potential negative
responses.  It is also important to encourage participation and presence of the parents whene

Myelomeningocele occurs when the neural tube fails to fuse properly during fetal development.  An
outpouching of spinal fluid, spinal cord, and nerves covered by only a thin membrane occurs, typically in the
lumbar area.  The newborn is at high risk for infection at this area.  A priority nursing intervention is to cover
the area with a sterile, moist dressing to decrease the risk of infection until surgical repair can occur.

(Option 1)  Assessing for an anal wink will assist in the assessment of the level of neurologic deficit but is not a
priority intervention.

(Option 3)  Myelomeningocele may decrease the absorption of cerebrospinal fluid, which would place the
newborn at risk for hydrocephalus from the excess cerebrospinal fluid.  An occipital frontal circumference is
needed as a baseline measurement but is not a priority.

(Option 4)  The newborn would be placed in the prone position (with face turned to the side) to prevent
rupture of the myelomeningocele.

 Dysfunctional gastric motility related to bowel manipulation during surgery, anesthesia, and opioid
analgesia as evidenced by absent or hypoactive bowel sounds 48-72 hours following surgery secondary to a
paralytic ileus.  It is a common complication following abdominal surgery (Option 2).

 Imbalanced nutrition, less than body requirements related to the increased metabolic demand needed for
tissue and wound healing as evidenced by the inability to ingest adequate caloric intake secondary to a paralytic
ileus and the lack of interest in eating secondary to the ileus, the adverse effects of anesthesia, and analgesic
medications

Fondaparinux (Arixtra), unfractionated heparin, and low molecular weight heparin (eg, enoxaparin, dalteparin)
are anticoagulants commonly used for deep vein thrombosis and pulmonary embolism prophylaxis after
hip/knee replacement or abdominal surgery.  However, fondaparinux is not administered until more than 6
hours after any surgery, and anticoagulants are not given while an epidural catheter is in place (Option 2).

Fondaparinux is associated with epidural hematoma.  Any bleeding in the tight epidural space, which does not
expand, could result in spinal cord compression.  Signs of epidural spinal hematoma can include severe back
pain and paralysis.

(Option 1)  Paresthesia is an expected finding from postoperative analgesia for 2-24 hours after surgery,
depending on the agent and location.  Continuously administered analgesia usually results in some paresthesia
until approximately 4-6 hours after discontinuance.  As long as the level remains relatively stable or improves,
it is an acceptable finding.  However, paresthesia or motor weakness is a concern when the sensory or motor
block outlasts the expected duration.
(Option 3)  Client response to operative analgesia and postoperative continued analgesia can range from
minimal to significant.  As long as the analgesic is infusing and findings remain stable, reduced muscle strength
is expected.

(Option 4)  Major orthopedic surgery can result in significant blood loss, and it is not unusual for the client to
have hemoglobin drop of 1-2 g/dL (10-20 g/L).  Blood loss should be monitored over time; transfusion usually
is not indicated unless hemoglobin is <7-8 g/dL (70-80 g/L).

Educational objective:
Residual paresthesia and motor weakness for several hours are expected findings after regional anesthesia. 
Anticoagulants are not given while an epidural catheter is in place.

Quadriplegia (tetraplegia) occurs when the lower limbs are completely paralyzed and there is complete or
partial paralysis of the upper limbs.  This is usually due to injury of the cervical spinal cord.  Depending on the
area of injury and extent of cord edema, the airway can be adversely affected.  The priority assessment for this
client is the status of the airway and oxygenation.  The nurse should frequently assess breath sounds,
accessory muscle use, vital capacity, tidal volume, and arterial blood gas values (if prescribed).

(Options 2, 3, and 4)  This client will need passive range of motion exercises on affected joints to prevent
contractures.  Turning the client will be necessary to prevent skin breakdown over bony prominences.  The
client will need to express feelings and work through the grief process related to loss of function.  Bladder and
bowel training programs will be necessary.  All of these interventions are important for this client but are not
the priority over maintenance of adequate oxygenation.

Educational objective:
The priority assessment in a client newly diagnosed with quadriplegia (tetraplegia) is airway management and
oxygenation.

Hepatic encephalopathy in cirrhosis results from higher serum ammonia levels that cause neurotoxic effects,
including mental confusion.  Oral lactulose is given to reduce the ammonia by trapping it in the gut and then
expelling it with a laxative effect.  Improved mental status implies reduction of ammonia levels.

(Option 1)  Clients with cirrhosis typically have hypokalemia due to hyperaldosteronism (as aldosterone is not
metabolized by the damaged liver).  Hypokalemia can also result from diuretics used to treat the fluid retention
and ascites.  Lactulose is not intended to treat this pathology.

(Option 3)  Lactulose is a laxative.  In cirrhosis, constipation (which allows more ammonia to be absorbed) and
hard stool (which irritates hemorrhoids) are to be avoided.  However, the main purpose of lactulose is expelling
the ammonia, with resulting benefits.

(Option 4)  Abdominal distension (ascites) in cirrhosis is treated with diuretics (eg, furosemide, spironolactone)
and paracentesis.  Lactulose does not influence this pathology or symptom.

Educational objective:
Lactulose is a laxative used to trap and expel ammonia in clients with cirrhosis who have hepatic
encephalopathy.  Elevated ammonia levels cause mental confusion.

Post-procedure care of a client who has undergone heart catheterization should focus on evaluating
hemodynamics - blood pressure, heart rate, strength of the distal pulses, color, and temperature of extremities. 
The client should be also assessed several times per hour for active bleeding, hematoma, or pseudoaneurysm
formation at the incision.
The first hour after cardiac catheterization requires assessment every 15 minutes.  Any report of back or flank
pain should be assessed for possible retroperitoneal bleeding as back pain, tachycardia, and hypotension may be
the only indication of internal bleeding.  More than a liter of blood can pool behind the peritoneum in the pelvis
undetected, and it may take up to 12 hours before a significant drop in hematocrit can be measured.  Internal
bleeding after cardiac catheterization is particularly dangerous due to frequent use of anticoagulant prescriptions
in these clients.

Clostridium difficile overgrowth in the intestine often occurs when normal gastrointestinal (GI) flora is
destroyed (eg, antibiotic use).  Clients with C difficile often have watery diarrhea, nausea, fever, and
abdominal pain.  Hypovolemia can easily develop through the loss of fluids and electrolytes in the stool,
especially in infants and the elderly.

Clients with hypovolemia from GI losses will often have hyponatremia, hypokalemia, and elevated blood
urea nitrogen (BUN) (poor renal perfusion).  This client has hyponatremia (normal, 135-145 mEq/L [135-145
mmol/L]), hypokalemia (normal, 3.5-5.0 mEq/L [3.5-5.0 mmol/L]), and an elevated BUN (normal, 6-20 mg/dL
[2.1-7.1 mmol/L]).  Hypovolemia can cause hypotension and renal failure, and electrolyte abnormalities can
cause cardiac arrhythmias; therefore, these are priority to report.  Fluid resuscitation and electrolyte replacement
should be initiated promptly to prevent complications.

(Options 3 and 4)  Expected findings of pneumonia include fever, chills, fatigue, crackles, and sputum
production.  There is nothing to indicate that the client is having respiratory difficulty.  Therefore, electrolyte
imbalance is a priority.

Educational objective:
Severe watery diarrhea can lead to hypovolemia.  Clients should be monitored for hyponatremia, hypokalemia,
elevated blood urea nitrogen, and other electrolyte imbalances.  These should be reported and treated
immediately to avoid cardiac arrhythmias, hypotension, and renal failure.

Thrombolytic agents (eg, alteplase, tenecteplase, reteplase) are often prescribed to resolve acute thrombotic
events (eg, ischemic stroke, myocardial infarction, massive pulmonary embolism).  They are recombinant
plasminogen activators that activate the blood fibrinolytic system and dissolve thrombi.

Thrombolytic agents are contraindicated in clients with active bleeding, recent trauma, aneurysm,
arteriovenous malformation, history of hemorrhagic stroke, and uncontrolled hypertension (blood pressure
>180/110 mm Hg).  Therefore, the health care provider should be consulted for clarification.  Administering
alteplase in the presence of these conditions can cause hemorrhage, including life-threatening intracerebral
hemorrhage (Option 1).

(Option 2)  Most penicillin derivates (eg, ampicillin, amoxicillin) and cephalosporins (eg, cephalexin,
ceftriaxone) are generally considered safe for use by women who are pregnant or lactating.

(Option 3)  Fentanyl is appropriate in postoperative clients with moderate to severe pain, even those with a
history of allergies to codeine.  Both drugs have opiate agonist effects but are chemically different.  Codeine is a
derivative of natural opiates (eg, morphine), whereas fentanyl is completely synthetic.

(Option 4)  Syndrome of inappropriate antidiuretic hormone (SIADH) secretion results in water retention and
dilutional hyponatremia.  Clients with SIADH often require hypertonic saline for sodium repletion to increase
serum sodium levels with a minimal infused volume of water.

Educational objective:
Thrombolytic agents (eg, alteplase, tenecteplase, reteplase) place clients at risk for bleeding.  Therefore, they
are contraindicated in clients with active bleeding, recent trauma, aneurysm, arteriovenous malformation,
history of hemorrhagic stroke, and uncontrolled hypertension.

Acute urinary retention is best treated with rapid, complete bladder decompression rather than the
intermittent urine drainage that is limited to 500 to 1000 mL at a time.  Rapid decompression can be associated
with hematuria, hypotension, and postobstructive diuresis (Option 2).  However, these are rarely clinically
significant if appropriate supportive care is administered, whereas inability to relieve the obstruction can be
associated with infection and kidney injury (Option 3).

(Option 1)  Dysuria from catheterization can be treated with analgesics or antispasmodic medications. 
Maintaining perfusion and adequate blood pressure is the priority concern.

(Option 4)  With sudden release of bladder obstruction, cardiovascular autonomic activity occurs and the blood
pressure and heart rate are reduced due to the excitation of the parasympathetic system.

Educational objective:
Acute urinary retention is best treated with rapid complete bladder decompression.  The nurse should carefully
assess for hypotension and bradycardia, which are potential complications.

Clients with sickle cell crisis often have excruciating pain and need large doses of narcotics.  The most effective method is
PCA of morphine or hydromorphone (Dilaudid).

Posttraumatic stress disorder (PTSD) is a reaction to a traumatic or catastrophic event that is typically life-
threatening to oneself or others.

There are 3 categories of PTSD symptoms:

1. Reexperiencing the traumatic event


Examples include intrusive memories, flashbacks, recurring nightmares, and feelings of intense
distress/loss of control or strong physical reactions to event reminders (rapid, pounding heart;
gastrointestinal distress; diaphoresis)

2. Avoiding reminders of the trauma


Examples include avoidance of activities, places, thoughts, or other triggers that could be trauma
reminders, feeling detached and emotionally numb, loss of interest in life, lack of future goals, and
amnesia related to important details of the event

3. Increased anxiety and emotional arousal


Examples include insomnia, irritability, outbursts of anger and/or rage, difficulty concentrating,
hypervigilance, and feeling jumpy

(Option 1)  Auditory hallucinations and feelings of paranoia are not characteristic symptoms of PTSD.  These
are characteristic of schizophrenia.

(Option 3)  Rapidly changing emotions, delusions, and lethargy are not characteristic symptoms of PTSD.

(Option 4)  Daytime sleepiness is not characteristic of PTSD.

Educational objective:
The 3 categories of PTSD symptoms include reexperiencing the traumatic event, avoiding reminders of the
trauma, and increased anxiety and emotional arousal.
Antiplatelet medication (eg, prasugrel [Effient], clopidogrel [Plavix], ticagrelor [Brilinta]) are often prescribed to a client
after a percutaneous coronary intervention such as angioplasty or stent placement.  These agents should be stopped at
least 5-7 days prior to the surgery to reduce the chance of intraoperative and postoperative bleeding.  The nurse should
immediately report to the HCP that the client is still taking prasugrel and took it the morning of the surgery.  Unless the
surgery is emergent, it will most likely be postponed at least a week.

The following reflect a client's physiologic state and not equipment malfunction:

Option 1:  "HI" is displayed when a client's glucose is too high (usually 500 mg or above).

Option 4:  When a client does not have adequate circulation/perfusion at the location of the sensor, the pulse
oximeter cannot locate an adequate pulsation and give a reading.

Option 5:  Ventilators sound an alarm to indicate high pressure when the machine is sensing increased
resistance.  The nurse should check to see if suctioning is needed (mucus causing resistance), if tubing is
bent/kinked, or if tension pneumothorax is present.  It is also possible that a client has a deteriorating lung
condition causing the high pressure alarm to sound.  The alarm is related to the client's condition and is not an
indication of ventilator malfunction.

Paracentesis is performed to remove excess fluid from the abdominal cavity or to provide a specimen of ascitic
fluid for diagnostic testing.  Paracentesis is not a permanent solution for resolving ascites and is performed only
if the client is experiencing impaired breathing or pain due to ascites.

Nursing actions include:

 Explain the procedure, sensations, and expected results


 Instruct the client to void to prevent puncturing the bladder
 Assess the client's abdominal girth, weight, and vital signs
 Place the client in high Fowler's position and remain with the client during the procedure
 After the procedure, assess and bandage the puncture site and reassess client weight, girth, and vital
signs

(Option 1)  NPO status is not required for this procedure.  Paracentesis is often performed at the bedside or an
HCP's office with only a local anesthetic.

(Option 2)  Informed consent can be obtained only by an HCP.  The nurse can witness informed consent
verifying that it is given voluntarily, the signature is authentic, and the client appears competent to consent.

Educational objective:
Paracentesis removes fluid from the abdominal cavity to improve symptoms or provide a specimen for testing. 
Nurses are expected to prepare the client (encourage voiding and place in high Fowler's position) and assess
symptoms, vital signs, weight, and abdominal girth before and after the procedure.

Clinical manifestations characteristic of moderate to severe asthma exacerbations include tachycardia


(>120/min), tachypnea (>30/min), saturation <90% on room air, use of accessory muscles to breathe, and peak
expiratory flow (PEF) <40% of predicted or best (<150 L/min).

Pharmacologic treatment modalities recommended by the Global initiative for Asthma (2014) to correct
hypoxemia, improve ventilation, and promote bronchodilation include the following:

1. Oxygen to maintain saturation >90%


2. High-dose inhaled short-acting beta agonist (SABA) (albuterol) and anticholinergic agent
(ipratropium) nebulizer treatments every 20 minutes
3. Systemic corticosteroids (Solu-Medrol)

(Option 4)  Montelukast (Singulair) is a leukotriene receptor blocker with both bronchodilator and anti-
inflammatory effects; it is used to prevent asthma attacks but is not recommended as an emergency rescue drug
in asthma.

(Option 5)  A long-acting beta agonist (Salmeterol) is administered with an inhaled corticosteroid for long-term
control of moderate to severe asthma; it is not used as an emergency rescue drug in asthma.

Educational objective:
Clinical manifestations characteristic of moderate to severe asthma exacerbations include tachycardia,
tachypnea, saturation <90% on room air, use of accessory muscles of respiration, and PEF <40% predicted. 
Management includes the administration of high-dose inhaled SABA and ipratropium nebulizer, systemic
corticosteroids, and oxygen to maintain saturation >90%.

Antiplatelet agents (eg, clopidogrel, ticagrelor, prasugrel, aspirin) prevent platelet aggregation and are given to
clients to prevent stent re-occlusion.  They prolong bleeding time and should not be taken by clients with a
bleeding peptic ulcer, active bleeding, or intracranial hemorrhage.  Ginkgo biloba also interferes with platelet
aggregation and can cause increased bleeding time.  Antiplatelet agents and Ginkgo biloba should not be
taken together.  If this were to occur, this client would be at an increased risk for bleeding.  This information
should be reported to the prescribing health care provider before the client is discharged.

(Option 1)  This blood pressure is slightly elevated, but is unaffected by antiplatelet agents.

(Option 2)  Normal heart rate is between 60/min-100/min.

(Option 3)  This is a normal platelet count (150,000/mm3-400,000/mm3 [150-400 x 109/L]).

Educational objective:
If a client is prescribed clopidogrel, the nurse should be concerned about a history of peptic ulcer disease and
Ginkgo biloba use.  In this situation, the client would be at increased risk for bleeding.  This data should be
reported to the prescribing health care provider before the client is discharged.

Cerebrospinal fluid (CSF) is assessed for color, contents, and pressure.  Normal CSF is clear and colorless, and
contains a little protein, a little glucose, minimal white blood cells, no red blood cells, and no microorganisms. 
Normal CSF pressure is 60–150 mm H2O.  Abnormal CSF pressure or contents can help diagnose the cause of
headaches in complicated cases.  CSF is collected via lumbar puncture or ventriculostomy.

Prior to a lumbar puncture, clients are instructed as follows:

1. Empty the bladder before the procedure (Option 2)


2. The procedure can be performed in the lateral recumbent position or sitting upright.  These positions
help widen the space between the vertebrae and allow easier insertion of the needle (Option 3).
3. A sterile needle will be inserted between the L3/4 or L4/5 interspace (Option 4)
4. Pain may be felt radiating down the leg, but it should be temporary (Option 1)

After the procedure, instruct the client as follows:


1. Lie flat with no pillow for at least 4 hours to reduce the chance of spinal fluid leak and resultant
headache
2. Increase fluid intake for at least 24 hours to prevent dehydration

Educational objective:
Lumbar puncture can be performed with clients in the sitting position or positioned on the left side with the
knees drawn up (fetal position).

Iron-deficiency anemia occurs when the body lacks sufficient iron to form red blood cells and synthesize
hemoglobin.  Iron-deficiency anemia can result from:

1. Diets low in iron (eg, vegetarian and low-protein diets)


2. Iron not being absorbed (eg, following many gastrointestinal [GI] surgeries, malabsorption syndrome)
3. Increased iron requirement (eg, children's growth spurts, pregnancy, breastfeeding)
4. Blood loss (eg, menstruation, bleeding in the GI tract [eg, ulcers, hemorrhoids])

Foods rich in iron include:

 Meats (eg, beef, lamb, liver, chicken, pork)


 Shellfish (eg, oysters, clams, shrimp)
 Eggs, green leafy vegetables, broccoli, dried fruits, dried beans, brown rice, and oatmeal

Eating foods rich in vitamin C (eg, citrus fruits, potatoes, tomatoes, green vegetables) with iron-rich foods will
enhance iron absorption but coffee and tea consumption interferes with this process.

A common risk factor for deep venous thrombosis (DVT) is traveling/sitting with prolonged periods (>4 hours) of
inactivity.  Common symptoms of a lower-extremity DVT include unilateral edema and calf pain.  Diagnosis and
treatment of DVT (circulation problem) is a high priority because a piece of the clot can break off, travel though the
systemic and/or pulmonary circulation, and cause a life-threatening complication (eg, pulmonary embolus).

Most bouts of diarrhea are self-limiting and last ≤48 hours.  Clients experiencing diarrhea that lasts >48 hours or
accompanied by fever or bloody stools should be evaluated by a health care provider (HCP).  Causes may include
infectious agents, dietary intolerances, malabsorption syndromes, medication side effects, or laxative overuse.  The HCP
will need to assess for dehydration and electrolyte imbalances and identify underlying causes of the diarrhea that may
require further treatment (eg, Clostridium difficile).

A Colles' fracture is a type of wrist fracture (distal radius fracture) that causes a characteristic dinner fork
deformity of the wrist.  It usually occurs when the client tries to break a fall with an outstretched arm or hand,
and lands on the heel of the hand.  It is one of the most common fractures in women age >50 and is related to
osteopenia or osteoporosis.

While the client is undergoing evaluation by the health care provider (HCP) in the emergency department (ED),
nursing interventions should include:

 Performing a neurovascular assessment (eg, pulse, temperature, color, capillary refill, sensation,
movement).  This is the priority nursing action as neurovascular insufficiency related to swelling (eg,
compartment syndrome) or arterial/nerve damage by the bone fragments is associated with a Colles'
fracture.  If neurovascular status is compromised, urgent reduction of the fracture is indicated.
 Administering analgesia to promote comfort (Option 1).
 Applying an ice pack to the wrist to help reduce edema and inflammation (Option 2).
 Elevating the extremity on a pillow above heart level to reduce edema (Option 4).
 Instructing the client to move the fingers to reduce edema, increase venous return, and help improve
range of motion.

Educational objective:
While a client with a traumatic wrist fracture is undergoing evaluation by the HCP in the ED to determine
appropriate treatment, the nurse assesses circulation, sensation, and movement of the affected hand, and then
performs nursing interventions to reduce pain and edema.

isotretinoin is an oral acne medication derived from vitamin A.  Due to teratogenic risk and severity of side
effects (eg, Stevens-Johnson syndrome, suicide risk), isotretinoin is used to treat only severe and/or cystic acne
not responding to other treatments.  Exposure to any amount of this medication during pregnancy can cause
birth defects.  Clients are required to enter a Web-based risk management plan (iPLEDGE) and use 2 forms of
contraception (Option 4).

Taking vitamin A supplements along with isotretinoin can cause vitamin A toxicity, which can cause increased
intracranial pressure, gastrointestinal upset, liver damage, and changes in skin and nails.  Therefore, clients
should be instructed to avoid vitamin A supplements while taking this medication (Option 3).

(Option 1)  Blood donation is also prohibited during the duration of treatment and up to a month after
treatment ends due to the possibility of inadvertent transfusion to a pregnant woman.

(Option 2)  Isotretinoin should not be taken with tetracycline because the latter also increases the risk for
intracranial hypertension.

Educational objective:
Isotretinoin is a vitamin A derivative prescribed to treat severe and/or cystic acne.  Side effects include birth
defects, skin changes (eg, dry skin, skin fragility, cutaneous atrophy), and risk for increased intracranial
pressure.  Clients need to be instructed to avoid tetracycline, excess sun and tanning, and vitamin A
supplements.  Women of child-bearing age should use 2 forms of contraception to prevent pregnancy.

HAP is a bacterial infection acquired in a health care facility that was not present on admission.  Almost all clients with
bacterial pneumonia develop leukocytosis (WBC >11,000/mm 3).  Antibiotic therapy is the first-line treatment, but
antibiotic resistance frequently occurs in HAP.  If antibiotic therapy is effective, clinical improvement usually occurs
within 3-4 days of initiation of IV antibiotics.

Cystic fibrosis is an inherited autosomal recessive disorder of the exocrine glands that results in physiologic
alterations in the respiratory, gastrointestinal, and reproductive systems.  It is theorized that the chloride
transport alternation and resulting thickened mucus inhibit normal ciliary action and cough clearance, and the
lungs become clogged with mucus.  The thickened mucus harbors bacteria.  Over time, airways develop chronic
colonization and frequent respiratory infections result.  Bronchial hygiene therapy, such as manual chest
physiotherapy, is used.  For physiotherapy, various positions are used, and this should be performed before
meals to avoid a full stomach and resultant regurgitation or vomiting.

(Option 1)  A white pupil (leukocoria, or cat's-eye reflex) is one of the first signs of retinoblastoma, an
intraocular malignancy of the retina.  Other symptoms include an absent red reflex, asymmetric or of a differing
color in the affected eye, and fixed strabismus (constant deviation of one eye from the other).  This disease is
not related to cystic fibrosis.

(Option 3)  Hypercyanotic episodes are associated with tetralogy of Fallot.  The knee-chest position increases
systemic vascular resistance in the lower extremities.  In addition, irritating stimuli should be limited, and
supplemental oxygen should be provided.
(Option 4)  The pancreatic ducts become damaged, and there is a decreased ability to digest fats and proteins
and absorb fat-soluble vitamins.  Pancreatic enzyme supplements are used.  Children with cystic fibrosis tend to
be hungry but underweight due to a decreased ability to use fat and its calories.

Educational objective:
Cystic fibrosis causes thickened mucus, making respiratory infections common.  Treatment includes chest
physiotherapy performed usually before meals.

This client with heart failure who is short of breath and coughing up pink frothy sputum has developed acute pulmonary
edema (fluid filling the alveoli), a potentially life-threatening condition.

Trousseau's sign may indicate hypocalcemia before other signs and symptoms of hypocalcemia, such as tetany,
occur.  Trousseau's sign can be elicited by placing the BP cuff on the arm, inflating to a pressure > than systolic
BP, and holding in place for 3 minutes.  This will occlude the brachial artery and induce a spasm of the muscles
of the hand and forearm when hypocalcemia is present.

Chevostek's sign another early indicator of hypocalcemia, should also be assessed.  It may be elicited by
tapping the face at the angle of the jaw and observing for contraction on the same side of the face.

(Option 1)  Phalen's maneuver is used to diagnose carpal tunnel syndrome.

(Option 2)  The heel-to-shin test is another means of assessing cerebellar function.  An abnormal examination
is evident when the client is unable to keep the foot on the shin.

(Option 3)  The Romberg test is a component of a neurological examination to assess vestibular function
(perception of head position in space), proprioception (perception of the body in space), and vision.

Hypospadias is a congenital defect in which the urethral opening is on the underside of the penis.  Except in
very mild cases, the condition is typically corrected around age 6-12 months by surgically redirecting the
urethra to the penis tip.  Circumcision is delayed so the foreskin can be used to reconstruct the urethra.  If not
corrected, clients may have toilet-training difficulties, more frequent urinary tract infections, and inability to
achieve erections later in life.

Postoperatively, the client will have a catheter or stent to maintain patency while the new meatus heals. 
Urinary output is an important indication of urethral patency.  Fluids are encouraged, and the hourly output is
documented.  Absence of urinary output for an hour indicates that a kink or obstruction may have occurred

A seizure is an uncontrolled electrical discharge of neurons in the brain that interrupts normal function.  Seizure
manifestations generally are classified into 4 phases:

1. The prodromal phase is the period with warning signs that precede the seizure (before the aural phase).
2. The aural phase is the period before the seizure when the client may experience visual or other sensory
changes.  Not all clients experience or can recognize a prodromal or aural phase before the seizure.
3. The ictal phase is the period of active seizure activity.
4. During the postictal phase, the client may experience confusion while recovering from the seizure. 
The client may also experience a headache.  Postictal confusion can help identify clients by
differentiating seizures from syncope.  In syncope, there will be only a brief loss of consciousness
without prolonged post-event confusion.
Educational objective:
Clients may experience confusion after a seizure during the postictal phase.  The client should be observed for
safety and abnormalities documented before and during this phase.

Infant growth is fast paced during the first year of life, with birth weight doubling by age 6 months and
tripling by age 12 months.  During the first year, birth length increases by approximately 50%.  At birth, head
circumference is slightly more than chest circumference, but these equalize by age 12 months.

(Options 1 and 2)  At birth, the infant has non-ossified membranes called fontanelles; these "soft spots" lie
between the bones of the cranium.  The 2 most noticeable are the anterior and posterior fontanelles, which are
soft and non-fused.  Fontanelles should be flat, but slight pulsations noted in the anterior fontanelle are normal
as is temporary bulging when the infant cries, coughs, or is lying down.  The posterior fontanelle fuses by age 2
months, and the anterior fontanelle fuses by age 18 months.

(Option 4)  This assessment shows tripling of the birth weight by age 12 months, a normal finding.

Educational objective:
Infants should double in birth weight by age 6 months and triple in birth weight by age 12 months.  At birth,
head circumference is slightly more than chest circumference, but these equalize by age 12 months.  The
posterior fontanelle fuses by age 2 months, and the anterior fontanelle fuses by age 18 months.

Tinea corporis (ringworm) is a fungal infection of the skin often transmitted from one person to another or
from an infected animal to a human.  It appears as a scaly, pruritic patch that is often circular or oval in shape. 
It is highly contagious and can be spread via items such as grooming tools, hats, towels, and bedding.  Tinea
corporis often spreads via shared athletic equipment or in athletic locker rooms due to the proximity of infected
gear.  This condition is treated with topical antifungals (eg, tolnaftate, haloprogin, miconazole, clotrimazole).

(Option 1)  Eczema is a skin rash caused by an immune disorder that is often triggered by an allergy.  Itching is
common, but the rash is not contagious.

(Option 2)  Oral candidiasis, or thrush, often occurs after a course of antibiotics or corticosteroids or can occur
in infants with immature immune systems.  An infant who is breastfed can transfer candidiasis to the mother's
breast.  There is also a small risk of transmission when infants place pacifiers or toys in their mouths and
subsequently transfer these items to another child's mouth.  However, oral candidiasis is significantly less
contagious than tinea corporis.

(Option 3)  Psoriasis is a chronic autoimmune disease that most often affects the skin by causing dry, scaly, red
rashes.  Psoriasis is not contagious.

Educational objective:
Tinea corporis (ringworm) is a highly contagious fungal skin infection.  It is treated with topical antifungals. 
Clients should be instructed not to share items such as grooming tools, towels, bedding, and hats.  Athletic
equipment should be cleaned routinely.

Gentle, continuous bubbling in the suction control chamber (section A) of a chest tube drainage unit indicates
that suction is present and the unit is functioning appropriately.  The nurse should document the finding and
continue to monitor.

(Option 1)  Subcutaneous emphysema is air that has leaked into the tissue surrounding the chest tube insertion
site.  A crackling sensation is felt when palpating the skin.  It does not affect bubbling within the chest tube
drainage unit.
(Option 2)  An air leak would cause bubbling in the air leak gauge (section C) or water seal chamber not in the
suction control chamber.

(Option 4)  Turning down the wall suction would effectively negate the presence of suction in the chest tube
drainage unit.

Educational objective:
Gentle, continuous bubbling in the suction control chamber of a chest tube drainage unit indicates the presence
of suction in the system and is an expected finding.

Large amounts of vitamin K-rich foods can decrease the anticoagulant effects of warfarin therapy.  Clients
are not instructed to remove those foods from their diet but are encouraged to be consistent in the intake of
foods high in vitamin K, including leafy green vegetables, asparagus, broccoli, kale, Brussels sprout, and
spinach.

Several beverages also affect warfarin therapy.  Green tea, grapefruit juice, and cranberry juice may alter its
anticoagulant effects.

Neonates are unable to generate heat by shivering due to their lack of muscle tissue and immature nervous
systems; they therefore produce heat by increasing their metabolic rates through nonshivering thermogenesis. 
Brown adipose tissue (BAT), developed during the third trimester, is metabolized for thermogenesis when
available.  Once BAT is depleted, nonshivering thermogenesis is less effective and the neonate may experience
cold stress, possibly leading to death.  Preterm neonates have fewer stores of BAT and are at higher risk for cold
stress.  Frequent temperature monitoring is the best method to assess if an infant is cold.

In cold stress, metabolism increases to generate heat, causing a greater demand for oxygen and glucose and the
release of norepinephrine.  If adequate oxygenation is not maintained, hypoxia and acidemia occur. 
Hypoglycemia develops when available glucose is depleted, and repletion of glucose is impaired by
gastrointestinal immotility and poor oral intake.

Clinical manifestations of cold stress include:

 Neurological - altered mental status (irritability or lethargy) (Option 1)


 Cardiovascular - bradycardia
 Respiratory - tachypnea early, followed by apnea and hypoxia
 Gastrointestinal - high gastric residuals, emesis, hypoglycemia (Option 2)
 Musculoskeletal - hypotonia, weak suck and cry (Option 4)

Educational objective:
Premature infants are at high risk for cold stress due to decreased brown adipose tissue and inability to generate
heat by shivering.  The nurse should carefully assess for signs of cold stress, which include decreased
temperature, altered mental status, bradycardia, hypoxia, hypotonia, and a weak cry and/or suck.

To provide full support when climbing stairs, clients should hold the cane on the stronger side and move the
cane before moving the weaker leg, regardless of the direction of the stairs (Option 2).  They should also keep
2 points of support on the floor at all times (eg, both feet, cane and foot) and face forward when going up or
down the stairs, especially if there is no handrail (Option 1).  The nurse should instruct the client on the
following:

When ascending stairs:


1. Step up with the stronger leg first (in this client, the right leg)
2. Move the cane next while bearing weight on the stronger leg
3. Finally, move the weaker leg (in this client, the left leg)

When descending stairs:

1. Lead with the cane


2. Bring the weaker leg down next
3. Finally, step down with the stronger leg

The nurse may use the mnemonic "up with the good and down with the bad."  The cane always moves before
the weaker leg.

(Option 4)  Clients are usually hospitalized for 3-4 days following a total knee replacement and can bear full
weight by the time of discharge.  Early ambulation and weight-bearing helps to hasten recovery and prevent
complications (eg, thromboembolism).

Educational objective:
Clients who have had total knee replacement surgery can typically bear full weight by the time of discharge.  To
reduce the risk of falls, the client should hold the cane on the stronger side and face forward when going up and
down the stairs.  To ascend the stairs, the client should first step up with the stronger leg, next bear weight on
that leg and move the cane, and finally step up with the weaker leg.

A lumbar puncture involves removing a sample of cerebrospinal fluid through a needle inserted between
vertebrae.  Elevated intracranial pressure is a contraindication to performing a lumbar puncture.  The client is
placed in the fetal position or sitting and leaning over a table.  Continued leaking fluid indicates that the site
did not seal off and a blood patch (autologous blood into the epidural space) is required.

(Option 1)  Fluids are encouraged to help replace the cerebrospinal fluid.

(Option 3)  The client should lie flat for at least 4 hours.  The prone or supine position is recommended to
help prevent a headache.

(Option 4)  Up to 5%-30% of clients have the common complication of headache.  It is thought to be a result
of leakage of fluid through the dural puncture site.  The symptom is treated and is normally self-limiting.

Educational objective:
After a lumbar puncture, cerebrospinal fluid leakage from the puncture site requires health care provider
notification for a blood patch.  A headache after the procedure is an expected finding.  The client should lie flat
and increase fluid intake afterwards.

This client is experiencing third-degree atrioventricular (AV) block, or complete heart block, which involves
complete inhibition of impulse conduction from the atria to the ventricles, usually at the AV node or bundle of
His.  The atrial and ventricular rhythms are regular but unrelated to each other.  A complete heart block results
in bradycardia, decreased cardiac output, syncope, and possibly heart failure/shock.  The client is typically
symptomatic and requires immediate treatment with transcutaneous pacing until a permanent pacemaker can
be inserted.  Atropine, dopamine, and epinephrine can be used to increase heart rate and blood pressure until
temporary pacing is available.

(Option 1)  A fluid bolus is warranted in a client who is hypotensive from hypovolemia; however, this client is
hypotensive due to a cardiac rhythm disturbance and slow heart rate.
(Option 2)  Adenosine is given rapidly via IVP for treatment of rapid tachycardic rhythms such as paroxysmal
supraventricular tachycardia (PSVT).

(Option 3)  Cardioversion is used in clients with tachydysrhythmias (eg, ventricular tachycardia with a pulse,
supraventricular tachycardia (SVT), or atrial fibrillation with a rapid ventricular response) who have been
unresponsive to medications or are hemodynamically unstable.

Educational objective:
A client in complete heart block is often bradycardic and hemodynamically unstable.  Transcutaneous pacing
should be used until a permanent pacemaker can be inserted.  Atropine, dopamine, or epinephrine may be used
to increase heart rate and blood pressure until temporary pacing is started.

Acute gastroenteritis is associated with nausea, vomiting, diarrhea, and dehydration.  An isotonic crystalloid
intravenous (IV) solution (eg, 0.9% normal saline, lactated Ringer's) has the same tonicity as plasma and when
infused remains in the vascular compartment, quickly increasing circulating volume.  It is appropriate to correct
the extracellular fluid volume deficit (dehydration) in this client.

(Option 1)  A hypertonic, rather than isotonic, solution would be infused in clients with ICP.  Increasing
circulating volume would only further increase ICP.

(Option 3)  Isotonic solutions can exacerbate fluid overload in chronic renal failure and increase blood
pressure.

(Option 4)  Clients with severe hyponatremia and neurologic manifestations need rapid correction of
hyponatremia with hypertonic saline (3% saline).

Educational objective:
Depending on the type/tonicity of intravenous (IV) solution infused, fluids can remain in the vascular
compartment or can shift from the extracellular to intracellular compartments, and vice versa.  The nurse must
be able to assess which type of IV fluid is appropriate in relation to a client's diagnosis and condition.

Hypoglycemia (blood glucose <70 mg/dL [3.9 mmol/L]) is an acute and sometimes life-threatening
complication of diabetes mellitus.  It occurs when the proportion of insulin exceeds the glucose in the blood. 
Counterregulatory hormones (eg, epinephrine) are then released and the autonomic nervous system is activated.

The 40-year-old experienced bicycle rider who increased his exercise routine has the highest risk of developing
hypoglycemia.  Aerobic exercise typically lowers blood glucose levels.  Physiologically, glucose production in
the liver fails to keep up with elevated glucose uptake by the muscles at work.  Even an experienced exerciser
should check blood glucose levels before, during, and after exercise and also carry a carbohydrate drink or
snack in case of a hypoglycemic episode.  Clients who exercise should have a snack to prevent hypoglycemia
when they increase their exercise level.

A client with an infection (Option 1) or cellulitis (Option 3) is more likely to experience hyperglycemia
instead.  The physiologic stress response caused by the infection will raise the blood glucose.  Hyperglycemia is
also a side effect of prednisone (Option 4).

Educational objective:
Aerobic exercise typically lowers blood glucose levels.  Physiologically, glucose production in the liver fails to
keep up with elevated glucose uptake by the muscles at work.
The radial artery site at the wrist is preferred for collecting an arterial blood gas sample because it is near the
surface, is easy to palpate and stabilize, and has good collateral supply from the ulnar artery.  The patency of the
ulnar artery can be confirmed with a positive modified Allen's test.

The modified Allen's test includes the following steps:

 Instruct the client to make a tight fist (if possible)


 Occlude the radial and ulnar arteries using firm pressure
 Instruct the client to open the fist; the palm will be white if both arteries are sufficiently occluded
 Release the pressure on the ulnar artery; the palm should turn pink within 15 seconds as circulation is
restored to the hand, indicating patency of the ulnar artery (positive Allen's test)

If the Allen's test is positive, the arterial blood gas can be drawn; if negative and the palm does not return to a
pink color, an alternate site (eg, brachial artery, femoral artery) must be used.

(Option 1)  Capillary refill is tested by applying pressure to the fingernail bed to cause blanching.  If refill is
adequate, the nail bed should become pink in less than 3 seconds after pressure is released.

(Option 2)  The radial artery is palpated with the fingertips to determine the presence of the radial pulse.

(Option 4)  A neurologic deficit is assessed by monitoring color, sensation, and movement of the hand.

Educational objective:
The radial artery site at the wrist is preferred for collecting an arterial blood gas sample because it is near the
surface, easy to palpate and stabilize, and has good collateral supply from the ulnar artery.  The patency of the
ulnar artery must be confirmed by performing a modified Allen's test to assure adequate circulation to the hand
before proceeding with the arterial blood gas collection.

The phlebostatic axis is an external anatomical point on the chest at the level of the atria of the heart (fourth
intercostal space at the midaxillary line or midway point of the anterior posterior diameter of the chest).  It is
used as a reference point for correct placement of the zeroing point of the transducer when measuring continual
arterial blood pressure (BP), central venous pressure (CVP) using a central line, and/or cardiopulmonary
pressures via a pulmonary artery (Swan-Ganz) catheter.  The nurse places the transducer and marks the chest at
the phlebostatic axis, which helps to assure accuracy of measurement.  After it is placed, the zero reference
stopcock of the transducer is "leveled," or aligned with the level of the atrium, using a ruler or carpenter's level. 
If the zeroing stopcock is placed below this level, falsely high readings occur; if it is too high, falsely low
readings are obtained.

The phlebostatic axis is also used as a reference point for the upper arm when measuring BP indirectly using a
noninvasive BP device or the auscultatory method with sphygmomanometer and stethoscope.  If the upper arm
is above or below this level, the BP reading will be inaccurate.

Educational objective:
The phlebostatic axis (fourth intercostal space at the midaxillary line or midway point of the anterior posterior
diameter of the chest) is an external reference point on the thorax used to determine proper placement of the
pressure monitoring system transducer when measuring direct BP, CVP, and/or cardiopulmonary pressures
invasively.  It is also used as a reference point for the upper arm when measuring BP indirectly.

Violence in the health care setting poses a safety risk to clients, staff, and visitors.  It also decreases the quality
of care that a violent client receives due to avoidant and fearful behaviors by staff.  Risk factors for violence
include altered level of consciousness, substance abuse, emotional stress, and behavioral/psychiatric disorders.
Nursing interventions that help prevent violence include clear, thorough communication (Option 2);
encouraging active participation in care; promoting a low-stimulation environment; and providing comfort
through pharmacological and nonpharmacological methods.  The nurse should demonstrate undivided attention
to the client (eg, facing the client, unhurried body language, calm tone).

(Option 1)  Administering lorazepam is appropriate to reduce agitation; however, this is a reaction to
behavioral precursors of violence and does not prevent violence.

(Option 3)  Placing the client near the nurses' station may increase anxiety due to the noise and activity in that
area.  The client should be closely monitored, but this is not the most effective intervention to prevent violence.

(Option 4)  The presence of security personnel does not prevent violence and may cause increased client
anxiety.  The nurse should consider other interventions first (eg, effective communication) to directly prevent
violence.

Educational objective:
Violence is a safety concern for all in the health care setting.  Nurses must identify those at risk for violent
behavior and use clear, thorough communication to prevent violence.  The nurse should provide undivided
attention to the client while explaining all activities of care in a calm tone.

The decision about providing artificial nutrition to a dying client is complex.  Although certain situations
involving terminal illness, such as a terminally ill client who wants to attend an important family function, can
justify the decision to provide IV fluids, providing artificial hydration in other situations may not be justified
and may even be harmful.  Ethical principles dictate that client preferences should be respected and that
clients/family members have the right to make decisions about artificial nutrition and hydration at the end of
life.

(Option 1)  Artificial hydration does not seem to help dying clients feel more comfortable, and IV fluids could
cause distressful symptoms such as respiratory distress, vomiting and diarrhea, and the need for urinary
catheterization.

(Option 3)  The majority of hospice and palliative health care providers do not recommend routine
administration of artificial hydration.

(Option 4)  There is no evidence that withholding artificial hydration at the end of life speeds up the dying
process.  Research indicates that dying clients who do not receive artificial hydration live just as long as those
who do receive IV fluids.

Educational objective:
The majority of hospice and palliative health care providers do not recommend routine administration of
artificial hydration; however, client preferences should be respected – clients/family members have the right to
make decisions about artificial nutrition and hydration at the end of life.

Disaster events cause a sudden increase in admissions to local hospitals.  The nurse identifies clients who are
safe to recommend for discharge to make room for newly admitted clients.

A client with acute asthma exacerbation may require treatment in the emergency department or hospitalization
for oxygen, inhaled bronchodilators, and corticosteroids.  The client can likely be discharged home when
respiratory status has stabilized and continue the previous home regimen of inhaled bronchodilators and
corticosteroids (Option 2).
(Option 1)  Clients who have received chemotherapy may be immunocompromised due to neutropenia.  An
immunocompromised client is at greater risk of sepsis from an infection.  Close monitoring and antibiotic
therapy are required.

(Option 3)  Clients with diabetes may develop diabetic ketoacidosis (DKA) during illness or infection. 
Features of DKA (eg, lethargy, abdominal pain, hyperglycemia, urine ketones) are a medical emergency. 
Untreated DKA may progress to loss of consciousness and coma.  Treatment includes frequent laboratory
monitoring and IV insulin, fluids, and potassium.

(Option 4)  Clients with ulcerative colitis are at risk for developing toxic megacolon (ie, severe inflammatory
colon distension).  Symptoms include fever, nausea, vomiting, pain, and abdominal distension.  Clients require
close monitoring, nasogastric tube for decompression, IV fluids, and antibiotics.  Emergency surgery may be
required.

Educational objective:
In response to a local disaster, the nurse identifies clients who can be safely discharged to make room for newly
admitted clients.  A client with acute asthma exacerbation can be safely discharged home when respiratory
status has stabilized.

The most common bacterial infection during pregnancy is a urinary tract infection (UTI).  Pregnant women
are predisposed to UTIs due to physiological changes (urine stasis) in the renal system.  Symptoms include
frequency, dysuria, urgency, foul-smelling urine, sediment/pus/blood in the urine, or sensation of bladder
fullness.  Diagnosis is made on signs/symptoms and urinalysis.  The prescribed antibiotic course must be
completed to treat the infection appropriately.  The priority is to deal with the current infection.  If the UTI is
untreated, the infection can lead to pyelonephritis or premature labor.

(Option 1)  Cranberry juice works by preventing the attachment of bacteria to the epithelial cells in the bladder
wall.  It is recommended that clients drink 8 ounces of unsweetened juice 3 times a day.  Although this is
reasonable teaching, it is not the priority at this time.

(Option 2)  This is good general advice to help prevent UTIs as the most common causative organism is
Escherichia coli.  However, completing a urinalysis and initiating antibiotic therapy should be implemented
before instructions related to UTI prevention.  Actual over potential is always the priority.

(Option 3)  Most women experience urinary symptoms of frequency (voiding >7 times/day), urgency, and
nocturia (voiding >2 times/night) beginning early in pregnancy and continuing to some degree throughout
pregnancy.  These are primarily a result of hormonal changes and may be due to anatomical changes in the renal
system.  They are not indicative of UTI.  However, the client has new, additional symptoms of a UTI (dysuria,
foul-smelling urine).  The client should be tested and treated with an antibiotic.

Educational objective:
Signs/symptoms of a UTI in pregnancy include new-onset dysuria and sediment/cloudy, foul-smelling urine. 
This differentiates it from the common urinary frequency throughout pregnancy.  The priority for the nurse is to
identify the need for a urinalysis and to provide instructions to finish the course of antibiotics to prevent the
untreated infection from causing pyelonephritis or premature labor.

When a pregnant client arrives and birth is imminent, the nurse should focus on collecting a brief, focused
history to elicit key information relevant to potential neonatal resuscitation.  Essential areas of history-taking
include:

 Multiple gestation:  To prepare for the potential of multiple newborn resuscitations (Option 1)
 Meconium-stained amniotic fluid:  To prepare for potential intubation and tracheal suctioning (Option
2)
 Narcotic/illicit drug use (especially within the last 4 hours):  To anticipate respiratory depression
(Option 3)
 Preterm labor/birth:  To anticipate respiratory immaturity and neonatal ventilation (Option 4)

(Option 5)  At this point, birth is imminent, and the name of the health care provider is not immediately
pertinent.  After the client has given birth, the nurse may attempt to obtain prenatal records from the health care
provider, if available.

Educational objective:
When a client arrives and birth is imminent, the nurse's priority is collecting a brief history to elicit key
information relevant to potential neonatal resuscitation.  Identifying multiple gestation, preterm gestational age,
meconium-stained fluid, and recent narcotic/drug use are essential areas of concern.

A vaginal hematoma is formed when trauma to the tissues of the perineum occurs during delivery.  Vaginal
hematomas are more likely to occur following a forceps- or vacuum-assisted birth or an episiotomy.  The client
reports persistent, severe vaginal pain or a feeling of fullness.  If the client had epidural anesthesia, pain may
not be felt until the effects have worn off.  Vaginal bleeding is unchanged.  The uterus is firm and at the midline
on palpation.  If the hematoma is large, the hemoglobin level and vital signs can change significantly.  In a
client with epidural analgesia, a change in vital signs may be an important indicator of hematoma.

(Option 1)  Cervical lacerations should be suspected if the uterine fundus is firm and midline on palpation
despite continued vaginal bleeding.  The bleeding can be minimal to frank hemorrhage.  Severe pain or a feeling
of fullness is not associated with cervical lacerations.

(Option 2)  Complete inversion of the uterus presents with a large, red mass protruding from the introitus.

(Option 3)  Uterine atony presents with a boggy uterus on palpation and an increase in vaginal bleeding.

Educational objective:
Vaginal hematomas are formed following trauma to the tissues of the perineum during vaginal delivery (eg,
vacuum- or forceps-assisted delivery, episiotomy).  The client reports severe pain or a persistent feeling of
fullness in the region.  Assessment shows a firm, midline uterine fundus with minimal or unchanged vaginal
bleeding.

Providing care establishes a legal caregiver obligation/relationship between the nurse and the visitor.  If a
relationship is started, the nurse has a duty to continue care until the visitor is stable or other health care
personnel can take over.  If proper care is not continued, the nurse could be accused of negligence (ie, failure to
act in a prudent manner as would a nurse with similar education/experience).

This visitor's symptoms are potentially serious as sudden onset of headache and numbness in half of the body
may indicate stroke.  In the event of a visitor emergency, the nurse should not establish a caregiver relationship
but rather implement facility protocol to help the visitor get to the emergency department promptly to receive
immediate assessment and further evaluation (Option 2).

(Options 1 and 4)  Asking the visitor to call the health care provider (HCP) or giving advice to lie down delays
the essential assessment and treatment that this visitor with potentially serious symptoms requires.
(Option 3)  When a nurse provides care (eg, takes blood pressure), a client-caregiver relationship is
established.  The nurse caring for a visitor is ill-equipped to provide care without any HCP prescriptions in
place and risks being negligent.

Educational objective:
Providing care establishes a legal caregiver obligation/relationship between the nurse and a visitor.  In the event
of a visitor emergency, the nurse should refrain from actions that establish this relationship and instead
implement facility protocol to help get the visitor promptly to the emergency department.

Pericardial effusion is a buildup of fluid in the pericardium.  Tamponade, a serious complication of pericardial
effusion, develops as the effusion increases in volume and results in compression of the heart.  The heart
struggles to contract effectively against the fluid, and cardiac output can decrease drastically.  This life-
threatening complication requires an emergency pericardiocentesis (a needle inserted into the pericardial sac to
remove fluid).

Signs and symptoms of cardiac tamponade include:

 Hypotension with narrowed pulse pressure (Option 1)


 Muffled or distant heart tones (Option 4)
 Jugular venous distension (Option 5)
 Pulsus paradoxus
 Dyspnea, tachypnea
 Tachycardia

(Option 2)  Bounding pulses may be present during fluid overload or hypertension.  They may also be present
with anxiety or fever.  The client with possible tamponade will have evidence of decreased cardiac output and is
more likely to have weak, thready pulses.

(Option 3)  Decreased breath sounds on the left side are not specific to the development of cardiac tamponade. 
Decreased breath sounds could indicate conditions such as atelectasis, pleural effusion, or pneumothorax.

Educational objective:
The client with a moderate to large pericardial effusion is at risk for the development of cardiac tamponade. 
Signs and symptoms of tamponade include muffled or distant heart tones, narrowed pulse pressure, jugular
venous distension, pulsus paradoxus, dyspnea, tachypnea, and tachycardia.  The nurse should report these
findings to the health care provider immediately and prepare for a pericardiocentesis.

Inconsolable crying and drawing up of the legs toward the abdomen in a child age 6-36 months could indicate
intussusception or some other abdominal pathology (eg, appendicitis).  Additional findings in intussusception
include stools that have mucus and blood, often called "currant jelly" stools, and vomiting.  Intussusception
occurs when one section of bowel telescopes over another, which can block the passage of intestinal contents,
interrupt blood supply, and cause intestinal tears (perforation).  It is an emergency, and the client should be
brought to the emergency department for further evaluation.

(Option 1)  Mild to moderate fever and local reactions are common after diphtheria, tetanus, acellular pertussis
(DTaP) injections.  Severe allergic reaction (eg, anaphylaxis) and encephalopathy (eg, decreased level of
consciousness, prolonged seizures) are the most serious reactions that require priority attention.

(Option 3)  Pneumonia is often accompanied by chest and side pain that worsens with deep breathing due to
rubbing of the nearby inflamed pleura (pleuritis).  This would not be the priority phone call.
(Option 4)  These symptoms are consistent with bacterial conjunctivitis, or inflammation of the clear membrane
(conjunctiva) that covers the eye.  This client is second in priority.

Educational objective:
Intussusception occurs when one section of bowel telescopes over another.  Inconsolable crying, drawing up of
the legs toward the abdomen, and "currant jelly" stools (mixed with blood and mucus) are the classic findings. 
It is an emergency and can lead to bowel obstruction, decreased blood supply, and perforation.

The client should receive aspirin (Option 1), docusate sodium (Option 2), and lisinopril (Option 3).

Aspirin is administered as an antiplatelet medication to clients with heart disease.  It is effective in reducing
inflammation and inhibits platelet aggregation.  The client's platelet count is within normal range (150,000–
400,000/mm3 [150-400 × 109/L]) and the hemoglobin and hematocrit are normal, so it is safe to administer this
medication.

Docusate sodium is a stool softener.  The client who has had a myocardial infarction (MI) should not strain
during bowel movements due to the risk of producing a vagal response, putting the client at risk for
bradycardia and other dysrhythmias.

Lisinopril is an angiotensin-converting enzyme (ACE) inhibitor commonly given to clients post MI to


prevent ventricular remodeling (hypertrophy) and the progression of heart failure.  Because ACE inhibitors
have the potential to cause hyperkalemia, the nurse should assess the potassium level when available.  This
client's potassium level is within normal range (3.5–5.0 mEq/L [3.5-5.0 mmol/L]).  The nurse should check the
client's blood pressure (BP) prior to administration as ACE inhibitors can lower BP.  This client's BP is within
the normal range.

(Option 4)  Metoprolol is a beta blocker given to clients post MI to reduce the risk of reinfarction and the
occurrence of heart failure.  A side effect of this medication is bradycardia (<60/min).  This client is
experiencing bradycardia with a heart rate of 52/min.  The nurse should hold this medication and notify the
health care provider.

(Option 5)  Simvastatin is a lipid-lowering medication given to all clients with elevated triglycerides and LDL
cholesterol.  This medication is not due at 0900.

Educational objective:
Blood pressure and serum potassium level are checked prior to administration of ACE inhibitors.  Heart rate
should be checked prior to administration of beta blockers.  Aspirin is given to clients with normal platelet
counts.

Signs of complications after a prostatectomy, such as bleeding, passage of blood clots, a decrease in the urinary
stream, urinary retention, or symptoms of a urinary tract infection, should be reported to the HCP for further
evaluation.

(Option 2)  The passage of blood clots may indicate bleeding from the prostatic fossa.  The client requires
further evaluation.

(Option 3)  While the prostatic tissue is healing, the Valsalva maneuver should be avoided as it may increase
venous pressure and produce hematuria.

(Option 4)  Maintaining adequate fluid intake to prevent dehydration, which increases the tendency for a blood
clot to form, is important.  However, this client is currently reporting blood clots and needs further evaluation.
Educational objective:
Following a prostatectomy, bleeding is a potential complication that requires a thorough assessment.  Any
bleeding, passage of clots, decrease in urinary stream, urinary retention, or symptoms of urinary tract infection
should be reported to the HCP.

Enteric-coated drugs have a barrier coating that dissolves at a slower rate (usually in the small intestine) to
protect the stomach from irritant effects.  Crushing enteric-coated medications (eg, ibuprofen) disrupts the
barrier coating and may cause stomach irritation.  In addition, the particles from the coating may clog the NG
tube, particularly small-bore NG tubes.

Slow-, extended-, or sustained-release drug formulations are designed to dissolve very slowly within a
specific time frame.  Crushing these medications alters this property and introduces the risk of adverse effects
from toxic blood levels due to more rapid drug absorption.  Therefore, the nurse should first contact the PHCP
for clarification.

(Options 2 and 4)  Double- and extra-strength drugs such as sulfamethoxazole and acetaminophen may be
crushed and administered separately through an NG tube as long as they are not enteric-coated.  The nurse
should flush the tube with water before and after each drug administration.

Educational objective:
Crushing an enteric-coated, slow-release, extended-release, or sustained-release drug disrupts its designed time
of release and is contraindicated.  The nurse should contact the PHCP for an alternate prescription if such a drug
is prescribed via NG route.

Giving oral medications to infants requires specialized techniques for safe administration.  A plastic,
disposable oral syringe can be used for accurate dosing and ease of delivery (Option 4).  Oral medication
should be administered with the infant in a semi-reclining position, which is similar to the feeding position
(Option 3).  This position promotes comfort, prevents aspiration, and may be better controlled by the nurse if
the infant resists the medication.  Liquid medications administered by oral syringe should be directed toward the
back and inside of the infant's cheek (Option 2).  The medication should be dispensed slowly in small
amounts, allowing the infant to swallow between squirts to prevent aspiration.

(Option 1)  Medications are never mixed in a bottle of infant formula as this can affect the taste and the infant
may then refuse the formula in the future.  In addition, if the infant does not complete the full feed, underdosing
will occur.

(Option 5)  Pinching the nose shut during medication administration may cause aspiration.  The infant's
mouth should be opened by applying gentle pressure to the chin or cheeks.

Educational objective:
Disposable oral syringes are the preferred tool to administer oral medications to infants.  Infants should be held
in a semi-reclining position, and medications should be given slowly in small amounts directed toward the back
and inside of the cheek.

Although this elderly client may be unconscious due to intoxication, vomit and decreased level of consciousness
place this client at risk for airway obstruction.  Treatment of this client is a priority, and measures must be taken
to protect the airway (eg, rescue position, head of bed elevation, intubation).

(Option 1)  Bacterial infection is the most common cause of COPD exacerbation.  Although clients with COPD
usually have cough and sputum, it becomes a concern when the sputum changes in color, consistency, or
volume.  This client needs antibiotics.  The goal pulse oximetry reading for COPD is typically 90%-93% as
many clients with COPD rely on their hypoxemic drive to breathe.  Therefore, this client is stable and can wait
until the unconscious elderly client is treated.

(Option 2)  This child has fifth disease ("slapped-cheek," erythema infectiosum), which is caused by parvovirus
B19.  Symptoms, in addition to a bright-red facial rash, include fever and general flulike symptoms.  It is
harmless unless the client has a hemolytic/immunodeficient condition.  Pregnant women should avoid contact
with infected individuals as the virus can be transmitted to the fetus and cause anemia.  Prioritization is
determined by acuity, and therefore children do not automatically receive higher priority.  However, due to the
potential exposure of this child to a pregnant client in the ED, the triage RN should prioritize this client ahead of
the one with vaginal infection.

(Option 3)  This client is exhibiting a classic sign of the common Candida vaginitis (yeast) infection.  Classic
signs and symptoms include itching and irritation in the vulva or vagina, white cheesy vaginal discharge, and
low vaginal pH.  Although uncomfortable, this client is stable and can safely wait up to 2 hours for treatment.

Educational objective:
A client with emesis and decreased level of consciousness is a priority due to airway obstruction risk.  Sputum
changes in a client with COPD signal infection that can cause exacerbation.  Although "slapped-cheek" disease
in children is usually a harmless viral infection, pregnant women should avoid exposure.

An important part of the nursing role is to advocate for the health and safety of the client.  This client has fallen
and lost weight when living in the child's home, prompting the nurse to advocate for the client by bringing in
other members of the interdisciplinary team to assess the home situation.

When a nurse is concerned about the client's living situation, the social worker is the most appropriate team
member to consult with first.  The role of the social worker includes assessing the client's living situation and
arranging for an alternate living situation or support services as needed.

(Option 1)  Adult protective services would be notified when abuse or neglect is suspected.  In the hospital
setting, a social worker should be contacted to do a detailed assessment of the situation before adult protective
services is notified.

(Option 2)  The physical therapist should be consulted when there is concern about the client's ability to
function safely in the home environment.

(Option 3)  The physician would not be the most appropriate person to appoint when a detailed assessment of
the home living situation needs to be conducted.  However, the physician should be notified if a social worker is
assigned to assess the home living situation.

Educational objective:
Nursing advocacy for the safety of the client includes the appropriate use of interdisciplinary team members,
such as the social worker.  Advocacy is especially important in younger and elderly clients and those who are
cognitively challenged or have mental health concerns.

Although the cause of autism spectrum disorder (ASD) is unknown, numerous studies indicate that it has a
strong genetic component.  The underlying genetic source is unknown in the majority of cases; however,
researchers hypothesize that genetic factors predispose to an autism phenotype and that genetic expression is
influenced by environmental factors.

(Option 1)  A 2-year-old with a vocabulary of a few words only is a concern; however, there are a number of
factors and/or conditions that could cause language or overall developmental delay.
(Option 2)  There is no scientific evidence that the measles, mumps, and rubella vaccine or thimerosal-
containing vaccines (eg, influenza) are linked to ASD.

(Option 4)  Retrospective studies have linked parents of older age to autism; however, this association is
inconclusive.

Educational objective:
There is strong scientific evidence of a genetic component to autism spectrum disorder (ASD).  As a result,
parents who have a child with ASD are at higher risk of having another child with this disorder.  There is no
evidence that supports a link between vaccines and ASD, and studies on associations between advanced
parental age and the disorder are inconclusive.

Postoperative pain control is a priority intervention for a child of any age.  However, the nurse needs to first
perform an assessment of the child's pain to determine the appropriate pharmacological or non-pharmacological
measure to implement.  This assessment will also provide a baseline against which the effectiveness of the
chosen pain relief method can be evaluated.

A numeric pain scale can be used with most children who can count and understand the concept of numbers,
generally at around age 5.  The scale uses a straight line with divisions marked in units from 0-10; 0 is identified
as no pain, 5 as moderate pain, and 10 as worst pain.

(Option 1)  Analgesics (opiates and nonsteroidal antiinflammatory drugs), along with adjuvant analgesics, are
appropriate pain control measures in children.  However, pain should be assessed before medications are
administered.

(Option 3)  Returning later in the day allows the child to rest but does nothing to relieve current pain.

(Option 4)  This non-therapeutic response ignores the child’s expressed pain and poses a threat that could be
upsetting to the child.

Educational objective:
When a client is in pain, assessment is the first necessary nursing action.  The pain assessment helps to
determine the appropriate relief measure and serves as a baseline for evaluating the effectiveness of the chosen
pharmacological or non-pharmacological measure.

A colostomy is a surgical procedure that creates an opening (stoma) in the abdominal wall for the passage of
stool to bypass an obstructed or diseased portion of the colon.  Stool drains through the intestinal stoma into a
pouch device secured to the skin.

Clients with a descending or sigmoid colostomy drain stool that is more formed and similar to a normal bowel
movement.  Although less common, some clients choose to irrigate their colostomy in order to create a bowel
regimen that allows them to wear a smaller pouch or a dressing over the stoma.  When irrigated daily, the client
gains increased control over the passage of stool.

The procedure for bowel irrigation is as follows:

 Fill the irrigation container with 500-1000 mL of lukewarm water, flush irrigation tubing, and reclamp;
hang the container on a hook or intravenous pole (Option 2)
 Instruct the client to sit on the toilet, place the irrigation sleeve over the stoma, extend the sleeve into the
toilet, and place the irrigation container approximately 18-24 inches above the stoma (Option 3)
 Lubricate cone-tipped irrigator, insert cone and attached catheter gently into the stoma, and hold in
place
 Slowly open the roller clamp, allowing irrigation solution to flow for 5-10 minutes
 Clamp the tubing if cramping occurs, until it subsides (Option 4)
 Once the desired amount of solution is instilled, the cone is removed and feces is allowed to drain
through the sleeve into the toilet

(Option 1)  A cone-tip applicator is used to instill the irrigation solution into the stoma.  An enema set should
never be used to irrigate a colostomy.  A cone-tip applicator is specifically made to avoid damage to the
sensitive colostomy opening.

Educational objective:
Colostomy irrigation allows the client to create a bowel regimen and to apply a dressing or smaller pouch
device over the stoma.  To properly irrigate the stoma, use 500-1000 mL of lukewarm water, hang the bag 18-
24 inches above the stoma, use the cone-tipped irrigator to slowly infuse the solution, and allow stool to drain
through the sleeve into the toilet.

Chest tubes are indicated to drain air or fluid from the pleural space and reestablish negative pressure, which
allows for proper lung expansion.  When the lung has reexpanded or fluid drainage is no longer needed, the
chest tube can be discontinued.  The client should be given an analgesic 30-60 minutes prior to the procedure. 
A suture removal kit, petroleum gauze, and occlusive dressing supplies will be needed.  The client should be
instructed to take a deep breath, hold it, and bear down (Valsalva maneuver) while the tube is being removed. 
This will prevent air from being pulled back into the pleural space and possibly causing a pneumothorax.  A
post-procedure chest x-ray must be performed to ensure there is no reaccumulation of air or fluid in the
pleural space.

(Options 1 and 2)  Breathing slowly or normally during the procedure may cause the client to inhale during the
removal, pulling air back into the pleural space.

(Option 4)  Rapid shallow breaths increase the chance of inhaling during removal and pulling air into the
pleural space, causing recollapse of the lung.

Educational objective:
During chest tube removal, the client should be instructed to take a deep breath, hold it, and bear down
(Valsalva maneuver) to prevent air from reentering the pleural space and possibly causing a pneumothorax. 
The site is covered with a sterile airtight petroleum jelly gauze dressing.  A post-procedure chest x-ray is
needed.

Malignant hyperthermia (MH) is a rare but life-threatening inherited muscle abnormality that is triggered
by specific, inhaled anesthetic agents and the depolarizing muscle relaxant succinylcholine (Anectine) used to
induce general anesthesia.

In MH-susceptible clients, the triggering agent leads to excessive release of calcium from the muscles, leading
to sustained muscle contraction and rigidity (usually of the jaw and upper body [early sign]), increased oxygen
demand and metabolism, and dangerously high temperature (later sign).

As MH is an inherited condition, proper screening and a thorough preoperative nursing assessment and health
history can help minimize the client's risk (Option 1).

(Option 2)  Cervical spine problems should be assessed before the intubation.  Low back pain history is not a
priority for general anesthesia.
(Option 3)  It would be appropriate to ask about allergies (eg, drugs, latex).  However, asking about an
anaphylactic reaction to a bee sting is not the most critical question.

(Option 4)  History of prior opioid intake may be helpful, but the most important question is to ask about side
effects and allergies.

Educational objective:
Malignant hyperthermia (MH) is a rare, life-threatening inherited muscular abnormality that is triggered by
specific drugs used to induce general anesthesia.  Therefore, it is critical for the perioperative nurse to screen
for MH susceptibility by asking if any of the client's blood relatives had ever experienced an adverse reaction to
general anesthesia, including unexplained death.

Triaging clients involves decision-making about whose needs/problems are most urgent and create the greatest
risk to survival.  Two popular frameworks can assist the nurse in making these decisions and setting priorities. 
In the "First, Second, and Third" priority level framework, the priority needs progress from the first (most
immediate) to the third (least) level of risk.  They include:

1. ABCs plus V – airway, breathing, circulation, and vital signs


2. Mental status changes, acute pain, unresolved medical issues, acute elimination problems, abnormal
laboratory values, and risk
3. Longer-term issues such as health education, rest, and coping

Maslow's Hierarchy of Needs is a 5-level framework in which the priority needs progress from the bottom to
the top level of the pyramid.

Infants have a high percentage of body water (70%-80% of body weight) and become dehydrated rapidly.  This
client is at increased risk for fluid and electrolyte disturbances.  In addition, the infant is lethargic (listless),
indicating a change in level of consciousness.  This client would be assessed first (Option 2).

(Option 1)  Because this client's laceration is superficial, excessive bleeding is an unlikely risk and is a
relatively minor problem.

(Option 3)  The hematuria and elevated temperature may be associated with a urinary tract infection or
glomerulonephritis in this client and do not present an immediate threat to survival.

(Option 4)  Acute abdominal pain can be a medical emergency that could indicate appendicitis, ovarian cyst,
ectopic pregnancy, ureteral colic, or bowel obstruction.  This client would be seen second.

Educational objective:
The "First, Second, and Third" priority level framework is used in emergency and non-emergency settings to
prioritize client needs from the highest to the lowest level of risk as follows:

1. ABCs plus V – airway, breathing, circulation, and vital signs


2. Mental status changes, acute pain, unresolved medical issues, acute elimination problems, abnormal
laboratory values, and risks
3. Longer-term issues such as health education, rest, and coping
4. Cirrhosis is a progressive, degenerative disease caused by destruction and subsequent disordered
regeneration of the liver parenchyma.  Clients with cirrhosis suffer from various complications (eg,
ascites, varices, encephalopathy) that will progressively intensify without lifestyle modifications.
5. (Option 1)  Alcoholism is one of the leading causes of cirrhosis.  All clients with alcoholism should
abstain from drinking to prevent further liver damage.
6. (Option 2)  Aspirin and ibuprofen (a nonsteroidal anti-inflammatory drug [NSAID]) may cause
gastrointestinal bleeding.  Clients with esophageal varices or portal hypertension have an increased risk
of bleeding and should avoid these medications.  They should contact the health care provider regarding
any pain or fever.
7. (Option 4)  Although a low-sodium diet is important to prevent worsening hypertension and ascites, a
low-protein diet is not usually recommended.  Many clients with cirrhosis suffer from protein-calorie
malnutrition; therefore, an intake of 1.2-1.5 g/kg of protein a day is commonly prescribed.
8. Educational objective:
Clients with cirrhosis should eat a high-calorie, high-carbohydrate, low-sodium, and low-fat diet;
moderate protein intake is recommended.  They should avoid hepatotoxic substances (eg, alcohol,
acetaminophen) and medications (NSAIDs) that increase bleeding risk and reduce activities that increase
intraabdominal pressure.
9. C difficile is often associated with antibiotic therapy but can also be a nosocomial hospital-acquired
infection.  Antibiotics, especially broad-spectrum, reduce normal bacteria in the body.  This allows other
bacteria, such as C difficile, to take over and cause a superinfection.  It grows in the intestinal tract and
causes antibiotic-associated diarrhea.
10. Metronidazole (Flagyl) is an anti-infective drug commonly used to treat C difficile.  For severe C
difficile infection, oral vancomycin may be used; intravenous vancomycin is ineffective.
11. (Option 1)  Ceftriaxone (Rocephin) is a cephalosporin antibiotic; its use could cause C difficile
infection.
12. (Option 2)  Fluconazole (Diflucan) is a broad-spectrum antifungal agent; it is not indicated to treat C
difficile.
13. (Option 4)  Pantoprazole (Protonix) is a proton pump inhibitor agent; its use has been associated with
development of C difficile infection.
14. Educational objective:
Antibiotics reduce normal bacteria in the body, allowing other bacteria or fungi, such as C difficile, to
take over.  C difficile is a toxin-producing microorganism that grows in the intestinal tract and causes
antibiotic-associated diarrhea.  Metronidazole (Flagyl) and oral vancomycin are commonly used to treat
this condition.
15. Diverticular disease of the colon is a condition in which there are sac-like protrusions in the large
intestine (diverticula).  Diverticulosis is characterized by the presence of these protrusions; the client is
asymptomatic and may not even be aware of the condition.  Diverticulitis occurs when diverticula
become infected and inflamed.  Complications of diverticulitis include abscess, fistula formation,
intestinal obstruction, peritonitis, and sepsis.  Diverticular bleeding occurs when a blood vessel next to
one of these pouches bursts; this may cause blood in the stool.
16. The etiology of diverticular disease has been linked to chronic constipation, a major cause of excess
intracolonic pressure.  Preventing constipation may help reduce the risk of diverticula forming and
becoming inflamed.
17. Measures to prevent constipation include a diet high in fiber (whole grains, fruits, vegetables), daily
intake of at least 8 glasses of water or other fluids, and exercise.  A fiber supplement such as psyllium
or bran may be advised.  In the past, clients have been taught to avoid consuming seeds, nuts, and
popcorn; however, current evidence does not indicate that avoidance of these foods will prevent an
episode of diverticulitis.
18. (Option 3)  A low-residue diet, which avoids all high-fiber foods, may be used in treating acute
diverticulitis.  However, after symptoms have resolved, a high-fiber diet is resumed to prevent future
episodes.
19. (Option 5)  Increased consumption of red meat and other high-fat foods can increase the risk of
diverticulitis.
20. Educational objective:
Clients with diverticulosis should take measures to prevent constipation (eg, high-fiber diet, increased
fluid intake, regular exercise), which may help prevent recurring episodes of acute diverticulitis.
21. Tolterodine (Detrol LA), oxybutynin (Ditropan), and solifenacin (Vesicare) are
antimuscarinic/anticholinergic medications used for overactive bladder and urge urinary incontinence. 
They decrease urinary urgency and frequency.  The most common side effects are anticholinergic (eg,
dry mouth, constipation, cognitive dysfunction).  The client's report of not urinating the entire day while
at work may indicate that the dosage is too high and is causing urinary retention.  Urinary retention
can lead to bladder infections and distension.  This information should be reported to the health care
provider (HCP).
22. (Option 2)  Artificial saliva products and sugar-free hard candy and gum are acceptable ways to manage
dry mouth caused by anticholinergic medications.
23. (Option 3)  Occasional dizziness is a side effect of tolterodine.  The client should rise and change
positions slowly.  However, if this client is receiving too high a dose, reduction of the dose may alleviate
the dizziness.  Severe dizziness should be reported to the HCP.
24. (Option 4)  Constipation can be managed with increased fiber in the diet, fluids, stool softeners, or
laxatives.

Educational objective:
Anticholinergic medications (eg, tolterodine, oxybutynin, solifenacin) are commonly used for overactive
bladder.  The client should experience a reduction in the number of times needed to urinate, but the number
should not decrease below typical urination frequency.  The nurse should also teach the client how to
manage the common side effects of dry mouth, constipation, and mild dizziness.

Thrombin inhibitors such as dabigatran (Pradaxa) reduce the risk of clot formation and stroke in clients with
chronic atrial fibrillation.  The nurse should educate the client about implementing bleeding precautions (eg,
using a soft-bristle toothbrush, shaving with an electric razor).  Dabigatran capsules should be kept in their
original container or blister pack until time of use to prevent moisture contamination (Option 3).

(Option 1)  Red urine or blood in the stool may indicate internal bleeding caused by thrombin inhibitors.  The
client should report these symptoms to the health care provider.

(Option 2)  Thrombin inhibitors should only be stopped under the direction of the health care provider.  The
nurse should educate the client that stopping dabigatran will increase the risk for stroke.  Taking the medication
with food will not affect how much is absorbed, and food or a full glass of water may prevent gastrointestinal
side effects (eg, nausea, indigestion).

(Option 4)  Thrombin inhibitor capsules should not be crushed or opened as crushing pills increases absorption
and risk of bleeding.

Educational objective:
Thrombin inhibitors such as dabigatran reduce the risk for clots and stroke in clients with chronic atrial
fibrillation.  The nurse should teach the client to use bleeding precautions and monitor for symptoms of
bleeding, swallow capsules whole with a full glass of water, and keep capsules in their original container until
time of use.

Diabetes insipidus (DI) results in low levels of antidiuretic hormone (ADH), which is produced by the
hypothalamus and stored in the pituitary gland.  The function of ADH is to concentrate urine by signaling the
kidneys to retain water in the setting of thirst.  When ADH levels are insufficient, the kidneys excrete large
quantities of very dilute urine (polyuria).  This causes hypernatremia (elevated serum sodium due to deficit of
free water) and increased serum osmolality, which lead to excessive thirst (polydipsia).

Desmopressin acetate (DDAVP) is a synthetic form of ADH, which can be administered intravenously, orally,
or via nasal spray.  Effectiveness of therapy with desmopressin would be manifested by decreased urinary
output and increased urine specific gravity as the urine becomes less dilute (Option 3).
(Option 1)  A client's thirst, not appetite, is affected by DI.

(Option 2)  DI is related to water balance, but not to diabetes mellitus, a disorder of glucose metabolism.

(Option 4)  If desmopressin therapy is effective, the client's urine specific gravity will be higher due to the urine
output decreasing and becoming less dilute.

Educational objective:
Use of desmopressin acetate (DDAVP) in clients with diabetes insipidus will lower urinary output and cause the
urine specific gravity to increase.

Thiazolidinediones (rosiglitazone [Avandia] and pioglitazone [Actos]) are used to treat type 2 diabetes
mellitus.  These agents improve insulin sensitivity but do not release excess insulin, leading to a low risk for
hypoglycemia (similar to metformin).  These drugs can worsen heart failure by causing fluid retention and
increase the risk of bladder cancer.  Heart failure or volume overload is a contraindication to thiazolidinedione
use.  These medications also increase the risk of cardiovascular events such as myocardial infarction.

(Option 2)  The target blood pressure for a client with diabetes is <140/90 mm Hg.

(Option 3)  The goal HbA1c for diabetic clients is <7%.

(Option 4)  Diabetic retinopathy, a condition treated with retinal photocoagulation, is unrelated to
thiazolidinedione use.  If the client has a history of bladder cancer, then it should be reported.

Educational objective:
Thiazolidinediones (rosiglitazone [Avandia] and pioglitazone [Actos]) increase the risk of cardiovascular events
(eg, mycoardial infarction, heart failure) and bladder cancer.  Thiazolidinedione use increase insulin sensitivity
but carries a low risk for hypoglycemia (similar to metformin).

Shock is a life-threatening syndrome characterized by decreased perfusion and impaired cellular metabolism.  A
lack of perfusion at both the tissue and cellular level (anaerobic metabolism) occurs due to decreased cardiac
output, ineffective blood flow, and inability to meet the body's demand for increased oxygen.  Sustained
hypoperfusion activates compensatory mechanisms (eg, neural, hormonal, biochemical) to maintain
homeostasis and reverse the consequences of anaerobic metabolism.  Shock will progress through 4 stages
(initial, compensatory, progressive, irreversible).  Early identification and intervention help to prevent stage
progression.

Adequacy of tissue perfusion in a client with shock syndrome and possible organ dysfunction is assessed by the
level of consciousness, urine output, capillary refill, peripheral sensation, skin color, extremity temperature,
and peripheral pulses.

Capillary refill indicates adequacy of blood flow to the peripheral tissues.  It is measured by the time taken for
color (pink) to return to an external capillary bed (nail bed) after pressure is applied to cause blanching.  In an
adult, color should return in less than 3 seconds.

Normal skin color and temperature are indicators of the adequacy of peripheral blood flow; these are usually
within normal limits during the initial and compensatory stages of shock.

(Option 1)  Apical pulse is a central pulse and does not indicate adequacy of peripheral tissue perfusion.

(Option 3)  Lung sounds indicate the adequacy of ventilation and gas exchange, not peripheral tissue perfusion.
(Option 4)  Pupillary response is an indicator of cerebral function, not peripheral tissue perfusion.

Educational objective:
The adequacy of blood flow to peripheral tissues is determined by measuring capillary refill and assessing skin
color and temperature; these are usually within normal limits during the initial and compensatory stages of
shock.

IV infusion pumps display an occlusion alarm when IV solution cannot be infused due to pressure in the line. 
Common causes of occlusion include clamped or kinked IV tubing, clotting in the IV catheter, and kinking in
the IV catheter with extremity movement (eg, elbow, wrist).  The nurse should assess the tubing and IV site and
flush the IV catheter to check patency.

In the absence of identifiable occlusion, an alarming IV pump should be exchanged for a different one (Option
2).  Malfunctioning equipment may harm the client and should be removed from the care area.  The
malfunctioning equipment is labeled as out of service and is sent for maintenance.

(Option 1)  An IV pump alarm does not indicate that a medication is no longer needed.  The nurse should
replace a malfunctioning pump and restart the medication.

(Option 3)  An IV catheter that has no symptoms of occlusion (ie, resistance to flushing) or infiltration (eg,
swelling, coolness, pain) does not need to be replaced.

(Option 4)  IV pump infusion is more accurate than gravity drip.  IV pumps are required when administering
high-risk IV medications (eg, heparin, insulin, potassium).  IV potassium should never be administered by
gravity as it may cause lethal arrhythmias if administered too quickly.

Educational objective:
If an IV infusion pump displays an alarm without an identifiable problem, the nurse should replace the pump. 
Malfunctioning equipment may harm the client and should be removed from the care area.  The equipment is
labeled as out of service and sent for maintenance.

Anorexia nervosa is an eating disorder common among adolescents and young adults.  Clinical manifestations
of anorexia nervosa include:

1. Fear of weight gain – clients resort to self-induced vomiting, extensive dieting, and intense exercise
resulting in excessive weight loss (<85% expected weight).  Clients who self-induce vomiting may
experience enlargement of the salivary glands and erosion of tooth enamel.
2. Fluid and electrolyte imbalance – excessive vomiting can cause hypokalemia and metabolic alkalosis
3. Amenorrhea – clients are often amenorrheic due to decreased body fat (low estrogen)
4. Decreased metabolic rate – severe weight loss results in hypotension, bradycardia, decreased body
temperature, and cold intolerance
5. Lanugo (fine terminal hair) can be seen in extreme cases

Manifestations of anorexia nervosa will resolve after the weight loss is corrected, although the recovery process
can take several months.

(Option 3)  Anorexia nervosa manifests as cold intolerance.

(Option 5)  Anorexia nervosa manifests as lengthy and vigorous exercise.


Educational objective:
The clinical manifestations of anorexia nervosa include extreme weight loss, amenorrhea, bradycardia, cold
intolerance, dry skin, and lanugo.  Life-threatening complications, such as cardiac arrhythmias associated with
hypokalemia, may develop.

A client who is postoperative total knee replacement is at risk for infection.  No postoperative client should be
assigned to a room with a client who has an actual infection or the potential for infection.

This client should be assigned to room 4 as the client with the cast has the lowest potential risk for infection
(Option 4).

(Option 1)  This client has erythema at the pin sites; this can be a sign of infection, a complication of skeletal
traction.

(Option 2)  This client has cellulitis, a bacterial infection of the skin, and osteomyelitis, an infection of the
bone.

(Option 3)  This client has a fasciotomy wound, which is usually kept open for several days to relieve the
pressure in the myofascial compartment.  This client is a potential source of infection and is susceptible to
infection as well.

Educational objective:
A client who is postoperative total knee replacement is at increased risk for infection.  This client should not be
assigned to a room with a client who has an actual (eg, cellulitis, osteomyelitis) or potential (eg, skeletal
traction, fasciotomy) infection.

The nurse should check on the assigned clients in the following order:

1. Client with the gastrointestinal bleed receiving packed red blood cells (PRBCs) – the nurse should:
o Check the infusion device; flow rate; and IV site, tubing, and filter
o Collect baseline physical assessment data against which to compare subsequent assessments
o Assess for complications associated with the administration of PRBCs, which include fluid
overload and an acute transfusion reaction; these can occur at any time during the transfusion
(Option 1)
2. Client with chronic kidney disease scheduled for dialysis in 30 minutes - the nurse should perform a
baseline assessment before dialysis is initiated.  The nurse should then prepare the client by making sure
the client eats breakfast, administering prescribed morning medications that are not dialyzed out, and
holding those that are dialyzed out.  Elevated creatinine level (eg, normal 0.6-1.3 mg/dL [53-115
µmol/L]) is an expected finding.  (Option 4)
3. Client with ulcerative colitis (UC) with elevated temperature and abdominal pain – UC is an
inflammatory bowel disease; fever and lower-quadrant abdominal cramping are expected findings. 
After assessing the client, the nurse will administer an analgesic and an antipyretic as prescribed. 
(Option 2)
4. Client with history of atrial fibrillation, prescribed warfarin (Coumadin) – the client is on telemetry; in
most facilities, if dysrhythmias occur, the monitor technician/nurse will notify the primary care nurse
immediately.  The goal INR is 2.0 to 3.0 for atrial fibrillation.  An INR of 3.2 is expected when
adjusting the warfarin dose.  (Option 3)

Educational objective:
To prioritize client care, the nurse first identifies the type of problem, its associated complications, and the
desired outcomes.  The nurse then decides which client has the most urgent problems and needs and assesses
that client first.

Peripheral artery disease (PAD [previously called peripheral vascular disease]) refers to arteries that have
thickened, have lost elasticity due to calcification of the artery walls, and are narrowed by atherosclerotic
plaques (made up of fat and fibrin).  Pain due to decreased blood flow is the most common symptom of PAD. 
Cramping pain in the muscles of the legs during exercise, known as intermittent claudication, is usually
relieved with rest.  However, with critical arterial narrowing, pain can be present at rest and is typically
described as "burning pain" that is worsened by elevating the legs and improved when the legs are dependent. 
Skin becomes cool, dry, shiny, and hairless (due to lack of oxygen).  Ulcers and gangrene occur usually at the
most distal part of the body, where circulation is poorest.  Clients should be advised that a progressive
walking program will aid the development of collateral circulation.

(Options 1, 2, and 3)  Chronic venous insufficiency refers to inadequate venous blood return to the heart. 
Too much venous blood remains in the lower legs, and venous pressure increases.  This increased venous
pressure inhibits arterial blood flow to the area, resulting in inadequate supply of oxygen and nutrients to area
cells and the development of stasis ulcers, which are typically found around the medial side of the ankle.  By
the end of the day or after prolonged standing, the legs become edematous with dull pain due to venous
engorgement.  The skin of the lower leg becomes thick with a brown pigmentation.

Educational objective:
The pain of peripheral artery disease is arterial in nature and results from decreased blood flow to the legs.  It is
made worse with leg elevation.  Arterial ulcers are formed at the most distal end of the body.  Venous ulcers
form over the medial malleolus, and compression bandaging is needed to reduce the pressure.

Marfan syndrome is an autosomal dominant disorder affecting the connective tissues of the body. 
Abnormalities are mainly seen in the cardiovascular, musculoskeletal, and ocular systems.  Clients with Marfan
syndrome are very tall and thin, with disproportionately long arms, legs, and fingers.

Cardiovascular manifestations of Marfan syndrome include abnormalities of the aorta and cardiac valves,
including aneurysms, tears (dissection), and leaky heart valves that may require replacement or repair. 
Therefore, competitive or contact sports are discouraged due to the risk of cardiac injury and sudden death
(Option 1).

(Option 2)  The client may also experience crowding of the teeth from a very high-arched palate.  Preventive
antibiotics prior to dental work may be needed to provide prophylaxis against infective endocarditis, especially
in clients with an artificial valve replacement.  However, this is not a priority.

(Option 3)  These clients have an increased risk for scoliosis, especially during the adolescent years of
increased growth; therefore, the child should be monitored regularly for curvature of the spine.  This is not a
priority.

(Option 4)  Ocular problems (eg, lens dislocation [ectopia lentis], retinal detachment, cataracts, glaucoma) can
be common for the child with Marfan syndrome.  Annual eye examinations with an ophthalmologist are
important to monitor for developing issues.

Educational objective:
Aortic root disease (ie, aneurysm, dissection) is the major cause of morbidity and mortality in Marfan
syndrome.  Therefore, competitive or contact sports are discouraged due to the risk of cardiac injury and sudden
death.
Syndrome of inappropriate antidiuretic hormone secretion (SIADH) is associated with increased water
reabsorption and excessive intra- and extracellular fluid, which result in hypervolemia from fluid retention and
dilutional hyponatremia.  In the setting of SIADH, the nurse should question a prescription for a hypotonic
solution (eg, 0.45% NaCl; or dextrose water) as it would worsen the fluid and electrolyte imbalance.  A
prescription for fluid restriction and a hypertonic IV solution (eg, 3% NaCl) administered in small quantities
would be appropriate to shift fluid back into the vascular compartment and correct hyponatremia.

(Option 2)  Isotonic fluids (eg, normal saline) are appropriate for clients with volume deficit such as those with
gastrointestinal bleeding.

(Option 3)  Septic shock involves an inflammatory response to pathogens that leads to massive vasodilation and
increased capillary permeability, resulting in intravascular hypovolemia and severe hypotension.  An isotonic
solution (eg, 0.9% NaCl) bolus is prescribed to expand intravascular volume and increase blood pressure.

(Option 4)  A burn injury causes tissue damage and increased capillary permeability; this leads to fluid and
electrolyte losses related to evaporation and intravascular fluid shifts into the interstitial tissue, which result in
hypovolemia, hemoconcentration (eg, hematocrit >53% [0.53]), and hypotension.  An isotonic solution (eg,
lactated Ringer's) is prescribed to replace fluid and electrolyte losses.

Educational objective:
Syndrome of inappropriate antidiuretic hormone secretion is associated with hypervolemia and dilutional
hyponatremia.  Fluid restriction and hypertonic IV solutions (eg, 3% saline) are prescribed to correct
hyponatremia.

To prevent air embolism when discontinuing a central venous catheter, the nurse should perform the following
interventions:

 Instruct the client to lie in a supine position.  This will increase the central venous pressure and decrease
the possibility of air getting into the vessel (Option 3).
 Instruct the client to bear down or exhale.  The client should never inhale during removal of the line;
inhalation will suck more air into the blood vessel via negative suction pressure (Options 2 and 5).
 Apply an air-occlusive dressing (usually gauze with a Tegaderm dressing) to help prevent a delayed air
embolism.  If possible, the nurse should attempt to cover the site with the occlusive dressing while
pulling out the line (Option 1).
 Pull the line cautiously and never pull harder if there is resistance.  Doing so could cause the catheter to
break or become dislodged in the client's vessel (Option 4).

Educational objective:
To prevent air embolism when discontinuing a central venous catheter, it is important for the nurse to pull the
line cautiously, have the client in a supine position, have the client bear down or exhale, and apply an air-
occlusive dressing.

Carbon monoxide (CO) is a colorless, odorless gas produced by burning fuel (eg, oil, kerosene, coal, wood) in
a poorly ventilated setting.  CO toxicity (poisoning) is most often associated with smoke inhalation from
structure fires, but is also generated by furnaces/hot water heaters fueled by natural gas or oil, coal or wood
stoves, fireplaces, and engine exhaust.

Clients with CO toxicity often have nonspecific symptoms, and the diagnosis can be missed.  It is important to
assess for possible CO exposure to initiate appropriate emergency care and prevent hypoxic neurologic
impairment.  To help identify elevated CO levels in the home, the nurse can ask about the following:
 Similar symptoms in other family members, or an illness in an indoor pet that developed at the same
time
 Fuel-burning heating/cooking appliances; risk of CO toxicity increases in the fall and winter due to
increased used of heat sources in an enclosed space (Option 3)

(Options 1, 2, and 4)  It is important to reconcile the client's vaccinations, obtain a nutritional history, and
explore the nature of the client's symptoms, but it is essential to rule out the possibility of CO toxicity given the
circumstances of this client's illness.

Educational objective:
Carbon monoxide (CO) toxicity can occur when fuel-burning (eg, wood, coal) stoves or appliances are used in
poorly ventilated settings.  Clients with CO toxicity may have vague symptoms (eg, headache, dizziness,
nausea), so it is important to assess for possible CO exposure to prevent delay of appropriate emergency care.

Polycythemia vera (PV) is a chronic disorder of the bone marrow in which too many red blood cells, white
cells, and platelets are produced.  Clients with PV are at risk of developing blood clots due to increased blood
volume and viscosity.  Clients are instructed to elevate the legs and feet when sitting, wear support stockings,
and report signs of thrombosis (eg, swelling and tenderness in the legs).  Adequate fluid intake during exercise
and hot weather is important to reduce fluid loss and decrease viscosity (Options 1, 3, and 5).

(Option 2)  Increasing intake of iron-containing foods and supplements can further increase hemoglobin
production and is not recommended.  Clients with PV need periodic phlebotomy to remove excess blood.

(Option 4)  Itching is a common and frustrating symptom of PV.  Reducing water temperature, using starch
baths, and patting the skin dry rather than rubbing vigorously are beneficial.

Educational objective:
Clients with polycythemia vera are at risk of developing thrombosis and should be taught preventive measures
(eg, elevating the legs when sitting) and symptoms to report.  They should take measures to prevent
dehydration, and avoid iron-rich foods and hot showers/baths.

The nurse should ascertain potential risks to the client and her baby due to teenage pregnancy.  Lack of
family/social support or fear of social discrimination may prevent the client from obtaining prenatal care
(Option 2).  Poverty, dangerous living conditions, and exposure to teratogens (eg, tobacco, alcohol, illicit
drugs) may place the client at risk for complications.  Adolescents are at risk for poor nutritional status and
poor pregnancy weight gain, which can have deleterious effects on the baby (eg, small for gestational age, low
birth weight) (Option 4).  They are also less likely to take prenatal vitamins with folic acid.  Adolescents who
are pregnant should be evaluated for sexual abuse; girls age 11-14 do not usually seek sexual relationships, and
the pregnancy may be evidence of abuse (Option 5).

(Option 1)  Adoption planning is not necessary at this first assessment, although the topic may be addressed
later in collaboration with a social worker.

(Option 3)  The younger adolescent (age <16) is emotionally and developmentally immature and may be
resistant to planning for the future.  Education planning may be approached at subsequent encounters, but it is
not a priority for the client and the baby's health at this time.

Educational objective:
Adolescents are at increased risk for obstetrical complications that can contribute to neonatal morbidity and
mortality.  Factors such as lack of family support, sexual abuse, and poor nutritional status can negatively
impact the pregnancy.
CVP is a measurement of right ventricular preload (volume within the ventricle at the end of diastole) and
reflects fluid volume problems.  The normal CVP is 2-8 mm Hg.  An elevated CVP can indicate right
ventricular failure or fluid volume overload.

Clinical signs of fluid volume overload include the following:

 Peripheral edema
 Increased urine output that is dilute
 Acute, rapid weight gain
 Jugular venous distension
 S3 heart sound in adults
 Tachypnea, dyspnea, crackles in lungs
 Bounding peripheral pulses

(Options 2 and 3)  Dry mucous membranes and hypotension are signs of deficient fluid volume or dehydration.

Educational objective:
Elevated CVP can indicate fluid volume overload.  The nurse should recognize clinical signs such as crackles in
lungs, jugular venous distension, and peripheral edema as evidence of fluid volume overload.

A flaccid extremity and change in verbal ability are symptoms of a stroke, which is considered an emergency. 
Clients with stroke symptoms are immediately triaged using a special team and set of tools to determine the
correct course of action with the goal of preventing further brain damage.  In any emergency, the first priority
nursing action is to maintain a patent airway (Option 3).  Depending on the mechanism of injury, the symptoms
may include changes in airway clearance, which is a priority.

The nurse, or another member of the emergency department or stroke alert team, will prepare the client for an
immediate head CT scan to rule out a hemorrhagic stroke and determine the location and extent of the injury
(Option 4).  This person will also ensure that the client has 2 large-bore IV lines for rapid infusion of fluids or
medications as needed (Option 2).

(Option 1)  It is vital to determine the onset of symptoms as thrombolytic medications are used in a short time
frame (typically within 4.5 hours of onset).  Thrombolytic medications are used only in ischemic strokes, so the
head CT must be completed to confirm the type of stroke (ischemic versus hemorrhagic).  With all of these
interventions, the priority nursing actions remain the same: ABC - airway, breathing, and circulation.

Educational objective:
In any emergency, the primary nursing interventions are the ABCs.  A patent airway should be maintained
while other care is provided and throughout the emergency treatment process.

Glargine (Lantus) is a basal insulin with no peak, also known as a "background insulin."  Lispro (Humalog) is a
rapid-acting insulin used as a bolus insulin for meals.  It is a "see food" insulin and should not be administered
until the food is in sight.  Insulin is used to control diabetes mellitus and if the glucose level is "within defined
limits," this means that the disease is adequately controlled.

(Options 2, 3, and 4)  Holding glargine will increase the blood sugar level over 24 hours.  The lispro insulin is
given to cover the meal being consumed.  After the meal, the blood sugar level will increase from the normal
level.  If the premeal blood sugar level is >150 mg/dL (8.3 mmol/L), additional sliding scale insulin is needed,
in addition to the fixed premeal insulin.  Holding it will cause blood glucose to rise uncontrollably.
Educational objective:
Continue the same insulin dose when the client's glucose level is within defined limits and the client is eating. 
It demonstrates that the medication dosage is controlling the disease.

Opioid analgesics (eg, hydromorphone, morphine) are effective for controlling moderate to severe pain.  Major
side effects include sedation, respiratory depression, hypotension, and constipation.  The client is at risk for
falls from sedation or hypotension and should not get out of bed unassisted (Option 5).  Slowed bowel motility
persists throughout opioid use, and measures to prevent constipation (eg, administration of daily stool
softeners) should be implemented (Option 2).

(Option 1)  IV hydromorphone should be administered slowly over 2-3 minutes.  Rapid IV administration of
opioid analgesics can cause severe hypotension and respiratory or cardiac arrest.

(Option 3)  Postoperative clients may experience pain with breathing exercises (eg, turning, coughing, deep
breathing, incentive spirometry).  Uncontrolled postoperative pain may cause clients to avoid deep breathing
and lead to atelectasis and pneumonia.  The nurse should administer opioids to achieve adequate pain control as
needed to encourage participation in postoperative exercises and prevent complications.

(Option 4)  The nurse should reassess pain and sedation level during the opioid's peak effect, which is 15-30
minutes after administration of IV hydromorphone.

Educational objective:
Opioid analgesics are effective for managing postoperative pain, which encourages participation in deep
breathing exercises.  Side effects of opioid analgesics include sedation, respiratory depression, hypotension, and
constipation.  The nurse should administer IV hydromorphone slowly over 2-3 minutes, monitor sedation level,
instruct the client not to get out of bed unassisted, and administer PRN stool softeners.

Parkinson disease (PD) is a progressive neurological disorder characterized by bradykinesia (loss of autonomic
movements), rigidity, and tremors.  Clients with PD have an imbalance between dopamine and acetylcholine in
which dopamine is not produced in high enough quantities to inhibit acetylcholine.  Anticholinergic
medications (eg, benztropine, trihexyphenidyl) are commonly used to treat tremor in these clients.  However,
in clients with benign prostatic hyperplasia or glaucoma, caution must be taken as anticholinergic drugs can
precipitate urinary retention and an acute glaucoma episode.  As a result, such medications are
contraindicated in these clients.

(Option 2)  Decreased ability to exercise is common in clients with PD due to tremors and bradykinesia, and
they require physical and occupational therapy consultations.  However, acute glaucoma can be sight
threatening and is the priority.

(Option 3)  Esomeprazole is safe to take with benztropine and will not cause an adverse reaction.

(Option 4)  Constipation is a common side effect of benztropine.  Due to the characteristic decreased mobility,
PD can also cause constipation.  The client should be instructed to increase dietary fiber intake and drink plenty
of water.  However, this is not the most concerning issue.

Educational objective:
Anticholinergic medications (eg, benztropine, trihexyphenidyl) are used to treat Parkinson disease tremor. 
However, they can precipitate acute glaucoma and urinary retention and are therefore contraindicated in
susceptible clients (eg, those with glaucoma or benign prostatic hyperplasia).

A small fire can quickly become very dangerous.  During an emergency situation, such as a fire, anxiety can
narrow a person's focus, causing hesitation or difficulty in responding to the situation, especially when
operation of unfamiliar equipment (eg, fire extinguisher) is involved.  The mnemonic PASS is often used to
help people remember the steps used in operating a fire extinguisher:

P – Pull the pin on the handle to release the extinguisher's locking mechanism
A – Aim the spray at the base of the fire
S – Squeeze the handle to release the contents/extinguishing agent
S – Sweep the spray from side to side until the fire is extinguished

(Option 3)  The extinguisher does not need to be shaken before use, and doing so would delay extinguishing the
fire.

Educational objective:
PASS is a mnemonic to help people remember the steps used in operating a fire extinguisher: P - Pull the pin; A
- Aim the spray at the base of the fire; S - Squeeze the handle; and S - Sweep the spray.

The presence of severe diaper rash should be reported to the HCP.  This could potentially delay the procedure if
the rash is in the groin area where access is planned for arterial cannulation.  Candida, yeast, or bacteria may be
present on the rash and could be introduced into the bloodstream with the arterial stick.

(Option 1)  A loud heart murmur can be an expected finding in a child with pulmonic stenosis.

(Option 2)  Children are allowed nothing by mouth for 4-6 hours or longer before the procedure.  Younger
children and infants may have a shorter period of NPO status and should be feed right up to the time
recommended by the HCP.

(Option 3)  Infants and children with polycythemia may need IV fluids to prevent dehydration and
hypoglycemia.  Polycythemia will not cause a delay in the procedure.

Educational objective:
The nurse should report the presence of severe diaper rash to the HCP in an infant who has an interventional
catheterization procedure planned.  If the rash is near the groin area, the procedure may be delayed due to
possible contamination at the insertion site.

Clients with one-sided weakness or injury, increased joint pressure, or poor balance can use a cane to provide
support and stability when walking.  Cane length should equal the distance from the client's greater trochanter
to the floor as incorrect cane length can cause back injury.  A cane measured from the waist would be too long
to provide optimal support (Option 4).

Teaching points to assist a client in appropriate use of a cane include:

1. Hold the cane on the stronger side to provide maximum support and body alignment, keeping the
elbow slightly flexed (20-30 degrees) (Option 1).
2. Place the cane 6"-10" (15-25 cm) in front of and to the side of the foot to keep the body weight on both
legs to provide balance (Option 3).
3. For maximum stability, move the weaker leg forward to the level of the cane, so that body weight is
divided between the cane and the stronger leg (Option 2).  If minimal support is needed, the cane and
weaker leg are advanced forward at the same time.
4. Move the stronger leg forward past the cane and the weaker leg, so the weight is divided between the
cane and the weaker leg.
5. Always keep at least 2 points of support on the floor at all times.
Educational objective:
Clients should hold the cane on the stronger side to provide maximum stability.  Cane length should equal the
distance from the greater trochanter to the floor.

Incentive spirometry is recommended in postoperative clients to prevent atelectasis associated with incisional
pain, especially in upper abdominal incisions (close to the diaphragm).  Adequate pain medication should be
administered before using the incentive spirometry.  Guidelines recommend 5-10 breaths per session every
hour while awake.  Volume-oriented or flow-oriented sustained maximal inspiration (SMI) devices can be
used.

The client instructions for using a volume-oriented SMI device include:

1. Assume a sitting or high Fowler position, which optimizes lung expansion, and exhale normally
2. While holding the device at an even level, seal the lips tightly on the mouthpiece to prevent leakage of
air around it
3. Inhale deeply through the mouth until the piston is elevated to the predetermined level of tidal volume. 
The piston is visible on the device and helps provide motivation.
4. Hold the breath for at least 2-3 seconds (up to 6 seconds) as this maintains maximal inhalation
5. Exhale slowly to prevent hyperventilation
6. Breathe normally for several breaths before repeating the process
7. Cough at the end of the session to help with secretion expectoration

Educational objective:
Incentive spirometry is recommended to prevent atelectasis in postoperative clients.  Clients with incisional
pain should receive adequate pain medication prior to the inhalations.  The client is instructed to use the device
while sitting upright, seal the lips tightly around the mouthpiece, inhale deeply, sustain the maximal inspiration
for at least 2-3 seconds, exhale slowly before repeating the procedure, and cough at the end of the session.

Sexual harassment, including soliciting sexual favors in exchange for favorable job benefits, is prohibited. 
Other behaviors that could be defined as sexual harassment include asking someone for a date after the other
person expressed disinterest or making remarks about a person's gender or body.

The receiving nurse should first immediately and clearly indicate that the attention is unwanted and the
offending HCP should stop.  The offending HCP may have erroneously perceived a mutual attraction.  If that is
not effective, additional action should be taken.  The American Nurses Association cites 4 tactics to fight
workplace sexual harassment: confront, report, document, and support.

(Option 1)  The incident should be reported, especially if the offending HCP does not stop.  If the harasser is
the immediate supervisor, the receiving nurse should go up the chain of command.  However, the nurse should
first simply tell the offending HCP to stop and see if that resolves the issue.

(Option 3)  The nurse should respond with assertiveness, not avoidance.  Ignoring the situation may imply that
the nurse does not mind the HCP's attention.

(Option 4)  The receiving nurse should document what occurred and how the nurse responded.  The presence of
witnesses should be documented.  Documentation should be stored somewhere other than the workplace. 
However, the nurse should initially communicate assertively that the actions are to stop before documenting
them.

Educational objective:
A nurse who receives unwanted sexual advances in the workplace should first immediately and clearly indicate
that the advances are unwanted and that the offending person should stop.
When performing the suctioning procedure, the nurse follows institution policy and observes principles of
infection control and client safety.  Strict aseptic technique is maintained because suctioning can introduce
bacteria into the lower airway and lungs.

1. Place the client in semi-Fowler's position, if not contraindicated, to promote lung expansion and
oxygenation.
2. Preoxygenate with 100% oxygen (hyper-oxygenate) to prevent hypoxemia and microatelectasis. 
Alternately, if the client is breathing room air independently, ask the client to take 3-4 deep breaths.
3. Insert the catheter gently the length of the airway without applying suction to prevent mucosal tissue
damage.  The distance can be premeasured (0.4-0.8 in [1-2 cm] past the distal end of the tube).
4. Withdraw the catheter slightly (0.4-0.8 in [1-2 cm]) if resistance is felt at the carina (bifurcation of the
left and right mainstem) to prevent mucosal tissue damage.
5. Apply intermittent suction while rotating the suction catheter during withdrawal to prevent mucosal
tissue damage.  Limit suction time to 5-10 seconds with each suction pass to prevent mucosal tissue
damage and limit hypoxia.

Educational objective:
Suctioning removes secretions from the airway.  The nurse should minimize risks associated with suctioning by
using correct aspiration technique and client positioning.  Semi-Fowler's position promotes lung expansion. 
Preoxygenation and limit of suction time to 5-10 seconds reduces hypoxia and trauma.

Creatinine level of 2.5 mg/dL (221 µmol/L) is the most important abnormal value (normal 0.6-1.3 mg/dL [53-
115 µmol/L]) for the nurse to report to the health care provider.  An elevated creatinine level increases the risk
for intra- and postoperative complications.  Nothing-by-mouth (NPO) status preoperatively, dehydration (ie,
fluid shift from peritonitis), intraoperative fluid losses, antibiotic therapy, and advanced age affect renal
function and increase the risk for postoperative exacerbation of kidney injury in this client.

(Option 2)  Potassium level 3.5 mEq/L (3.5 mmol/L) is within normal limits (3.5-5.0 mEq/L [3.5-5.0 mmol/L]).

(Option 3)  Sodium level 134 mEq/L (134 mmol/L) is decreased (normal 135-145 mEq/L [135-145 mmol/L])
but is most likely related to NPO status and fluid shift from peritonitis.

(Option 4)  Elevated White blood cell count (normal 4,000-11,000/mm3 [4.0-11.0 ×109/L]) is an expected
finding related to ruptured diverticulum and peritonitis.

Educational objective:
NPO status preoperatively, dehydration, intraoperative fluid losses, antibiotic therapy, and advanced age can
negatively affect renal function.  An elevated serum creatinine level preoperatively increases the risk for
postoperative kidney injury.

Immediately following a thoracotomy, chest tube drainage (50-500 mL for the first 24 hours) is expected to be
sanguineous (bright red) for several hours and then change to serosanguineous (pink) followed by serous
(yellow) over a period of a few days.  A rush of dark bloody drainage from the chest tube when the client was
turned following a period of minimal drainage is most likely related to retained blood due to a partial blockage
in the tube.  Bright red drainage indicates active bleeding and would be of immediate concern.

(Option 2)  The chest tube should not be clamped because it is placed to drain the fluid leaking after surgery.

(Option 3)  The nurse would notify the health care provider immediately of bright red drainage or continued
increased drainage (>100 mL/hr) and of changes in the client's vital signs and cardiovascular status that could
indicate bleeding (eg, hypotension, tachycardia, tachypnea, decreased capillary refill, cool and pale skin).  This
is not the appropriate action.
(Option 4)  It would be appropriate to request repeat serum hematocrit and hemoglobin levels if active bleeding
is suspected, but the postoperative levels are stable at this time.  This is not the appropriate action.

Educational objective:
A client will usually have a chest tube in place for several days following a thoracotomy to drain blood from the
pleural space.  A rush of dark bloody drainage from the tube when the client coughs, turns, or is repositioned
following a period of minimal drainage is most likely related to retained blood due to a partial blockage in the
tube.  Bright red chest drainage indicates active bleeding and would be of immediate concern.

Toilet training is a major developmental achievement for the toddler.  The degree of readiness progresses
relative to development of neuromuscular maturity with voluntary control of the anal and urethral sphincters
occurring at age 18-24 months.  Bowel training is less complex than bladder training; bladder training requires
more self-awareness and self-discipline from the child and is usually achieved at age 2½-3½ years.

In addition to physiological factors, developmental milestones rather than the child's chronological age signal a
child's readiness for toilet training.  These include the ability to:

 Ambulate to and sit on the toilet


 Remain dry for several hours or through a nap
 Pull clothes up and down
 Understand a two-step command
 Express the need to use the toilet (urge to defecate or urinate)
 Imitate the toilet habits of adults or older siblings
 Express an interest in toilet training

(Option 1)  In order to achieve toilet training, the child will need to be able to pull clothing up and down but not
necessarily dress and undress autonomously.

(Option 2)  Having the child sit on the toilet until urination occurs is not appropriate and will not facilitate
bladder control; any urination that occurs is accidental and not due to sphincter control.  However, the child
should have the ability to remain on the toilet for about 5 – 8 minutes without getting off or crying.

(Option 4)  Age 15 months is too early to begin toilet training; voluntary control of the anal and urethral
sphincters does not occur until age 18-24 months.

Educational objective:
Readiness for toilet training is dependent on the child's ability to voluntarily control the anal and urethral
sphincters, which usually occurs at age 18-24 months.  Other developmental and behavioral indicators of toilet
training readiness include the child's ability to express the urge to defecate or urinate, understand simple
commands, pull clothing up and down, and walk to and sit on the toilet.

Retinoblastoma, a unilateral or bilateral retinal tumor, is the most common childhood intraocular malignancy. 
It is typically diagnosed in children under age 2 and is usually first recognized when parents report a white
"glow" of the pupil (leukocoria).  Light reflecting off the tumor will cause the pupil to appear white instead of
displaying the usual red reflex (Option 1).  Parents may even accidentally visualize leukocoria when taking a
photograph of the child using a flash.  Strabismus (misalignment of the eyes) is the second most common sign;
visual impairment is a late sign indicative of advanced disease.

Treatment depends on severity and may include radiation therapy or enucleation (removal of the eye) and fitting
for prosthesis.  Siblings should undergo regular ocular screening, as some forms of retinoblastoma are
hereditary.
(Option 2)  A fixed and mid-dilated pupil is seen in acute glaucoma.

(Option 3)  Ptosis is drooping of the upper eyelid often associated with injury of the oculomotor nerve (cranial
nerve III).  Ptosis is also characteristic for myasthenia gravis.

(Option 4)  Bacterial conjunctivitis causes eyelid swelling, a red conjunctiva, eye discomfort, and purulent eye
discharge.

Educational objective:
Retinoblastoma is a malignancy of the eye typically diagnosed in children under age 2.  The pupil will reflect a
white color, and the child will not have the normal red eye reflex.  Treatment may include radiation therapy or
enucleation (removal of the eye).

Radiation therapy to the head and neck can decrease a client's oral intake due to the development of mucositis
(ie, inflammation of the mouth, esophagus, and oropharynx) and xerostomia (ie, dry mouth).  These adverse
side effects affect speech, taste, and ability to swallow and can have a significant impact on the client's
nutritional status.

The nurse teaches the client to:

 Avoid irritants such as spicy, acidic, dry, or crumbly foods; coffee; and alcohol (Option 1).
 Consume supplemental nutritional drinks (eg, Ensure), which are often easier to swallow (Option 3).
 Use artificial saliva to manage xerostomia and the production of thick saliva due to altered salivary
gland function (Option 5).  Sipping water throughout the day is equally effective and less expensive.

(Option 2)  Topical anesthetics (eg, lidocaine) have been found to increase comfort and improve oral intake in
clients with mucositis due to radiation therapy.

(Option 4)  Clients on radiation therapy need to maintain more frequent (eg, before and after meals, at bedtime)
oral hygiene (eg, using soft toothbrush, rinsing with baking soda solution) due to the drying effects of mucositis.

Educational objective:
Radiation therapy to the head and neck can cause mucositis (ie, inflammation of the mouth, esophagus, and
oropharynx) and xerostomia (ie, dry mouth), leading to decreased nutrition.  Care includes avoiding irritants,
consuming supplements, using artificial saliva or sipping water, and performing frequent oral hygiene.

The suction control chamber (Section A) maintains and controls suction to the chest drainage system;
continuous, gentle bubbling indicates that the suction level is appropriate.  The amount of suction is controlled
by the amount of water in the chamber and not by wall suction.  Increasing the amount of wall suction would
cause vigorous bubbling but does not increase suction to the client as excess suction is drawn out through the
vent of the suction control chamber.  Vigorous bubbling would increase water evaporation and therefore
decrease the negative pressure applied to the system.  The nurse should check the water level and add sterile
water, if necessary, to maintain the prescribed level.

(Option 1)  The air leak monitor (Section C) is part of the water seal chamber.  Continuous or intermittent
bubbling seen here indicates the presence of an air leak.

(Option 2)  The collection chamber (Section D) is where drainage from the client will accumulate.  The nurse
will assess amount and color of the fluid and record as output.
(Option 4)  The water seal chamber contains water, which prevents air from flowing into the client.  Up and
down movement of fluid (tidaling) in Section B would be seen with inspiration and expiration and indicates
normal functioning of the system.  This will gradually reduce in intensity as the lung reexpands.

Educational objective:
The nurse should observe gentle, continuous bubbling in the suction control chamber.  This indicates patency
and the appropriate level of suction being applied to the drainage system.

Conditions that increase intraabdominal pressure (eg, pregnancy, obesity, ascites, tumors, heavy lifting) and
weaken the muscles of the diaphragm may allow a portion of the stomach to herniate through an opening in
the diaphragm, causing a hiatal hernia.  A sliding hernia occurs when a portion of the upper stomach squeezes
through the hiatal opening in the diaphragm.  A paraesophageal hernia (rolling hernia) occurs when the
gastroesophageal junction remains in place but a portion of upper stomach folds up along the esophagus and
forms a pocket.  Paraesophageal hernias are a medical emergency.

Although hiatal hernias may be asymptomatic, many clients experience signs and symptoms commonly
associated with gastroesophageal reflux disease (GERD), including heartburn, dysphagia, and pain caused by
increased intraabdominal pressure or supine positioning.  Interventions to reduce herniation include the
following:

 Diet modification—avoid high-fat foods and those that decrease lower esophageal sphincter pressure
(eg, chocolate, peppermint, tomatoes, caffeine).  Eat small, frequent meals, and decrease fluid intake
during meals to prevent gastric distension.  Avoid consumption of meals close to bedtime and nocturnal
eating (Option 3).
 Lifestyle changes—smoking cessation, weight loss (Option 2).
 Avoid lifting or straining (Option 5).
 Elevate the head of the bed to approximately 30 degrees—this can be done at home using pillows or 4 -
6 inch blocks under the bed (Option 1).

(Option 4)  Wearing a girdle or tight clothes increases intraabdominal pressure and should be avoided.

Educational objective:
Hiatal hernias occur due to a weakening diaphragm and increased intraabdominal pressure.  Nursing
interventions to prevent hiatal hernias are similar to those used for gastroesophageal reflux disease (GERD), and
they focus on decreasing intraabdominal pressure.

Prior to starting therapy with statin medications (eg, rosuvastatin, simvastatin, pravastatin, atorvastatin), the
client's liver function tests should be assessed.  The drug is metabolized by the hepatic enzyme system and
could cause drug-induced hepatitis and increased liver enzymes.  Liver function tests should be assessed prior to
the start of therapy.

(Option 1)  Statins can also cause muscle aches and, rarely, severe muscle injury (rhabdomyolysis).  Clients
should be educated to report the development of muscle pains while on therapy.  Assessment of muscle strength
is not necessary prior to starting therapy.

(Option 2)  Assessment of dietary intake prior to therapy is not essential.  Dietary teaching would have been
performed prior to determining that medication therapy was necessary.

(Option 3)  Atorvastatin may slightly increase serum digoxin levels; however, it is not essential to determine if
the client is on this medication prior to starting therapy.
Educational objective:
Statin medications (eg, rosuvastatin, atorvastatin) can cause hepatotoxicity and muscle aches.  Liver function
tests should be assessed prior to the start of therapy.

Benzodiazepines (eg, alprazolam [Xanax], lorazepam [Ativan], clonazepam, diazepam) are commonly used
antianxiety drugs.  They work by potentiating endogenous GABA, a neurotransmitter that decreases excitability
of nerve cells, particularly in the limbic system of the brain, which controls emotions.  Benzodiazepines may
cause sedation, which can interfere with daytime activities.  Giving the dose at bedtime will help the client
sleep.

(Option 1)  Eliminating aged cheeses and processed meats, which contain tyramine, is necessary with
monoamine oxidase inhibitors (eg, tranylcypromine, phenelzine), which are used for depressive disorders.  It is
not necessary with benzodiazepines.

(Option 2)  A benzodiazepine should never be stopped abruptly.  Instead, it should be tapered gradually to
prevent rebound anxiety and a withdrawal reaction characterized by increased anxiety, confusion, and more.

(Option 4)  Photosensitivity is a problem with most antipsychotics and many antidepressants, but not with
benzodiazepines.

Educational objective:
Benzodiazepines have a sedative effect and should be administered at bedtime when possible.  Benzodiazepines
should never be stopped abruptly in long-term users as this can precipitate withdrawal symptoms.

The experienced nurse should intervene if the new RN starts BP measurement with the client in the standing
position.  Orthostatic BP measurement may be done to detect volume depletion or postural hypotension caused
by medications or autonomic dysfunction.

Procedure for measurement of orthostatic BP

1. Have the client lie down for at least 5 minutes (Option 1)


2. Measure BP and HR
3. Have the client stand
4. Repeat BP and HR measurements after standing at 1- and 3-minute intervals (Option 4)

A drop in systolic BP of ≥20 mm Hg or in diastolic BP of ≥10 mm Hg, or experiencing lightheadedness or


dizziness is considered abnormal (Option 2).

Educational objective:
To measure orthostatic BP, the nurse should have the client lie supine for 5-10 minutes and then measure BP
and HR.  The nurse should then have the client stand for 1 minute, measure BP and HR, and repeat the
measurements at 3 minutes.  Findings are significant if the systolic BP drops ≥20 mm Hg or the diastolic BP
drops ≥10 mm Hg.

Chest tube drainage >3 mL/kg/hr for 3 consecutive hours or 5-10 mL/kg in 1 hour should be reported
immediately to the HCP.  This could indicate postoperative hemorrhage and needs immediate intervention. 
Cardiac tamponade can develop rapidly in children and can be life-threatening.  This child weighs 4 kg and an
output of 30 mL in 1 hour is excessive.

(Option 2)  For infants age 1-12 months, the normal heart rate is 100-160/min.  However, this can be increased
slightly with crying or surgery.
(Option 3)  Hypothermia is common after surgery and warmers are often used.

(Option 4)  Hourly urine output should be measured in the postoperative infant.  A urinary catheter is often
placed during surgery.  Urine output should be between 1-2 mL/kg/hr.

Educational objective:
The nurse should immediately report chest tube drainage >3 mL/kg/hr over 3 consecutive hours or 5-10 mL/kg
over 1 hour.  This could indicate postoperative hemorrhage.  Cardiac tamponade can occur rapidly in children
and can be life-threatening.

Difficulty swallowing is the most important symptom to report to the HCP.  A thoracic aortic aneurysm can
put pressure on the esophagus and cause dysphagia.  The development of this symptom may indicate that the
aneurysm has increased in size and may need further diagnostic evaluation and treatment.

(Option 1)  This BP reading is slightly elevated.  The nurse would need to assess further to find out if this is a
typical BP for this client.  Given the client's history of aneurysm, this elevated BP may warrant treatment.

(Option 3)  The nurse would need to assess the client further as there are multiple causes of cough.

(Option 4)  Low back pain would be a concern if the client had a history of abdominal aortic aneurysm.

Educational objective:
The nurse should report swallowing difficulty immediately in a client with a thoracic aortic aneurysm.  This
could indicate that the aneurysm has increased in size and may require treatment.

This client's initial vital signs show tachycardia and hypotension, which are classic signs of hypovolemia.

Normal capillary refill is less than 3 seconds and is an indication of normal hydration and perfusion (Option 1). 
Obligatory urine output is 30 mL/hr, and this client has 90 mL/hr.  Urine output is one of the best indicators of
adequate rehydration (Option 4).  The urine specific gravity is within a normal range (1.003 to 1.030), which
can indicate normal hydration (Option 5).

(Option 2)  Narrowing pulse pressure (the difference between systolic and diastolic) is a sign of hypovolemic
shock and would not indicate adequate rehydration.  The client arrived with a narrow pulse pressure already.

(Option 3)  This is indicative of orthostatic vital signs.  When a client stands, the body normally vasoconstricts
to maintain the blood pressure from the effects of gravity.  If a client is dehydrated, the body has already
maximally vasoconstricted, and there is no compensatory mechanism left to adjust to the position change.

Educational objective:
Signs of adequate hydration are normal urine specific gravity (1.003 to 1.030), adequate volume of urine output
(>30 mL/hr), and capillary refill of less than 3 seconds.  Pulse pressure narrows in shock, and positive
orthostatic vital signs (decreasing systolic blood pressure and rising heart rate) with position change indicate
dehydration.

A reduction or energy expenditure of 3,500 calories (kcal) will result in a weight loss of 1 lb (0.45 kg).  To lose
20 lb (9 kg), the client needs to reduce intake by a total of 70,000 kcal (3500 kcal x 20 lb [9 kg] = 70,000 kcal). 
Over a period of 16 weeks, this would require a daily reduction of:

625 kcal (70,000 kcal / [16 weeks x 7 days] = 625 kcal/day)


Adding an exercise regimen to the client's daily routine would facilitate additional weight loss and/or reduce the
need for severe caloric restriction.

(Option 1)  Reducing intake by 450 kcal/day over 16 weeks would result in a weight loss of 14.5 lb (6.5 kg).

(Option 3)  Reducing intake by 860 kcal/day over 16 weeks would result in a weight loss of 27.5 lb (12.4 kg).

(Option 4)  Reducing intake by 1000 kcal/day over 16 weeks would result in a weight loss of 32 lb (14.5 kg).

Educational objective:
A reduction or energy expenditure of 3500 calories (kcal) will result in a weight loss of 1 lb.

Oropharyngeal candidiasis, or thrush (moniliasis), is an infection of the mucous membranes generally caused by
the yeastlike fungus Candida albicans.  The fungus causes pearly, "milk-curd" lesions on the oral or laryngeal
mucosa that may bleed when removed.

Immunosuppressed individuals such as those taking corticosteroid medications, clients undergoing


chemotherapy or radiation, or clients with immune deficiency states (eg, AIDS) have an increased incidence. 
Clients receiving prolonged or high-dose antibiotic treatment are at increased risk as the normal microbial
flora of the mouth is reduced, allowing other opportunistic infections to arise (Option 2).  Individuals with
dentures and infants also commonly experience monilial infections.  Treatment is antifungal medications (eg,
nystatin) and proper oral hygiene.

(Option 1)  Inhaled beta-2 agonists (eg, albuterol) do not increase the risk for fungal infections.  However,
individuals taking an inhaled corticosteroid (eg, budesonide, fluticasone) are at increased risk for oral
candidiasis.  To reduce this risk, the client should rinse the mouth after each inhaled dose and maintain good
oral hygiene.

(Options 3 and 4)  Proper oral hygiene and nutrition are important in prevention of oral candidiasis.  However,
the client with braces or poor hygiene and inadequate nutrition is at lower risk than one who is
immunosuppressed or taking antibiotics.

Educational objective:
Immunosuppressed clients (eg, taking steroids, undergoing chemotherapy or radiation, with immunodeficient
states) and those taking prolonged or high-dose antibiotics are at increased risk of oral candidiasis.  Elderly
clients with dentures are also at high risk.  Infection is treated with antifungals (eg, nystatin) and proper oral
hygiene.

A client who has undergone ostomy surgery must become independent in self-care.  This requires adaptation
to a significant alteration in body image and dealing with a number of psychosocial issues that are associated
with a change in appearance and the loss of bowel control.  It is not uncommon for a client to cope with this loss
by refusing to look at or participate in the care of the stoma.

Nursing interventions for this client will include:

 Supportive counseling and assistance in psychosocial adjustment


 Teaching and facilitating self-care
 Providing information about the reason for the surgery, prognosis, potential complications, and
community resources
The priority outcome of nursing care is that the client will look at and touch the stoma; this is an indication that
the client has accepted or begun to accept the change in body image and functioning and can begin participating
in self-care.

(Option 1)  This is an appropriate outcome; community organizations can offer support and educational
materials to the client; however, it is not the priority.

(Option 3)  This is an appropriate outcome, but as a passive activity, it is not a strong indicator that the client is
ready for self-care.

(Option 4)  This is an appropriate outcome as it indicates effective ostomy teaching; however, it is not the
priority.  Teaching will be more effective once the client has accepted the ostomy.

Educational objective:
A client with a change in body image and functioning, such as the creation of an ostomy, will need to adapt to
and cope with the significant changes.  Support and teaching will assist the client in overcoming psychosocial
barriers to self-care; performance of a desired action is the strongest indicator of learning and acceptance.

The client is exhibiting the cardinal signs and symptoms of diabetic ketoacidosis (DKA).  DKA is an acute
life-threatening complication, typically of type I diabetes, characterized by hyperglycemia, ketosis, and
acidosis.  It is caused by an intense deficit of insulin.  Glucose cannot be used properly for energy when this
deficit occurs and the body begins to break down fat stores, producing ketones, a byproduct of fat metabolism,
resulting in metabolic acidosis.  The lack of insulin also results in increased production of glucose in the liver,
further exacerbating hyperglycemia.  Because some of the symptoms of hypoglycemia and DKA overlap, a
blood glucose level should be checked to ensure that hyperglycemia is present.

Hyperglycemia can cause osmotic diuresis, leading to dehydration.  In addition, ketones are excreted in the
urine as the body tries to restore its pH balance.  Vital electrolytes such as sodium, potassium, chloride,
phosphate, and magnesium become depleted during the process.  Cardinal signs of dehydration such as poor
skin turgor, dry mucous membranes, tachycardia, orthostatic hypotension, weakness, and lethargy can occur.

The nurse should start an IV and bolus the client with normal saline or 1/2 normal saline to reverse
dehydration.  This should occur prior to treating the hyperglycemia with regular insulin IV infusion.  Because
insulin promotes water, potassium, and glucose entrance into the cell, it can exacerbate vascular dehydration
and imbalance of electrolytes, particularly potassium.  A potassium level (along with other electrolytes) should
also be assessed prior to beginning the prescribed insulin therapy.

Other signs associated with DKA include Kussmaul respirations, deep, rapid respirations that have a
fruity/acetone smell as the carbon dioxide is exhaled.  This compensatory mechanism results in a lowered PaC02
in an attempt to restore the body's normal pH level and should not be reversed (Option 2).

(Option 1)  IV dextrose is administered during acute hypoglycemic episodes and would worsen DKA.

Educational objective:
DKA is an acute life-threatening complication, typically of type I diabetes, characterized by hyperglycemia,
ketosis, and acidosis.  It is caused by an intense deficit of insulin and should be treated first with rehydration
(normal saline) and then insulin administration.

Many individuals diagnosed with cancer experience anxiety and fear related to death and desire to talk with
someone about these feelings.  To promote a therapeutic relationship, the nurse should initiate conversations by
acknowledging clients' fears, use open-ended statements to invite them to talk about death, and actively listen
as they verbalize their feelings.
(Option 1)  The nurse offers false reassurance by making this statement.  Providing false reassurance is not part
of a therapeutic relationship or an effective communication strategy.

(Option 2)  This statement does not acknowledge the client's concerns and blocks communication.  The nurse
should first assess the client's cultural and spiritual practices.  If the client requests spiritual support, then the
nurse may make a referral to the chaplain's office.

(Option 4)  By changing the subject, the nurse is attempting to redirect the conversation away from the client's
desire to talk about death; this does not promote a therapeutic relationship.

Educational objective:
Fear of dying is a common concern for many clients with a terminal disease.  The nurse should acknowledge
these feelings and use open-ended statements and active listening to invite clients to talk about death.

C difficile poses a unique hazard in health care settings.  This infection of the colon may develop/spread
through contact with the organism or after prolonged antibiotic therapy alters normal bowel flora, allowing for
C difficile overgrowth.  Clients with C difficile infection should be placed on strict contact precautions in
private rooms.  These precautions require staff to wear protective gowns and gloves when entering the client's
room (Options 4 and 5).  Hand hygiene using soap and water is the only effective method for removing C
difficile spores (Option 2).  In addition, alcohol is not an effective agent for killing C difficile spores; therefore,
a diluted bleach solution must be used to disinfect contaminated equipment and surfaces (Option 1).

(Option 3)  Contact precautions require the caregiver to wear a gown and gloves.  A face mask must be worn as
personal protective equipment if an organism is spread via droplets.  However, it is not required to prevent the
spread of a contact-transmissible infection.  The nurse should not wear a mask solely to avoid the unpleasant
odor associated with C difficile diarrhea as this may be offensive and embarrassing to the client.

Educational objective:
C difficile infection requires strict contact precautions, including wearing a gown and gloves at all times. 
Alcohol cannot kill C difficile spores, so caregivers must use soap and water in place of alcohol-based hand
sanitizers.  Contaminated surfaces and equipment should be disinfected using a diluted bleach solution.

Group B Streptococcus (GBS) may be present as part of normal vaginal flora in up to 30% of pregnant clients. 
Although colonization with GBS rarely poses harm to the client, it can be transmitted to the newborn during
labor and birth, resulting in serious complications (eg, neonatal GBS sepsis, pneumonia).  Pregnant clients are
tested for GBS colonization at 35-37 weeks gestation and receive prophylactic antibiotics during labor if results
are positive.

If GBS status is unknown, antibiotics are typically indicated when membranes have been ruptured for ≥18
hours, maternal temperature is ≥100.4 F (38 C), or gestation is <37 weeks (Option 1).

(Option 2)  Part of the client's assessment includes evaluation of the uterine contraction pattern.  However, the
client and newborn are at risk for infection due to prolonged rupture of membranes and unknown GBS status, so
antibiotic administration is the priority.

(Option 3)  A urine specimen is often collected to evaluate for proteinuria in clients with elevated blood
pressure or to assess for urinary tract infection in symptomatic clients.  Urine specimen collection is not the
priority for this client.

(Option 4)  Vaginal examinations should be limited in the presence of ruptured membranes.  Multiple vaginal
examinations in such a client correlate with an increased risk for infection (eg, chorioamnionitis).
Educational objective:
Group B Streptococcus (GBS) infection can be transmitted to the newborn during labor and birth and cause
serious complications.  Indications for prophylactic antibiotics during labor include maternal GBS-positive
status or unknown GBS status with fever ≥100.4 F (38 C), preterm gestation, and/or prolonged rupture of
membranes.

This client's findings indicate a possible uterine infection (postpartum endometritis).  Clients develop fever,
chills, tachycardia, uterine tenderness, and foul-smelling lochia.  Postpartum endometritis is usually caused
by polymicrobial infection and is treated with broad-spectrum antibiotics.  If the health care provider prescribes
blood cultures (Option 4), they must be obtained prior to initiating antibiotic therapy as the medication may
alter laboratory results (Option 2).  After the results of the blood culture are obtained, the antibiotic prescribed
may be changed for appropriate treatment.

(Option 1)  Acetaminophen administration for fever is not the priority action for this client.  The nurse can treat
the client's fever after blood cultures are drawn and IV antibiotic therapy is initiated.

(Option 3)  Placement of a saline IV lock for administration of antibiotics may be completed after obtaining a
blood culture.  Drawing a blood culture from an IV site is not recommended.  Careful preparation of the skin
prior to needle puncture will decrease the chance of specimen contamination.

Educational objective:
Postpartum uterine infection may occur following vaginal or cesarean delivery.  A blood culture is obtained
prior to starting broad-spectrum antibiotics.

Mastitis is a common infection in postpartum women due to multiple risk factors leading to inadequate milk
duct drainage (eg, poor latch).  Bacteria are transmitted from the infant's nasopharynx or the mother's skin
through the nipple and multiply in stagnant milk.  Staphylococcus aureus is the most common offending
organism.  Symptoms of mastitis include fever, breast pain, and focal inflammation (redness, edema).

In addition to antistaphylococcal antibiotics (dicloxacillin or cephalexin) and analgesics (eg, ibuprofen),


treatment of lactational mastitis requires effective and frequent milk drainage.  Milk ducts are most efficiently
drained by direct breastfeeding.  The correct position for optimal milk intake involves the infant forming a
tight seal around most of the areola.  However, a common cause of severe pain during latching occurs when the
infant only suckles on the nipple.  This improper position can cause nipple blistering.  Adequate rest and
increase of oral fluid intake is also recommended.

(Options 2 and 3)  Breastfeeding should be continued every 2-3 hours to relieve milk duct obstruction. 
Mothers should be reassured that the infant can safely feed from the infected breast as the newborn is already
colonized with the mother's skin flora.

(Option 5)  Underwire bras (tight bras) are not recommended with breastfeeding or mastitis as milk flow is
impeded, worsening engorgement.  Soft cup bras are recommended for support and to encourage milk flow.

Educational objective:
Treatment of lactational mastitis includes antibiotic therapy, breast support, adequate hydration, analgesics, and
frequent continued breastfeeding (every 2-3 hours).

Alzheimer disease is a neurological condition that increases the risk for aspiration and aspiration pneumonia, a
common cause of death in clients with swallowing dysfunction.

The nurse activates a rapid response because the client is in acute respiratory distress.  While waiting for the
team, the nurse should implement the following actions in order:
1. Place in high Fowler's position – quickly maximizes ability to expand lungs, promotes oxygenation,
and helps to decrease risk of further aspiration
2. Perform oropharyngeal suctioning – the priority is clearing the airway after the client has been placed
in a position that prevents further aspiration
3. Administer 100% oxygen by nonrebreather mask – corrects hypoxemia/hypoxia once the airway has
been cleared to allow passage of oxygen.  The nurse has already gathered focused assessment data and
determined the need for emergent oxygen delivery (eg, tachycardia, tachypnea, hypoxia, cyanosis,
decreased level of consciousness).
4. Assess lung sounds – determines air movement and presence of adventitious sounds (eg, crackles,
wheezing, stridor) that can indicate obstruction, secretions, atelectasis, or fluid.  This assessment is
performed once emergency measures are in place (eg, oxygen) and the client has been stabilized.
5. Notify the primary HCP – to report the situation and assessment data

To provide more efficient care, any of these tasks can be delegated to a second RN.

Educational objective:
While waiting for the rapid response team to respond to an adult client with acute respiratory distress, the nurse
implements the following actions: positioning; suctioning to clear the airway, administering high-concentration
oxygen; assessing lung sounds; and notifying the HCP.

The prioritization principle is that systemic symptoms are more important than local symptoms.  Trousseau's
sign (carpal spasm with blood pressure cuff inflation) indicates hypocalcemia.  This is a known risk after a
thyroidectomy as the parathyroid gland can be inadvertently removed during the surgery due to its very small
size.  Acute hypocalcemia can cause tetany, laryngeal stridor, seizures, and cardiac dysrhythmias.  Assessing
this client is a priority over pain or expected findings.

(Option 1)  This client likely has postoperative urinary retention and needs to be evaluated as soon as possible
(second in priority).  Although, this condition is painful and could result in kidney injury, it is not immediately
life-threatening.

(Option 2)  This client has isolated systolic hypertension, which is common in elderly clients and they are often
asymptomatic.  Systolic blood pressure is usually >160 mm Hg but diastolic blood pressure is <90 mm Hg. 
Treatment might benefit these clients, but this is not a priority.

(Option 3)  ALT and AST are enzymes released when hepatocytes are destroyed as part of the hepatitis
pathology.  Hepatitis is diagnosed when these enzymes are ≥2-3 times the normal value.  The hepatitis C virus
usually causes chronic infection.  The client's acuity is not directly related to the level of enzymes; this client is
not more seriously ill because the enzymes are higher than a client whose labs results are twice the normal
value.  This is an expected finding and is not a priority.

Educational objective:
Acute hypocalcemia can be life-threatening due to seizures, tetany (laryngeal stridor), and cardiac arrhythmias. 
Inadvertent removal of the parathyroid gland can result in rapid development of severe hypocalcemia.

When caring for a client at risk for venous thromboembolism (VTE), the registered nurse (RN) is responsible
for assessing the client, including risk factors present, current medications and supplements that affect
coagulation (eg, ginger, gingko biloba), and development of complications (eg, pulmonary embolism).  The RN
must also develop and implement the plan of care, provide client teaching, and evaluate the outcome of
interventions.  The RN may delegate care to licensed practical nurses (LPNs) and unlicensed assistive personnel
using the five rights of delegation.
It is within the scope of practice of the LPN to administer most anticoagulant medications and measure and
apply compression devices (Options 1, 2, and 4).

(Option 3)  The LPN can collect and report data (eg, vital signs, results from complete blood count and
coagulation studies), but analysis and evaluation of the data require the RN's expertise.

(Option 5)  The RN must provide initial education about VTE prevention (eg, medications, preventive
measures, bleeding precautions).  The LPN may then reinforce teaching to clients as necessary.

Educational objective:
The registered nurse (RN) is responsible for assessing the client at risk for venous thromboembolism,
developing and implementing the plan of care, teaching, and evaluating interventions.  The RN may delegate
anticoagulant administration, measurement and application of compression devices, and teaching reinforcement
to the licensed practical nurse.

Parkinson disease is caused by low levels of dopamine in the brain.  Levodopa is converted to dopamine in the
brain, but much of this drug is metabolized before reaching the brain.  Carbidopa helps prevent the breakdown
of levodopa before it can reach the brain and take effect.  This combination medication is particularly effective
in treating bradykinesia (generalized slowing of movement).  Tremor and rigidity may also improve to some
extent.  Carbidopa-levodopa (Sinemet) once started should never be stopped suddenly as this can lead to
akinetic crisis (complete loss of movement).  However, prolonged use can also result in dyskinesias
(spontaneous involuntary movements) and on/off periods when the medication will start or stop working
unpredictably. 

(Option 1)  Carbidopa-levodopa does not improve memory.  Medications for the treatment of Alzheimer
disease, such as donepezil and rivastigmine, are used to improve cognition and memory.

(Options 3 and 4)  Orthostatic hypotension and neuropsychiatric disturbances (eg, confusion, hallucinations,
delusions, agitation, psychosis) are serious and important adverse effects of carbidopa-levodopa.  Health care
providers usually start the medications at low doses and gradually increase them to prevent these effects.

Educational objective:
The combination medication carbidopa-levodopa is most helpful for treating bradykinesia in Parkinson disease
and can also improve tremor and rigidity to some extent.  It is started in low doses to prevent orthostatic
hypotension and neuropsychiatric adverse effects.  Carbidopa-levodopa once started should never be stopped
suddenly as doing so can lead to akinetic crisis (complete loss of movement).

Bronchiolitis is a lower respiratory tract infection most commonly caused by respiratory syncytial virus.  It
causes inflammation and obstruction of the lower respiratory tract.  Depending on the severity of the infection,
infants with bronchiolitis can experience mild cold symptoms or respiratory distress.  The infant will have
difficulty feeding and can become dehydrated.  Medical care is supportive and includes suctioning, oxygen,
and hydration.  The infant with irritability may be exhibiting signs of hypoxia.  The nurse should see this
client first.

(Option 1)  Intussusception can be reduced with hydrostatic enema (nonoperative approach).  This is important
but is not a priority over a child with bronchiolitis and respiratory distress.

(Option 2)  Chemotherapy can result in neutropenia and immunosuppression.  Even a low-grade fever should
be taken seriously as it can result in lethal sepsis.  The client needs cultures and empiric antibiotics.  However,
the client with bronchiolitis is the priority.
(Option 3)  Increased intracranial pressure will occur with shunt malfunction.  The nurse should routinely
measure the head circumference, but it is not a priority over a client with respiratory distress.

Educational objective:
A client with bronchiolitis will require frequent suctioning, especially before feeding.  The nurse should use the
ABC (airway, breathing, circulation) guidelines and see this client first.

A client with narcissistic personality disorder (NPD) exhibits a recurrent pattern of grandiosity, need for
admiration, and lack of empathy.  Clients with NPD may project a picture of superiority, uniqueness, and
independence that hides their true sense of emptiness.

From a psychodynamic perspective, individuals with NPD have a fragile and damaged ego resulting from a
childhood environment that fostered a sense of inferiority, poor self-esteem, and severe self-criticism. 
Narcissistic characteristics develop as a way to regulate self-esteem and protect the ego from further psychic
injury.

(Option 2)  Delusions of grandeur are experienced by clients with a psychotic disorder; NPD is a personality
disorder.

(Option 3)  Clients with NPD may feel threatened if criticized or if others do not meet their emotional
demands.  However, this is not the best explanation of the clinical characteristics associated with NPD.

(Option 4)  Panic attacks are characteristic of clients with an anxiety disorder, not NPD.

Educational objective:
The clinical characteristics of narcissistic personality disorder can best be explained as an attempt to maintain a
fragile self-esteem that was damaged during childhood due to an environment that was highly critical,
demanding, and fostered a sense of inferiority.

Children age <1 year should not be given honey (in raw or wild form) due to the risk of infant botulism. 
Botulism can vary in severity from constipation to respiratory failure secondary to loss of neurologic
function.  Infants initially have constipation, generalized weakness, and diminished deep-tendon reflexes. 
Additional symptoms include lack of head control, difficulty feeding, and decreased gag reflex.

Infants with botulism should be admitted to the intensive care unit for monitoring of respiratory status,
nasogastric tube feedings, laxative medication, and physical and occupational therapies.  Intravenous human-
derived botulism immune globulin should be given as soon as possible to reduce the severity and duration of
symptoms.

(Option 2)  The client with elbow pain may need an x-ray to rule out fracture but is the lowest priority.

(Option 3)  The client with acute appendicitis needs surgery as soon as possible.  However, loss of respiratory
function in botulism is the priority.  This client is the second priority.

(Option 4)  Urinary tract infection without pyelonephritis (eg, nausea, vomiting, shaking chills) is not an
emergency.  This client is the third priority.

Educational objective:
Symptoms of infant botulism include constipation, generalized weakness, difficulty feeding, and diminished
deep-tendon reflexes.  This condition is a high priority due to the risk of respiratory failure.
Topical capsaicin cream (Zostrix) is an over-the-counter analgesic that effectively relieves minor pain (eg,
osteoarthritis, neuralgia).  The nurse should instruct the client to wait at least 30 minutes after massaging the
cream into the hands before washing to ensure adequate absorption (Option 2).  The client should avoid
contact with mucous membranes (eg, nose, mouth, eyes) or skin that is not intact, as capsaicin is a component
of hot peppers and can cause burning.  When applying cream to other areas of the body (eg, knee), the client
should wear gloves or wash hands immediately after application.

(Option 1)  The application of heat with capsaicin is contraindicated as heat causes vasodilation, which
increases medication absorption and can possibly lead to a chemical burn.

(Option 3)  Local irritation (burning, stinging, erythema) is quite common and usually subsides within the first
week of regular use.  If the client experiences persistent pain, redness, or blistering, the cream should be
discontinued and the health care provider notified.

(Option 4)  Topical capsaicin is often used concurrently with acetaminophen or nonsteroidal anti-
inflammatory drugs (eg, naproxen, celecoxib) to effectively treat osteoarthritis pain.  Capsaicin should be used
regularly (3-4 times daily) for long periods (eg, weeks to months) to achieve the desired effect.

Educational objective:
The topical analgesic capsaicin relieves minor peripheral pain (eg, osteoarthritis, neuralgia) with regular use. 
Local irritation (burning, stinging, erythema) is quite common.  The client should wait at least 30 minutes
before washing the affected area to ensure adequate absorption.

An inferior vena cava filter is a device that is inserted percutaneously, usually via the femoral vein.  The filter
traps blood clots from lower extremity vessels (eg, embolus from deep venous thrombosis) and prevents them
from migrating to the lungs and causing a pulmonary embolism (PE).  It is prescribed when clients have
recurrent emboli or anticoagulation is contraindicated.

Clients should be questioned about and report any metallic implants (eg, vascular filters/coils) to the health care
team prior to radiologic imaging, specifically MRI (Option 1).  Physical activity should be promoted, and
clients should avoid crossing their legs to promote venous return from the legs (Option 2).  Leg pain,
numbness, or swelling may indicate impaired neurovascular status distal to the insertion site and should be
reported immediately (Option 3).

(Option 4)  Symptoms of PE (eg, chest pain, shortness of breath) and vascular injury (bleeding causing back
pain) are not expected findings after the procedure and should be reported immediately.

Educational objective:
An inferior vena cava filter traps thrombi migrating from the lower extremities to the lungs.  Discharge teaching
includes promotion of physical exercise, reporting of symptoms of pulmonary embolism (eg, chest pain,
shortness of breath) and impaired lower extremity circulation (eg, pain, numbness), and notification of the
health care team prior to MRI.

An electroencephalogram (EEG) is a diagnostic procedure used to evaluate the presence of abnormal electrical
discharges in the brain, which may result in a seizure disorder.  The EEG can be done in a variety of ways, such
as with the child asleep or awake with or without stimulation.

Teaching for the parent includes the following:

1. Hair should be washed to remove oils and hair care products, and accessories such as ribbons or
barrettes should be removed.  Hair may need to be washed after the procedure to remove electrode gel.
2. Avoid caffeine, stimulants, and central nervous system depressants prior to the test.
3. The test is not painful, and no analgesia is required.

(Option 1)  Food and liquids are not restricted prior to an EEG except for caffeinated beverages.  Cocoa
contains caffeine.

(Option 3)  This test (EEG) is not painful as it only records brain electrical activity.  Electrode gel is
nonirritating to the skin.

(Option 4)  A routine EEG is not performed under sedation, and so the child should remember the procedure.

Educational objective:
An EEG is used to diagnose the presence of a seizure disorder.  Electrodes are secured to the scalp to observe
for abnormal electrical discharges in the brain.  Preprocedure teaching includes avoiding stimulants and CNS
depressants and washing the hair.

Some children have a mild reaction to the MMRV vaccine within 5-12 days after the first dose.  Problems
include low-grade fever, mild rash, swelling and erythema at the injection site, irritability, and restlessness.

Although rare, fever after MMRV vaccination can lead to febrile seizures.  Therefore, it is important for the
nurse to determine the child's temperature to evaluate the risk for a febrile convulsion.  It would also be
important for the nurse to instruct the parent to monitor the child's temperature and administer acetaminophen
for a fever above 102 F (38.9 C).

Children with a history of seizures should be vaccinated with separate MMR and varicella vaccines instead of
the combination MMRV vaccine.

(Option 1)  This is not an appropriate intervention.  The rash should disappear in 2-3 days.

(Option 2)  The child seems to be experiencing a normal reaction to the vaccine; a clinic visit is not necessary.

(Option 3)  Although this is an appropriate response, it is most important for the nurse to first determine the
child's temperature and the extent of the fever.

Educational objective:
The normal MMRV vaccine reactions that occur within 5-12 days after vaccination include mild fever and rash,
irritability and restlessness, and swelling and erythema at the injection site.  Febrile seizure is a rare but more
serious reaction to the vaccine.

Sucralfate is an oral medication that forms a protective layer in the gastrointestinal mucosa, which provides a
physical barrier against stomach acids and enzymes.  It doesn't neutralize or reduce acid production.  It is
prescribed to treat and prevent both stomach and duodenal ulcers.  This medication is generally prescribed 1
hour before meals and at bedtime and, for effective results, is administered on an empty stomach with a glass
of water (Option 1).

Sucralfate also binds with many other medications (eg, digoxin, warfarin, phenytoin) and reduces their
bioavailability.  Therefore, all other medications are generally administered at least 1-2 hours before or after
sucralfate administration (Option 3).  Constipation is a common side effect of this medication (Option 4).

(Option 2)  Sucralfate forms a better protective layer at low pH.  Therefore, antacids or other acid-reducing
medications (eg, proton pump inhibitors or H2 blockers) should not be taken 30 minutes before or after taking
sucralfate to avoid altered absorption.
Educational objective:
Sucralfate should be taken on an empty stomach with a glass of water.  Sucralfate forms a better protective layer
at low pH.  Therefore, acid-reducing agents (eg, antacids, proton pump inhibitors, H2 blockers) should be
avoided 30 minutes before and after administration to avoid altered absorption.  Other medications should be
administered 1-2 hours before or after sucralfate.

If applicable, the nurse requests that the client remove contact lenses.  The nurse then dons clean gloves and
uses aseptic technique to administer ophthalmic medications (eg, eye drops, lubricant) that lubricate the eye and
treat eye conditions (eg, glaucoma, infection).  The Joint Commission disallows the use of abbreviations for
right eye (OD), left eye (OS), and both eyes (OU).  The nurse must verify the prescription if the health care
provider (HCP) uses these abbreviations.

The general procedure for the administration of ophthalmic medications includes the following steps in
sequence:

1. Remove dried secretions with moistened (warm water or normal saline) sterile gauze pads by wiping
from the inner to outer canthus to keep eyelid and eyelash debris from entering the eye and to prevent
transfer of debris into the lacrimal (tear) duct (Option 3)

2. Place client in the supine or sitting position with head tilted back toward side of the affected eye to
prevent excess medication from flowing into the lacrimal duct and minimize systemic absorption
through the nasal mucosa

3. Rest hand on client's forehead and hold dropper 1-2 cm (1/2-3/4 in) above the conjunctival sac, which
keeps the dropper away from the eye globe and avoids contamination (Option 4)

4. Pull lower eyelid down gently with thumb or forefinger against bony orbit to expose the conjunctival
sac (Option 2)

5. Instruct client to look upward and then instill drops of medication into the conjunctival sac.  This
minimizes the blink reflex and retracts the cornea up and away from the conjunctival sac to avoid
instillation onto the cornea

6. Instruct client to close the eyelid and move the eye around (if able).  Then apply pressure to the
lacrimal duct for 30-60 seconds if medication has systemic effects (eg, beta blocker, timolol maleate
[Timoptic]).  This will distribute the medication, prevent overflow into the lacrimal duct, and reduce
possible systemic absorption (Option 1)

7. Remove excess medication from each eye with a new tissue or gauze pad to prevent cross-contamination

8. Wait 5 minutes before instilling a different medication into the same eye

Educational objective:
To administer ophthalmic medications, follow these steps:  (1) Remove secretions from the eyelid by wiping
from the inner to outer canthus; (2) pull lower eyelid downward, have client look upward, and instill drops into
the conjunctival sac; and (3) apply pressure to the lacrimal duct if medication has systemic effects (eg, beta
blocker, timolol maleate).

Client teaching related to peptic ulcer disease (PUD) includes lifestyle changes (eg, dietary modifications,
stress reduction), PUD complications, and medication administration.  Helicobacter pylori infection and
treatment with nonsteroidal anti-inflammatory drugs (NSAIDs) are risk factors for complicated PUD.  H
pylori treatment includes antibiotics and proton-pump inhibitors for acid suppression.  The recommended initial
treatment is 7-14 days of triple-drug therapy with omeprazole (Prilosec), amoxicillin, and clarithromycin
(Biaxin).

(Option 5)  Clients with PUD should avoid NSAIDs [eg, aspirin, ibuprofen (Motrin)] as they inhibit
prostaglandin synthesis, increase gastric secretion, and reduce the integrity of the mucosal barrier.

Educational objective:
Clients with peptic ulcer disease should avoid NSAIDs, smoking, and excess use of alcohol or caffeine.

Losartan is an angiotensin II receptor blocker (ARB) prescribed to treat hypertension.  ACE inhibitors (eg,
lisinopril, enalapril) and ARBs are teratogenic, causing renal and cardiac defects or death of the fetus.  ARBs
and ACE inhibitors have black box warnings that indicate contraindication in pregnancy.

The nurse should not give an ARB to a pregnant client (Option 2).  The health care provider should be notified
so that an alternate antihypertensive may be prescribed that is safe to take during pregnancy (eg, labetalol,
methyldopa).

(Option 1)  Antiplatelet agents (eg, clopidogrel) are prescribed to prevent thromboembolic events in clients
with increased risk for stroke or myocardial infarction.  Laboratory values are monitored periodically as these
drugs increase bleeding time (normal, 2-7 minutes [120-420 seconds]) and, rarely, may lower platelet count
(normal, 150,000-400,000/mm3 [150-400 × 109/L])

(Option 3)  Bell palsy presents as acute onset of unilateral facial paralysis related to inflammation of the facial
nerve (ie, cranial nerve VII) that may be triggered by a viral illness (eg, herpes simplex virus).  Standard
treatment includes corticosteroids (eg, prednisone) within 72 hours of symptom onset.

(Option 4)  Tiotropium is an inhaled anticholinergic drug that inhibits receptors in the smooth muscles of the
airways.  It is prescribed daily for the long-term management of bronchospasm in clients with chronic
obstructive pulmonary disease.

Educational objective:
Angiotensin II receptor blockers and ACE inhibitors are teratogenic, causing fetal injury or death, and are
contraindicated in pregnancy.

Cardiac catheterization involves injection of iodine contrast using a catheter to examine for obstructed
coronary arteries.

Complications include:

 Allergic reaction:  Clients with a previous allergic reaction to IV contrast may require premedication
(eg, corticosteroids, antihistamines) or another contrast medium (Option 2).  Clients with shellfish
allergies were once believed to be at higher risk, but this has been disproved.

 Contrast nephropathy:  Iodine-containing contrast can cause kidney injury, although this risk can be
reduced with adequate hydration.  However, clients with renal impairment (eg, serum creatinine >1.3
mg/dL [115 µmol/L]) should not receive IV contrast unless absolutely necessary (Option 4).

 Lactic acidosis:  Metformin (Glucophage) with IV iodine contrast increases the risk for lactic acidosis. 
Metformin is usually discontinued 24-48 hours before exposure and restarted after 48 hours, when
stable renal function is confirmed (Option 5).
(Option 1)  C-reactive protein, produced during acute inflammation, may reflect an elevated risk for coronary
artery disease.  However, it does not indicate an acute event and is not a safety concern for this procedure.

(Option 3)  First-degree atrioventricular block (ie, PR interval >0.20 second) may precede more serious
conditions.  However, clients are usually asymptomatic and do not require treatment except for stopping the
causative medication (eg, beta blockers, digoxin).  This would not prevent the test from proceeding.

Educational objective:
Cardiac catheterization uses IV contrast to assess for artery obstruction.  Complications include allergic
reactions, lactic acidosis, and kidney injury.  Contrast is avoided in clients who had a previous allergic reaction
to contrast agents, took metformin in the last 24 hours, or have renal impairment.

Clients with sepsis are at risk for developing disseminated intravascular coagulation (DIC), a condition that
initially causes clotting within the microvessels.  Platelets and clotting factors are consumed in clotting and
become unavailable for body use, leading to bleeding complications.  The initial clotting also disrupts blood
flow to extremities and organs.

Signs of DIC include frank external bleeding (eg, venipuncture site bleeding), signs of internal bleeding
(petechiae, ecchymosis, hematuria, hematemesis, and bloody stools), and respiratory distress (eg,
bleeding/clotting into lungs).  Signs of DIC need immediate assessment and emergency intervention.  Rapid
replacement of clotting factors (fresh frozen plasma), platelets, and blood is needed to save the client from
death.

(Option 1)  Stool leaking from an ileostomy bag is not a priority.

(Option 2)  It is common for clients with chronic obstructive pulmonary disease to have diminished breath
sounds; the goal SpO2 level in this population is generally ≥90%.

(Option 3)  Although missing warfarin can increase the risk of clotting, most clients will usually have a
therapeutic INR for 1-2 days.  This is not a priority over the DIC client.

Educational objective:
Disseminated intravascular coagulation (DIC) results from abnormal activation of clotting cascade followed by
consumption of clotting factors and platelets; this quickly leads to life-threatening external and internal
bleeding.  Any signs of DIC should be assessed immediately as emergent replacement of clotting factors, blood,
and platelets is needed to save the client.

The nurse should question the prescription for furosemide (Lasix), a potent loop diuretic, before administering
the medication.  The client has a significant decrease in systolic blood pressure (50 mm Hg), a negative fluid
balance of 2000 mL for 24 hours, hypernatremia (normal sodium, 135-145 mg/dL [135-145 mmol/L]), and a
potassium level that is trending downward.  These parameters indicate hypotonic dehydration, which is often
caused by diuretic use.  If the diuretic were administered, the fluid volume deficit would increase further.

(Option 2)  Glipizide, an oral sulfonylurea drug used to control blood sugar, is prescribed once or twice a day
30 minutes before meals.  The client's blood sugar is within normal limits (70-99 mg/dL [3.9-5.5 mmol/L]), so
there is no need for the nurse to question the prescription.

(Option 3)  Antibiotic therapy with levofloxacin (Levaquin) is appropriate for a client with a urinary tract
infection, so there is no need for the nurse to question the prescription.

(Option 4)  Potassium chloride is usually prescribed with a diuretic to prevent hypokalemia.  The potassium is
within normal limits (normal, 3.5-5.0 mEq/L [3.5-5.0 mmol/L]) but is trending downward.  A further decrease
in potassium from the diuretic would increase the risk for cardiac dysrhythmias associated with hypokalemia. 
Most clients need a potassium level of around 4.0 mEq/L (4.0 mmol/L) to prevent arrhythmias.  If the
furosemide is discontinued, the health care provider and nurse should check serum potassium levels the next
day to determine whether further dosing is necessary.

Educational objective:
Decrease in blood pressure, increase in pulse rate, output greater than intake, hypernatremia, and decrease in
serum potassium are manifestations that can indicate hypotonic dehydration in a client receiving diuretic
therapy.

The 2 most common causes of pancreatitis are obstructive biliary tract disease from gallstones and alcohol use
disorder (Options 1 and 5).  Other important causes include severe hypertriglyceridemia and post-ERCP
procedure.  Occasionally some medications can also induce pancreatitis (eg, thiazides, sulfonamides).

Chronic pancreatitis may damage the pancreas and is a risk factor for type 2 diabetes mellitus; however, type 1
diabetes mellitus is not a risk factor for developing pancreatitis (Option 4).

Immune thrombocytopenia involves destruction of platelets by an abnormal immune response causing increased
risk of bleeding, but is unrelated to pancreatitis (Option 2).

Psoriasis is a skin condition and is not related to pancreatitis (Option 3).

PID is an acute infection of the upper genital tract.  The most common organisms are Chlamydia and Neisseria
gonorrhea; PID would not be contagious by being in the same room.  There is no infection risk for a client with
gastrointestinal bleeding (Option 4).

Clients with the same organism can room together (Option 5).

(Option 1)  Varicella (chicken pox, herpes zoster) requires airborne precautions (and contact precautions also if
open lesions are present).  Pertussis requires droplet precautions.  Both the precautions and the organisms are
different, and the clients could cross-infect each other.

(Option 2)  An AIIR (formerly negative-airflow room) is indicated when the client has an organism transmitted
by the airborne route (eg, tuberculosis).  No other client should be in the room with a client with this type of
infection, especially one with a significant co-morbidity.

(Option 3)  Chemotherapy causes bone marrow suppression with immunosuppression.  Although the client may
not need reverse or protective isolation (eg, when absolute neutrophil count is ≤500/mm3), an infectious client
should not be placed with this client.  Yellow sputum typically indicates bacterial infection.  COPD clients can
have chronic colored sputum, but infection (bacterial or viral) is the primary cause of exacerbations (the most
likely reason the client is in the hospital).  This is not a safe option.

Educational objective:
For infection control, clients with same organisms can be placed together.  Infectious clients cannot be placed
with immunosuppressed or at-risk clients.

To provide maximum support, when climbing up and down any stairs, the client should keep 2 points of support
on the floor at all times (eg, both feet or a foot and the cane) and move the cane just before moving the weaker
leg, regardless of the direction.

When descending stairs, the client should:


1. Lead with the cane
2. Bring the weaker leg down next (in this client, it is the right leg)
3. Finally, step down with the stronger leg  (Option 2)

When ascending stairs, the client should:

1. Step up with the stronger leg first


2. Move the cane next while bearing weight on the stronger leg
3. Finally, move the weaker leg

The nurse may use the mnemonic "up with the good and down with the bad."  Cane always moves before the
weaker leg.

(Options 1, 3, and 4)  These options do not provide as much support to the weaker leg when descending.

Educational objective:
To reduce the risk of falls when descending the stairs using a cane, the client should lead with the cane, follow
with the weaker leg, and then step down with the stronger leg.

Patent ductus arteriosus (PDA) is an acyanotic congenital defect more common in premature infants.  When
fetal circulation changes to pulmonary circulation outside the womb, the ductus arteriosus should close
spontaneously.  This closure is caused by increased oxygenation after birth.  If a PDA is present, blood will
shunt from the aorta back to the pulmonary arteries via the opened ductus arteriosus.  Many newborns are
asymptomatic except for a loud, machine-like systolic and diastolic murmur.  The PDA will be treated with
surgical ligation or IV indomethacin to stimulate duct closure.

(Option 1)  A harsh systolic murmur is heard in the setting of ventricular septal defect, an opening between the
ventricles of the heart.  Ventricular septal defect is an acyanotic defect.

(Option 3)  A diastolic murmur is heard in mitral stenosis and aortic regurgitation but not in PDA.

(Option 4)  A systolic ejection murmur is heard in pulmonic stenosis.  Right ventricular hypertrophy will
develop if this defect is not repaired.  In adults, systolic ejection murmur is usually due to aortic stenosis.

Educational objective:
The ductus arteriosus of a newborn should close spontaneously when fetal circulation changes to pulmonary
circulation.  If the ductus arteriosus remains open, blood will shunt from the aorta to the pulmonary arteries. 
The child will be acyanotic but will have a machine-like murmur heard on both systole and diastole.

Serum cardiac markers are proteins released into the bloodstream from necrotic heart tissue after a myocardial
infarction (MI).  Troponin is a highly specific cardiac marker for the detection of MI.  It has greater sensitivity
and specificity for myocardial injury than creatine kinase (CK) MB.  Serum levels of troponin increase 4–6
hours after the onset of MI, peak at 10–24 hours, and return to baseline after 10–14 days.

A troponin value of 0.7 ng/mL (0.7 mcg/L) indicates cardiac muscle damage and should be the priority and
immediate focus of the nurse.  Normal values: troponin I <0.5 ng/mL (<0.5 mcg/L); troponin T <0.1 ng/mL
(<0.1 mcg/L).

(Option 1)  The glucose is elevated (normal random glucose 70-110 mg/dL [3.9-6.1 mmol/L]) but is not the
priority in this situation.  The nurse will need to assess whether the client has a history of diabetes and time of
the last meal.
(Option 2)  Normal hematocrit for a male is 39%–50% [0.39-0.50] and 35%-47% [0.35-0.47] for a female.  The
hematocrit value is not the priority.

(Option 3)  The potassium is slightly below normal (3.5–5.0 mEq/L [3.5-5.0 mmol/L]).  This should be the
nurse's second priority.  A low potassium level can precipitate dysrhythmias.

Educational objective:
An elevated troponin value holds the highest priority for intervention when a client is experiencing chest pain. 
Positive troponin levels are indicative of myocardial injury and require immediate attention by the nurse. 
Normal values are <0.5 ng/mL (<0.5 mcg/L) for troponin I and <0.1 ng/mL (<0.1 mcg/L) for troponin T.

Fires can be extremely dangerous in health care facilities, where clients may be incapacitated.  Nurses must be
aware of the agency's fire safety plan.  Most agencies use the mnemonic RACE in their protocols to ensure that
all employees perform the priority actions consistently.  These actions are:

R - Rescue any clients in immediate danger and move them to safety (Option 5)
A - Alarm - sound the alarm and activate the agency's fire response (Option 4)
C - Confine the fire by closing all doors to all rooms and fire doors to the entrance of the unit (Option 2)
E - Extinguish the fire, if possible, with a fire extinguisher (Option 1)

Visitors may be discouraged from using the elevators after the other actions have taken place (Option 3).

Educational objective:
When a fire occurs in a health care agency, the nurse should use the mnemonic RACE to remember the priority
steps: R - Rescue any clients in immediate danger; A - Alarm, sound the fire alarm; C - Confine the fire by
closing doors; E- Extinguish the fire, if possible, with a fire extinguisher.

Invasive arterial line and manual cuff readings measure BP via 2 different methods.  The arterial line measures
flow of the blood past a catheter, and the manual cuff measures pressure based on compression of the artery. 
Because of the differences, the 2 pressures may not match.  The arterial line can be highly useful to the clinician
as it gives a continuous measurement of accurate BP.  The manual cuff will give a reading of the pressure only
at the moment the pressure is measured.  The following steps should be instituted to ensure accuracy of invasive
pressure readings:

1. Position the client supine, flat, prone, or with the head of the bed <45 degrees
2. Confirm zero reference stopcock (port of the stopcock nearest to the transducer) to be at the level of the
phlebostatic axis (4th intercostal space, midaxillary line), which approximates the level of the atria of
the heart
3. Zero the system after initial setup, with disconnection of the transducer or when accuracy of the
measurements is questioned
4. Perform a dynamic response test (square wave test) every 8-12 hours, when the system is opened to air
or when accuracy of measurements is questioned
5. Measure pressures at the end of expiration

(Option 2)  The client does not need to be flat for all pressure readings.  As long as the zero reference stopcock
is level with the phlebostatic axis, the position can be supine, flat, prone, or with the head of the bed <45
degrees.

(Option 3)  Comparison of the 2 methods of blood pressure measurement will not facilitate accurate
functioning of the arterial line.
Educational objective:
Blood pressure measurement via an invasive arterial line is a very effective clinical tool in client care
management.  For accurate measurements, the nurse should follow systematic assessment and management of
the arterial line.

The head of the bed should be elevated to a minimum of 30 degrees (semi-Fowler position) during enteral
feedings and for 30-60 minutes afterward, thereby decreasing aspiration risk.  Many institutions have policies
that require the nurse to hold the feeding if the client must be supine (eg, diagnostic tests).

Gastric residual volumes are checked every 4 hours with continuous feeding or before each intermittent feeding
and medication administration.  Continuing feedings despite a large volume residual increases the client's risk
for emesis and aspiration.  Recent evidence suggests that holding the feeding for a residual volume >100 mL is
not necessary, and some institutional policies allow a residual volume of up to >500 mL as long as the client is
asymptomatic.

Flush the tube before and after bolus feedings to keep the tube patent and avoid contamination of the stagnant
feeding solution.  Sterile fluid is used to help prevent infection in vulnerable clients.

(Option 1)  Aspirated residual volume should be returned to the stomach.  If acidic gastric juices are
repeatedly discarded (2,500 mL secreted daily), there is risk for metabolic alkalosis and hypokalemia.

(Option 5)  Gastric pH should be acidic (pH ≤5).  A pH ≥6 requires x-ray confirmation of tube placement.  In
addition, any newly inserted nasogastric tube requires x-ray confirmation of tube location.

Educational objective:
Care of a client receiving enteral feedings requires elevating the head of the bed, monitoring for acidic pH ≤5,
assessing for excessive residual volume, returning aspirated residual volume to the stomach, and flushing the
tubing before and after bolus intermittent feeding.

Assault is an act that threatens the client and causes the client to fear harm, but without the client being
touched (Option 1).

False imprisonment is the confinement of a client against the client's will or without legal justification (eg,
client is not a threat to self or others) (Option 3).

Invasion of privacy includes disclosing medical information to others without client consent.  Under the
Health Insurance Portability and Accountability Act (HIPAA), a client's information regarding medical
treatment is private and cannot be released without the client's permission (Option 5).

(Option 2)  Battery involves making physical contact with the client without permission.  This includes
harmful acts or acts that the client refuses (eg, performing a procedure).  When interacting with the client, it is
important to practice veracity, the ethical principle of being truthful.

(Option 4)  An emancipated minor is an individual under the age of legal responsibility who has been legally
freed from parental control through a court order (eg, due to enlistment in the military, marriage, pregnancy). 
The parent in this situation would not need to be called.  Clients have the right to be informed of risks and
benefits of procedures prior to care and to give informed consent.

Educational objective:
Clients have the right to privacy and to give informed consent prior to medical care.  Assault is an act that
threatens the client, causing the client to fear harm without the client being touched.  Battery is physical contact
against a client's will or without legal justification.  False imprisonment includes restraining a competent client
without the client's permission.

SLE is an autoimmune disorder in which the body's immune system produces autoantibodies that attack the
body's tissues and cells.  It is characterized by alternating periods of exacerbation (flare) and remission.  The
skin is one of the target organs commonly affected by the disease.

The characteristic cutaneous manifestation of SLE (> 50%) is a flat or raised red rash that forms a butterfly
shape across the bridge of the nose and cheeks.  It is often related to sunlight exposure (ultraviolet light) and
is more pronounced during a disease flare (Option 1).  Recurrent oral ulcers are also very common.

(Options 2, 3, and 4)  Although petechiae, pruritus, and urticaria may be associated cutaneous manifestations,
they are not characteristic specifically to SLE.

Educational objective:
The characteristic cutaneous manifestation of SLE is a flat or raised red rash that forms a butterfly shape across
the bridge of the nose and cheeks.

During the school-age years (6-12), sleep needs of a child depend on health status, activity level, and age. 
Children in this age group need approximately 11 hours of sleep daily at age 5 and 9 hours at age 12 (Option
4).  Children are often unaware of their level of fatigue.  Bedtimes should be established to prevent fatigue the
next day.  Bedtime issues are usually not a concern, although many children retain bedtime rituals such as
reading or listening to music.

(Option 1)  Quiet activity (eg, coloring, reading) prior to bedtime should be planned to promote restful sleep.

(Option 3)  Growth rate is slowed during the school-age years, which accounts for variations in sleep needs.

Educational objective:
Sleep needs of school-age children are dependent on health status, activity level, and age.  Required sleep
averages 11 hours (for 5-year-olds) to 9 hours (for 12-year-olds).  It is important to establish bedtime hours and
bedtime rituals.  These children usually do not need daytime naps if they have slept well at night.

The client with severe COPD will have a chronically low oxygen level, hypoxemia.  To compensate, the body
produces more red blood cells (RBCs) to carry needed oxygen to the cells.  A high RBC count is called
polycythemia.

(Option 1)  Anemia is not expected and will worsen symptoms of COPD.

(Option 2)  Neutropenia (low white blood cell count) is not expected in COPD.  Chemotherapy and many
medications (clozapine [antipsychotic], methimazole [antithyroid]) can cause neutropenia which increases the
risk of infection.

(Option 4)  Thrombocytopenia (low platelet count) is not anticipated in COPD.  Alcohol use, HIV infection,
and many medications (heparin) can cause thrombocytopenia.

Educational objective:
Polycythemia, an increase in RBCs, is an anticipated compensatory response to chronically low blood oxygen
levels in clients with severe COPD.
An 18-month-old should have a vocabulary of 10 or more words and be able to hold and drink from a cup. 
Both of these types of delays (communication and language development, and fine motor skills) may be more
apparent at age 18 months than at earlier ages.  Either finding should prompt the nurse that further evaluation is
needed.

(Option 1)  An 18-month-old can climb stairs with assistance, use a pull-toy, and turn the pages of a book.

(Option 2)  An 18-month-old may continue to be bottle fed at times, can hold and clumsily use a spoon, and
can creep down stairs.

(Option 3)  An 18-month-old might be able to scribble but would not be able to draw a picture; an 18-month-
old can throw a ball and point to body parts.

Educational objective:
An 18-month-old should have a vocabulary of at least 10 words and be able to use a spoon.
Otitis media (OM) is inflammation or infection of the middle ear.  The eustachian tubes in infants and young
children are short, wide, and fairly horizontal, which results in ineffective draining of respiratory secretions
and a potential for recurrent infections.  Episodes of OM often follow a respiratory tract infection, such as
influenza or respiratory syncytial virus (RSV).  OM risk also increases when fluid pools in the mouth and then
reaches the eustachian tubes (eg, drinking from a bottle while lying down).

Established family patterns that can play a role in recurrent OM should be assessed and include:

 Recurring exposure to tobacco smoke


 Regular pacifier use, particularly after age 6 months
 Drinking from a bottle while lying down
 Lack of immunizations, particularly the pneumococcal vaccine series

(Option 2)  Excess water in the ears from bathing or swimming can alter the protective environment of the
external ear and contribute to otitis externa, also known as swimmer's ear.  However, this does not contribute to
OM.  Fluid to the middle ear enters from the mouth through the eustachian tubes.

(Option 3)  Damage due to a foreign body in the ear places the client at risk for otitis externa, but it is not
associated with an increase in recurrent OM.

Educational objective:
In clients with recurrent OM, assessment should include established family patterns, such as the client's position
when drinking from a bottle, pacifier use, exposure to tobacco smoke, and lack of immunizations (particularly
the pneumococcal vaccine series).

Children age 3-6 (preschool) are in Piaget's preoperational stage of cognitive development.  At age 5,
children are not able to fully understand cause and effect and will therefore ascribe inappropriate causes to
phenomena (eg, scraped knee was caused by earlier misbehavior).
Five-year-olds are developmentally capable of understanding adoption on a basic level; however, it may be
difficult for them to understand the concept of having another family.  The child might notice that friends are
not adopted.  Preschool-age children may also believe they are responsible for being adopted and may develop
separation issues and fear abandonment.

(Option 2)  School-age children may imagine how life would be different if they were with their biological
parents.  Self-esteem issues begin to develop around this time with the possible sense of loss of the biological
family.  School-age children may be sensitive to physical differences between themselves and their adoptive
family.

(Option 3)  Children age 3 and under are unable to understand differences between adoptive and biological
parents.  The age at which to begin discussions about being adopted is debatable, but use of positive language is
always encouraged.  The terms "given up" or "put up" for adoption or someone's "real" parents are
inappropriate and should be avoided.

(Option 4)  Adolescents have abstract thinking abilities that enable introspection about their adoption.  They
typically do not like differing from their peers.  Open and honest communication is important at this age.

Educational objective:
Preschool-age children (3-6 years) are in Piaget's preoperational stage of cognitive development.  They become
increasingly verbal but are unable to understand cause and effect, often ascribing inappropriate causes to
phenomena; therefore, the adopted child may feel responsible for being adopted.

Angina pectoris is defined as chest pain brought on by myocardial ischemia (decreased blood flow to the
heart muscle).  Any factor that increases oxygen demand or decreases oxygen supply to cardiac muscle may
cause angina, including the following:

 Physical exertion (eg, exercise, sexual activity):  Increases heart rate and reduces diastole (time of
maximum blood flow to the myocardium)
 Intense emotion (eg, anxiety, fear):  Initiates the sympathetic nervous system and increases cardiac
workload
 Temperature extremes:  Usually cold exposure and hypothermia (vasoconstriction); occasionally
hyperthermia (vasodilation and blood pooling)
 Tobacco use and second-hand smoke inhalation:  Replaces oxygen with carbon monoxide; nicotine
causes vasoconstriction and catecholamine release
 Stimulants (eg, cocaine, amphetamines):  Increase heart rate and cause vasoconstriction
 Coronary artery narrowing (eg, atherosclerosis, coronary artery spasm):  Decreases blood flow to
myocardium

(Option 4)  Deep sleep doesn't increase oxygen demand.

Educational objective:
Angina pectoris is chest pain caused by myocardial ischemia.  Any factor that increases oxygen demand or
decreases oxygen supply may deprive the myocardium of necessary oxygen needed to function effectively.

Venous thrombosis involves the formation of a thrombus (clot) and the inflammation of the vein. 
Anticoagulant therapy such as heparin does not dissolve the clot.  The clot will be broken down by the body's
intrinsic fibrinolytic system over time.  The heparin slows the time it takes blood to clot, thereby keeping the
current clot from growing bigger and preventing new clots from forming.
(Option 1)  Anticoagulants do not dissolve clots.  Thrombolytic agents (fibrinolytics), such as tissue
plasminogen activator (tPA), are used to break the clots, but they also carry the risk of serious intracranial
hemorrhage and are used only for acute life-/organ-threatening conditions.  The body will break down the clot
over a period of time.

(Option 2)  Heparin does not prevent the clot from breaking off but will deter the clot from growing larger.

(Option 4)  The nurse should be able to answer client questions regarding medications being administered.  The
HCP can answer any further questions the client may have.

Educational objective:
The nurse should teach the client that the purpose of unfractionated heparin infusion in the treatment of DVT is
to slow the time it takes blood to clot, thereby keeping the current clot from getting bigger and preventing new
clots from forming.

CT scan with oral contrast is a noninvasive radiographic procedure often used to obtain cross-sectional images
of the gastrointestinal anatomy.  Iodine-containing contrast dye is administered orally to better define
gastrointestinal structures (eg, intestinal lumen, gastric features).

Contraindications to CT scan with oral contrast include iodine allergy, kidney disease, pregnancy, and any
condition that significantly impairs swallowing or airway protection (Option 4).  Some metallic implants and
pacemakers are contraindications for MRI but present no hazard to the client undergoing CT scan (Options 2
and 3).  Abdominal pain and a history of breast cancer are not contraindications for CT scan (Option 1).

Fatigue and weight gain are classic manifestations of hypothyroidism.  Features of hypothyroidism typically
result from decreased metabolic rate and include cold intolerance, constipation, dry skin, irregular or prolonged
menstrual periods, and mental slowing or difficulty concentrating.

(Options 3, 5, and 6)  Fever, tachycardia, and sweating are signs of hyperthyroidism, which is a
hypermetabolic state, with signs and symptoms that are usually the opposite of those seen in hypothyroidism. 
The presenting symptoms of a hyperthyroid client would likely include weight loss despite an increased appetite
and difficulty sleeping.

Educational objective:
Hypothyroidism is associated with symptoms of a low metabolic rate; hyperthyroidism causes symptoms of a
high metabolic rate.

Dysphagia increases the risk for aspiration of oropharyngeal secretions, gastric content, food, and/or fluid into
the lungs.  Aspiration of foreign material into the lungs increases the risk for developing aspiration pneumonia. 
Interventions to help decrease aspiration and resulting aspiration pneumonia in susceptible clients (eg, elderly,
neurologic dysfunction, decreased cough or gag reflexes, decreased immunity, chronic disease), include the
following:

 Swallowing 2 times before taking another bite of food.  This clears food from the pharynx.
 Thickening liquids to assist swallowing
 Avoiding over-the-counter cold medications.  Antihistamine cold preparation medications also have
some anticholinergic properties, such as causing drowsiness, decreasing saliva (xerostomia) production,
and making the mouth dry.  Saliva is a lubricant, and it helps bind food together to facilitate swallowing.
 Sitting upright for at least 30-40 minutes after meals.  This uses gravity to move food or fluid
through the alimentary tract, decreases gastroesophageal reflux, and helps decrease risk for aspiration.
 Brushing teeth and using antiseptic mouthwash before and after meals.  This reduces the bacterial
count before eating because bacteria as well as food can be aspirated.  After-meal use removes particles
of food that can be aspirated later.
 Smoking cessation.  Smoking decreases mucociliary clearance and increases bacterial count in the
mouth.

(Option 1)  Positioning the chin slightly downward toward the neck (chin-tuck) when swallowing can be
effective in some clients with dysphagia due to its facilitating closure of the epiglottis to help prevent tracheal
aspiration.

Educational objective:
Teaching clients who are susceptible to aspiration about swallowing techniques, positioning, avoidance of over-
the-counter cold preparation medications (cause drowsiness and dry mouth), oral care, and smoking cessation
can decrease the risk for aspiration pneumonia

Opioids, including morphine sulfate, dilate peripheral blood vessels and can cause hypotension.  The side
effect is not as noticeable when the client is lying down; however, once the client attempts to stand, it can cause
orthostatic hypotension.  It is more common in clients who have some underlying volume depletion (eg,
opioid-induced nausea/vomiting).  Due to the safety risk, clients must be taught to rise slowly from a sitting to a
standing position.  The nurse should first assist the client to sit if the client feels lightheaded in a standing
position.  Safety is the client's priority.  If orthostasis is evident, fluid bolus may be needed and should be
communicated to the health care provider.

(Options 1 and 4)  Assessing the client's orthostatic vital signs and recommending bed rest until the
lightheadedness resolves are important but not first-priority actions.

(Option 3)  Walking with the client is not recommended when the client is symptomatic on standing.

Educational objective:
Client safety is the priority action in any situation.  The nurse should assist the client to a safe position prior to
proceeding with other interventions.

Medications commonly prescribed to treat bacterial pneumonia include antibiotics, expectorants, mucolytics,
antipyretics, analgesics, and anti-inflammatories.  Furosemide (Lasix) is a diuretic and is not appropriate for
treating the fine crackles associated with pneumonia.  The crackles result from alveolar filling and
atelectasis, not from heart failure or pulmonary edema.

(Option 2)  Extended-release guaifenesin (Mucinex) is an expectorant medication that increases respiratory
fluids and thins secretions to facilitate mobilization and expectoration.  It is appropriate to prescribe in clients
who have pneumonia with a productive cough and low-pitched wheezing (rhonchi).

(Option 3)  Ibuprofen is an anti-inflammatory medication administered to relieve pleuritic chest pain associated
with pneumonia and is appropriate to prescribe in clients with pneumonia.

(Option 4)  Recommended antibiotic therapy for hospitalized clients with community-acquired streptococcal
pneumonia includes monotherapy with a fluoroquinolone or combined therapy with a macrolide (eg,
azithromycin [Zithromax]) plus a beta-lactam (eg, a cephalosporin [ceftriaxone]) for 1-2 days before
transitioning to oral antibiotics.
Educational objective:
Medications prescribed to treat hospitalized clients with community-acquired bacterial pneumonia include IV
antibiotics, expectorants, mucolytics, antipyretics, analgesics, and anti-inflammatories.

Nursing assessment of the client in the last hours to days of life should include the physical changes that
indicate where the client is in the dying process.  Some signs that the client is nearing death include the
following:

 Coolness and paleness or mottling of the extremities


 A slack, relaxed jaw and open mouth from loss of facial muscle tone
 Difficulty in maintaining body posture or positions
 Eyelids half-open
 Cheyne-Stokes or uneven respirations with periods of apnea

(Option 2)  Urine output usually decreases and darkens (concentrated) from dehydration as the client nears
death.

(Option 3)  Low blood sugar is not a typical finding in the client who is nearing death.

Educational objective:
The client who is nearing death will exhibit a series of physical changes, including alterations in breathing
patterns, impaired perfusion, and slack jaw muscles.

Nausea and vomiting are expected side effects of opioid medications (eg, morphine sulfate) when the treatment
is initiated.  However, tolerance develops quickly and persistent nausea is rare.  It is recommended that the
client take an anti-emetic with the pain medication.

(Option 1)  Nausea and vomiting are expected side effects of this pain medication and occur up to 40% of the
time.

(Option 2)  Nausea and vomiting are less likely to occur in a recumbent client, and the risk increases up to 40%
in clients who are up and walking.

(Option 3)  Taking opioid medications on an empty stomach may increase the risk of nausea.

Educational objective:
Nausea and vomiting are expected side effects when opioid pain medications are initiated.  However, tolerance
develops quickly and persistent nausea is rare.  Nausea and vomiting are decreased when the client lies still in a
flat position.  Anti-emetics may be needed initially.

The nasal cannula is the most appropriate oxygen delivery device to apply at this time because it is
comfortable, used for the short term, inexpensive, and permits the client to eat and drink fluids.  It can
supply adequate oxygen concentrations of up to 44%.  This client is most likely hypoventilating as a result of
the opioid medication.  The client is alert and oriented and able to follow directions.  Because pain relief is
effective according to the pain scale, the client should be able to breathe deeply through the nose, and the
hypoxemia should reverse rapidly.

(Option 2)  The non-rebreather mask is used in emergencies, delivers high concentrations of oxygen (up to
90%–95%), requires a tight face seal, and is restrictive and uncomfortable.
(Option 3)  The simple face mask delivers a higher concentration of oxygen (40%–60%), is more
uncomfortable and restrictive, must be removed to eat or drink, and is not appropriate at this time.  It can be
used if hypoxemia does not resolve.

(Option 4)  The Venturi mask is a more expensive device used to deliver a guaranteed oxygen concentration to
clients with unstable chronic obstructive pulmonary disease.  These clients cannot tolerate changes in oxygen
concentration.

Educational objective:
The nasal cannula is an inexpensive, comfortable, low-flow oxygen delivery device capable of delivering
oxygen concentrations of up to 44%.  It can be used in the short term in responsive postoperative clients to treat
hypoventilation and reverse hypoxemia effectively.

In cystic fibrosis (CF), a defective protein responsible for transporting sodium and chloride causes the
secretions from the exocrine glands to be thicker and stickier than normal.  The sticky respiratory secretions
lead to the inability to clear the airway and a chronic cough.  The client eventually develops chronic lung
disease (bronchiectasis) and is at risk for recurrent lung infections.  These clients are also at risk for rupture of
the damaged alveoli, which results in sudden-onset pneumothorax.  Findings of pneumothorax include sudden
worsening of dyspnea, tachypnea, tachycardia, and a drop in oxygen saturation.  Because many of these
findings can be seen with lung infection, a sudden drop in oxygen saturation could be the only early clue.

The client with CF will often have a decreased pulse oximetry (reflects oxygen saturation in the blood) reading
due to the chronicity of the disease process and damage to the lungs; however, a reading of 90% requires urgent
intervention.

(Option 2)  Clients with CF often cough up blood-streaked sputum (hemoptysis) as a result of damage to blood
vessels in the airway walls secondary to infections.  However, this usually resolves with treatment of the
infection.  Frank hemoptysis needs urgent assessment.

(Option 3)  Maintaining weight is a challenge in those with CF due to the malabsorption of carbohydrates, fats,
and proteins caused by the impaired enzyme secretions in the gastrointestinal tract.  In addition, weight and
appetite loss may indicate an undiagnosed underlying lung infection.  This will need to be addressed, but
oxygenation is the priority.

(Option 4)  Fecal retention and impaction are common in CF due to decreased water and salt secretion into the
intestines.  This will need to be addressed, but oxygenation is the priority.

Educational objective:
When addressing the multiple needs of a client with cystic fibrosis, airway and oxygen saturation are the
priorities.  Pneumothorax can be a complication of cystic fibrosis.

Amyotrophic lateral sclerosis (ALS) is characterized by the progressive loss of motor neurons in the
brainstem and spinal cord.  Clients have spasticity, muscle weakness, and atrophy.  Neurons involved in
swallowing and respiratory function are eventually impaired, leading to aspiration, respiratory failure, and
death.  Care of clients with ALS focuses on maintaining respiratory function, adequate nutrition, and quality of
life.  There is no cure, and death usually occurs within 5 years of diagnosis.

The client with ALS and worsening ability to speak (dysarthria) may also have dysphagia and respiratory
distress; this client should be seen first (Option 1).

(Option 2)  The client with chronic obstructive pulmonary disease and peripheral edema may have cor
pulmonale, or right-sided heart failure, from vasoconstriction of the pulmonary vessels.  Cor pulmonale is
treated with long-term, low-flow oxygen; bronchodilators; and diuretics.  This client should be seen second. 
Right-sided heart failure (peripheral edema) is not as dangerous as left-sided heart failure (pulmonary edema).

(Option 3)  Fever often occurs with strep throat and may persist for ≥24 hours after initiation of antibiotics. 
This client should be seen last and should receive an antipyretic.

(Option 4)  Wavelike flank pain is characteristic of urolithiasis (urinary stones).  This client needs pain
medication and, possibly, further treatment (eg, lithotripsy) and should be seen third.

Educational objective:
Amyotrophic lateral sclerosis causes progressive loss of motor neurons, resulting in muscle weakness and
spasticity.  Muscles involved in respiration and swallowing are affected, leading to aspiration and, ultimately,
respiratory failure.  Treatment focuses on maintaining respiratory function, adequate nutrition, and quality of
life.

Mitral valve regurgitation is the result of a disrupted papillary muscle(s) or ruptured chordae tendineae,
allowing a backflow of blood from the left ventricle through the mitral valve into the left atrium.  This
backflow can lead to dilation of the left atrium, reduced cardiac output, and pulmonary edema.  Clients are often
asymptomatic but are instructed to report any new symptoms indicative of heart failure (eg, dyspnea,
orthopnea, weight gain, cough, fatigue).  This client should be assessed first due to possible heart failure,
which would require immediate intervention.

(Option 1)  Kidney transplant recipients are on an immunosuppressant regimen to prevent rejection of the
transplanted organ, which can leave them susceptible to infections such as candidiasis (thrush) of the oral
cavity.

(Option 3)  The client with a spider bite is displaying signs and symptoms of infection, and further assessment
is required to evaluate for conditions such as cellulitis.  This client should be called second.

(Option 4)  Clients with hypertension who develop sinus or nasal congestion have limited options for symptom
relief.  Decongestants containing a vasoconstrictor (eg, pseudoephedrine) can exacerbate hypertension.

Educational objective:
Chronic mitral valve regurgitation is often asymptomatic, but many clients eventually develop heart failure;
therefore, early recognition of symptoms is a priority.  Mitral regurgitation causes a backflow of blood from the
left ventricle to the left atrium, resulting in pulmonary edema (eg, dyspnea, orthopnea) and decreased cardiac
output (eg, fatigue).  Left atrial enlargement can also result in atrial fibrillation (eg, palpitations).

Compartment syndrome, a serious postoperative complication, is caused by decreased blood flow to the tissue
distal to the injury.  It results from either decreased compartment size (restrictive dressings, splints, or casts) or
increased pressure within the compartment (bleeding, inflammation, and edema).  Earliest symptoms may
include pain or numbness that is unrelieved by medication.  Subsequent findings include diminished/absent
pulses, pallor, coolness, swelling, decreased movement, and cyanosis.  Failure to treat this condition can lead
to loss of limb function, paralysis, and tissue necrosis.  The nurse should assess neurovascular status and report
to the health care provider immediately (Options 2 and 4).  Removal of tight bandages/casts and fasciotomy
(surgery) are required to relieve the pressure.

(Option 1)  Heat should not be applied to a client experiencing altered sensation, as it may burn the client. 
Active range of motion will not resolve compartment syndrome and delays needed care.

(Option 3)  Elevating the arm on pillows and providing additional analgesia may help reduce symptoms but
may also reduce perfusion of the extremity.  Instead, the extremity should be positioned at the level of the heart.
(Option 5)  Documenting findings is important.  However, reassurance and reassessment 1 hour later without
immediate intervention delays needed care.

Educational objective:
Compartment syndrome is caused by decreased blood flow to the tissue distal to the injury and can cause
ischemic necrosis.  Acute compartment

Parkinson's disease (PD) is a chronic, progressive neurodegenerative disorder that involves degeneration of
the dopamine-producing neurons.  Damage to dopamine neurons makes it difficult to control muscles through
smooth movement.  PD is characterized by a delay in initiation of movement (bradykinesia), increased muscle
tone (rigidity), resting tremor, and shuffling gait.

The most helpful response by the nurse is the one that acknowledges the concern of the client and also asks for
more information.  The nurse should assess for additional information and perform a more focused physical
assessment given this new information (Option 4).

(Option 1)  It is incorrect to say that the client is too young to have PD although it is usually seen after age 60;
about 15% of PD cases are diagnosed before age 50.

(Option 2)  Although the nurse should encourage the client and family to discuss concerns with the health care
provider, this is not the most helpful response.

(Option 3)  Although the typical parkinsonian tremor occurs at rest and not during purposeful movement, it is
not helpful to dismiss a concern without probing for more information.

Educational objective:
Therapeutic communication includes acknowledging concerns and probing for additional information as part of
an assessment.

Allopurinol is a medication frequently used in the prevention of gout.  Gout is a buildup of uric acid deposited
in the joints that causes pain and inflammation.  The medication helps to prevent uric acid deposits in the joints
and the formation of uric acid kidney stones.  Any rash in a client taking allopurinol, even if mild, should be
reported immediately to the HCP.  The nurse should direct the client to stop taking the medication
immediately, schedule an appointment, and notify the HCP.  A rash caused by allopurinol may be followed
by more severe hypersensitivity reactions that can be fatal, including Stevens-Johnson syndrome and toxic
epidermal necrolysis.

(Option 1)  Allopurinol can take several months to become effective.  Its primary use is to prevent gout
attacks;  it is not effective in treating acute attacks.  The client will need to continue to take anti-inflammatory
drugs (eg, nonsteroidal anti-inflammatory drugs or colchicine) for acute attacks.

(Option 2)  Clients are directed to take allopurinol with a full glass of water and to increase daily fluid intake to
prevent kidney stones.  This will cause an increase in urination and is an expected outcome.

(Option 4)  Nausea can be prevented by instructing the client to take the medication with food or following a
meal.

Educational objective:
The nurse should direct the client taking allopurinol for gout to immediately discontinue the medication and
report to the HCP if any rash develops.  Allopurinol-induced rashes can develop into severe and sometimes fatal
hypersensitivity reactions, such as Stevens-Johnson syndrome.  Similar instructions should be given to clients
taking anticonvulsants (eg, carbamazepine, phenytoin, lamotrigine) and sulfa antibiotics.
Coup-contrecoup injury occurs when a body in motion stops suddenly (eg, head hits car windshield), causing
contusions (bruising) of brain tissue as the brain moves back and forth within the skull.  First, the soft tissue
strikes the hard skull in the same direction as the momentum (coup).  As the body bounces back, the brain
strikes the opposing side of the skull (contrecoup).

When the forward collision occurred, the frontal lobe most likely suffered the primary impact (coup). 
Executive function, memory, speech (Broca area), and voluntary movement are controlled by the frontal lobe. 
The contrecoup most likely injured the occipital lobe, where vision is processed.

(Option 2)  The temporal lobe (lateral aspect of the brain) controls hearing and integrates sensory data (eg,
auditory, visual, somatic).  The Wernicke speech area in the temporal lobe is responsible for language
comprehension.  Light touch is processed by the sensory cortex in the parietal lobe.

(Option 3)  An interruption of sensory function indicates injury to either the spinal column or the parietal lobe. 
These injuries affect proprioception (awareness of body positioning) and graphesthesia (ability to identify
writing on the skin, by touch).

(Option 4)  The Weber test screens for conductive hearing loss by checking whether a tuning fork held along
the midline of the head is heard evenly in both ears.  Cranial nerve I is the olfactory nerve.  Hearing and smell
are both processed by the temporal lobe.

Educational objective:
Coup-contrecoup injuries usually affect the frontal and occipital lobes.  The frontal lobe controls executive
function, memory, speech, and motor skills.  The occipital lobe processes vision.

Abdominal aortic aneurysms are surgically repaired when they measure about 6 cm or are causing symptoms. 
Repair can be done via femoral percutaneous placement of a stent graft (endovascular aneurysm repair) or via
an open surgical incision of the aneurysm with synthetic graft placement.  The client must be monitored
postoperatively for graft leakage and hemodynamic stability.  Adequate blood pressure is necessary to
maintain graft patency, and prolonged hypotension can lead to the formation of graft thrombosis.  Signs of
graft leakage include a decreasing blood pressure and increasing pulse rate.

(Option 1)  Following surgery, the client will experience abdominal tenderness.  The abdomen should remain
soft and nondistended.  A rigid, distended abdomen would indicate possible blood (graft leakage) in the cavity.

(Option 3)  Pain is an expected finding following abdominal surgery.  However, increasing pain that is not
relieved by medication can indicate possible graft leakage and should be investigated.

(Option 4)  During abdominal surgeries, it is customary to insert a nasogastric tube that is left in place during
the immediate postoperative period.  Green bile-colored drainage would be expected.  Bloody drainage would
cause concern.

Educational objective:
Following repair of an abdominal aortic aneurysm, hemodynamic stability is a priority.  Prolonged hypotension
can lead to graft thrombosis.  A falling blood pressure and rising pulse rate can also signify graft leakage.

VF is characterized on the ECG by irregular waveforms of varying shapes and amplitudes.  This represents
the firing of multiple ectopic foci originating in the ventricle.  Mechanically, the ventricle is quivering with no
effective contraction or cardiac output.  VF is considered a lethal dysrhythmia.  It results in an unresponsive,
pulseless, apneic state.  If not treated rapidly, the client will not recover.
VF commonly occurs in acute myocardial infarction and myocardial ischemia and in chronic heart diseases
such as heart failure and cardiac myopathy.  It may occur in cardiac pacing or catheterization procedures due to
catheter stimulation of the ventricle.

Treatment consists of rapid initiation of CPR, defibrillation, and the use of drug therapy (eg, epinephrine,
vasopressin, amiodarone).

(Option 1)  Asystole is the total absence of ventricular electrical activity.

(Option 2)  Atrial fibrillation is characterized by total disorganization of atrial, not ventricular, activity.  QRS
complexes are usually normal in morphology.  P waves are not seen.

(Option 4)  Ventricular tachycardia has a ventricular rate of 150-250/min and originates from foci firing
repeatedly in the ventricle.

Educational objective:
The nurse should recognize VF, a potentially lethal dysrhythmia.  The ECG shows irregular waveforms of
varying shapes and amplitudes.  The client is unresponsive, pulseless, and apneic.  Rapid treatment should
include CPR, defibrillation, and drug therapy (eg, epinephrine, vasopressin, amiodarone).

Children of all ages experience loss at some point in their lives – pets, grandparents, and, in some cases, their
own parents.  In providing guidance to a parent about helping a child cope with or anticipate a parent's or loved
one's death, it is essential for the nurse to have a sense of how children at different ages perceive illness and
death.  In addition to the cognitive, emotional, and developmental age of the child, the parent's or caregiver's
response, attitude toward death, and interaction with the child will also influence the child's reaction.

(Option 1)  This applies to a child age 3-5.

(Option 2)  By age 10-12, a child will be aware that death affects everyone.

(Option 3)  An adolescent has the cognitive ability to think about the spiritual and religious aspects of death.

Educational objective:
A child age 3-5 believes death is reversible.  A child age 6-9 understands that death is permanent but has
difficulty in perceiving one's own death.  By age 10-12, a child will be aware that death affects everyone.

Psoriasis is a chronic autoimmune disease that causes a rapid turnover of epidermal cells.  Characteristic
silver plaques on reddened skin may be found bilaterally on the elbows, knees, scalp, lower back, and/or
buttocks.  The goal of therapy is to slow epidermal turnover, heal lesions, and control exacerbations.

There is no cure for psoriasis; disease management includes avoidance of triggers (eg, stress, trauma,
infection), topical therapy (eg, corticosteroids, moisturizers), phototherapy (eg, ultraviolet light), and
systemic medications, including cytotoxic (eg, methotrexate) and biologic (eg, infliximab) agents (Options 3
and 4).  The client should avoid alcohol as it can worsen psoriasis (Option 2).  In addition, the liver, kidneys,
and bone marrow are specifically affected by the systemic medications commonly used to control psoriasis.

(Option 1)  Exposure to ultraviolet light (eg, phototherapy, sunlight) can help slow epidermal turnover and
decrease exacerbations; however, there is a greater long-term risk of skin cancer.  Therefore, frequent skin
examinations by a health care provider are important.

Educational objective:
Psoriasis is a chronic autoimmune condition characterized by exacerbations of silver plaques on reddened skin. 
Although there is no cure, management includes topical and systemic medications, phototherapy, and avoidance
of triggers.

alcoholic cirrhosis, as this condition is associated with gastritis, clotting abnormalities (eg, thrombocytopenia,
coagulation disorders), and esophageal varices that increase the risk for hemorrhage (coffee ground emesis
from oxidized blood).  Hypotension and tachycardia in the presence of blood loss can indicate hypovolemia. 
The nurse should monitor for signs of hemodynamic instability (eg, hypotension, decreased urine output,
peripheral vasoconstriction, pallor) and notify the health care provider of any significant changes from baseline
as immediate esophagogastroduodenoscopy is necessary to determine the bleeding site.  Treatment to stop the
bleeding (eg, heat probe, sclerotherapy) may be indicated.

(Option 1)  A paralytic ileus is a non-mechanical intestinal obstruction that can occur following abdominal
surgery.  Expected manifestations include absent or hypoactive bowel sounds due to the lack of bowel motility
and peristalsis, and abdominal distension and nausea due to the accumulation of gas and fluids in the bowel.

(Option 3)  Bacterial peritonitis (peritoneal inflammation) involves the entry of bacteria into the peritoneal
cavity and is associated with a ruptured appendix.  Elevated temperature would be an expected finding.

(Option 4)  Sore throat discomfort from irritation of the oropharynx is expected in a client with a nasogastric
tube.
Educational objective:
Clients with alcoholic cirrhosis are at increased risk for hemorrhage due to esophageal varices and coagulation
disorders.  Hypotension and tachycardia in the presence of blood loss can indicate hypovolemia and require
immediate assessment.

Green amniotic fluid indicates that the fetus has passed its first stool (meconium) in utero.  Meconium-stained
amniotic fluid places the newborn at risk for meconium aspiration syndrome, a type of aspiration pneumonia. 
A skilled neonatal resuscitation team should be present at the birth of any newborn with meconium-stained fluid
for immediate evaluation and stabilization (Option 4).  Previously, endotracheal (ET) suctioning was
recommended for nonvigorous newborns (eg, depressed respirations, decreased muscle tone, heart rate
<100/min) born with meconium-stained fluid; however, recent guidelines indicate that routine ET suctioning is
no longer necessary.

(Option 1)  Neonates born to mothers with gestational diabetes are at risk for hypoglycemia after birth and
should be monitored closely during the first 6 hours of life.  The risk of newborn hypoglycemia is lower if the
mother's diabetes is well-controlled and not insulin-dependent.

(Option 2)  Clients with severe preeclampsia may need magnesium sulfate therapy for seizure prevention. 
Maternal magnesium therapy can cause newborn respiratory depression at birth.  However, this client's mild
preeclampsia does not require magnesium therapy.

(Option 3)  Premature rupture of membranes (PROM) refers to the rupture of membranes prior to the onset of
labor at term gestation (≥37wk 0d).  PROM on its own does not harm the fetus.  However, if labor does not
begin after PROM, induction of labor may be necessary to decrease the risk for infection (eg, chorioamnionitis).

Educational objective:
Meconium-stained amniotic fluid places the newborn at risk for meconium aspiration syndrome.  A skilled
neonatal resuscitation team should be present at birth for immediate newborn evaluation and stabilization.

Saturating a peripad in 1-2 hours could indicate hemorrhage, a life-threatening condition.  The nurse should
assess the client's fundus and, if it is boggy, massage it.  The nurse should also assess the client's vital signs and
should never leave the client alone.

(Option 1)  The nurse can delegate changing the peripad to the UAP; however, it is not the priority at this time.

(Option 3)  Determining the client's hemoglobin and hematocrit levels will help determine the amount of blood
that has been lost, but it is not the priority for this client.

(Option 4)  The nurse cannot delegate changing IV fluid rates to the UAP as this is beyond the scope of
practice.

Educational objective:
Postpartum hemorrhage is a potentially life-threatening condition that should be addressed immediately.  The
nurse should first assess the fundus and massage it if boggy.

Approximately 20% of clients with partial gastrectomy experience dumping syndrome, which occurs when
gastric contents are emptied too rapidly into the duodenum, causing a fluid shift into the small intestine.  This
fluid shift results in hypotension, abdominal pain, diarrhea, nausea, vomiting, dizziness, generalized sweating,
and tachycardia.  The symptoms usually diminish over time.

Dietary recommendations aimed at delaying gastric emptying include the following:


 Consume small, frequent meals to reduce the amount of food in the stomach at any one time (Option
5).
 Eat slowly in a relaxed environment.
 Avoid meals high in simple carbohydrates (eg, sugar, syrup) as these may trigger dumping syndrome
when carbohydrates are broken down into simple sugars (Option 3).
 Consume meals high in protein, fat, and fiber, which take longer to digest and remain in the stomach
longer than carbohydrates (Options 1 and 4).
 Separate fluids from meals.  If fluids are taken with meals, stomach contents pass more easily into the
jejunum and worsen symptoms.  Fluid intake should occur only after or between meals, separated from
solid intake by at least 30 minutes (Option 2).
 Avoid sitting up after a meal.  Gravity increases gastric emptying.  Lying down after meals slows
down the gastric emptying and is preferred.

Educational objective:
Dumping syndrome is a complication of gastrectomy.  Measures that delay gastric emptying and reduce the risk
of dumping syndrome include consuming meals low in carbohydrates and high in fiber, proteins, and fats;
avoiding fluids during meals; and eating small, frequent meals.

A sentinel event is any unanticipated event in a health care setting that results in death or serious physical or
psychological injury.

Warfarin is an anticoagulant often used in clients with the following:

 Atrial fibrillation (to prevent clot formation and reduce the risk for stroke)
 Deep venous thrombosis and pulmonary embolism (to prevent additional clots)
 Mechanical heart valves (to prevent clot formation on valves)

The International Normalized Ratio (INR) is a blood test used to monitor the effectiveness of warfarin therapy. 
The typical target INR is 2-3.  In some instances (eg, mechanical heart valves), the therapeutic INR target is as
high as 3.5.  The higher the INR, the higher the bleeding risk.  The nurse should not administer warfarin if the
INR is over 4.

(Option 1)  Flumazenil is the appropriate antidote for a benzodiazepine overdose.

(Option 2)  Insulin quickly lowers serum potassium by pushing it intracellularly.  Dextrose is given to prevent
hypoglycemia.  This is an appropriate action.

(Option 4)  Nitroprusside is a potent vasodilator often used for hypertensive urgencies.

Educational objective:
The target International Normalized Ratio (INR) for most conditions in which warfarin is used is normally 2-3
and is occasionally 3.5.  The risk of bleeding increases as the INR rises.
Renal perfusion status is monitored closely in a client who has had abdominal aneurysm repair.  Hypotension,
dehydration, prolonged aortic clamping during surgery, blood loss, or embolization can lead to decreased renal
perfusion and potential kidney injury.  The nurse should routinely monitor the client's blood urea nitrogen
(BUN) and creatinine levels as well as urine output.  Urine output should be at least 30 mL/hr.  This client
should have an output of at least 120 mL of urine in a 4-hour period.

(Option 1)  Diminished breath sounds in the lung bases are a common occurrence after surgery, especially in a
client who has an abdominal incision that is painful with deep inspiration.  The nurse should medicate the client
for pain and encourage coughing, deep breathing, and use of an incentive spirometer.

(Option 2)  Hypoactive bowel sounds are typical after abdominal surgery as the bowel has been handled and
manipulated.  Clients often have a nasogastric tube for suction until bowel sounds return.  The nurse should
continue to monitor for bowel sounds and the presence of flatus.

(Option 4)  A decreased or absent pulse, together with cool, pale, or mottled extremities, would be cause for
concern.  This client has warm extremities.  Pulses should be compared with preoperative status and can be
verified with a Doppler if needed.

Educational objective:
The nurse should carefully monitor renal status in a client who has had abdominal aortic aneurysm repair. 
BUN, creatinine, and urine output should be assessed.  Urine output of at least 30 mL/hr is expected.

age <12).  There is no known age at which the diet can be discontinued safely, and lifetime dietary restrictions
are recommended for optimal health (Option 3).

Management of the client with PKU includes:

 Monitoring serum levels of phenylalanine


 Including synthetic proteins and special formulas (eg, Lofenalac, Phenyl-Free) in the diet (Option 4)
 Eliminating high-phenylalanine foods (eg, meats, eggs, milk) from the diet (Option 2)
 Encouraging the consumption of natural foods low in phenylalanine (most fruits and vegetables)
(Option 5)  Restriction of dietary tyrosine is not necessary.  Tyrosine levels in clients with PKU may be normal
or slightly decreased.

Educational objective:
Phenylketonuria requires lifetime dietary restrictions.  Infants should be given special formulas (eg, Lofenalac). 
For children and adults, high-phenylalanine foods (eg, meats, eggs, milk) should be restricted and replaced with
protein substitutes.

The child with a recent tonsillectomy is at highest safety risk.  Postoperative hemorrhage from tonsillectomy
is uncommon but may occur up to 14 days after surgery.  During the healing process, white scabs will form at
the surgical sites.  Sloughing then occurs approximately 7 days after the procedure, increasing the risk for
bleeding.  Caregivers should be taught to observe for signs of bleeding (eg, frequent swallowing or throat
clearing).  The child may also experience increased pain.  The nurse should instruct this parent that the child
should not resume strenuous activity or contact sports for at least 7-14 days post surgery.

(Option 1)  Tympanostomy tubes or grommets are pressure-equalizing tubes placed in the tympanic membrane
to facilitate drainage of middle ear fluid (eg, for eustachian tube dysfunction or recurrent otitis media with
effusion).  One of this child's tubes has most likely fallen out of the eardrum.  No immediate intervention is
required; however, the health care provider should be notified.

(Option 2)  Clients often report ear pain (otalgia) following adenotonsillectomy due to irritation of the 9th
cranial nerve (glossopharyngeal) in the throat, causing referred pain to the ears.  This is a normal, expected
finding.

(Option 3)  The contagious period for strep throat starts at the onset of symptoms and lasts through the first 24
hours of beginning antibiotic treatment.  This client is able to return to activities and does not require an
immediate call back.

Educational objective:
The risk of post-tonsillectomy hemorrhage persists for up to 14 days after surgery, and resuming strenuous
activity too early increases this risk.  The potential for bleeding is higher 7-10 days postoperatively while
sloughing occurs.

The client who just underwent a bronchoscopy is at risk for becoming unstable.  During a bronchoscopy, the
larynx, trachea, and bronchi are visualized using an endoscope while the client is under sedation.  Respiratory
status, airway patency, vital signs, and sedation level should be assessed immediately upon return from the
procedure and at regular intervals until the client becomes stable.  The client must be kept NPO until alert with
a positive gag reflex.

(Option 1)  Clients with a positive tuberculin skin test and hemoptysis may have tuberculosis.  They will need
a chest x-ray and sputum culture to confirm the diagnosis and may be initiated on an antitubercular medication
regimen.  However, this client does not require immediate attention.

(Option 2)  The nurse should address any factors that may cause a client to refuse incentive spirometry (eg,
pain).  This client may require education on the importance of incentive spirometry (eg, prevention of
atelectasis and pneumonia) or administration of PRN analgesics to encourage participation in deep-breathing
exercises.  However, this client does not require immediate attention.

(Option 4)  Air bubbles in the suction control chamber of a chest tube drainage system would be a normal
finding.  Air bubbles in the water seal chamber may indicate a leak and would require immediate intervention.
Educational objective:
Clients returning from procedures performed under sedation should receive priority assessment of vital signs
and respiratory status.

Propranolol is a nonselective beta-blocker that inhibits beta1 (heart) and beta2 (bronchial) receptors.  It is used
for many indications (eg, essential tremor) in addition to blood pressure control.  Blood pressure decreases
secondary to a decrease in heart rate.  Bronchoconstriction may occur due to the effect on the beta2 receptors. 
The presence of wheezing in a client taking propranolol may indicate that bronchoconstriction or
bronchospasm is occurring.  The nurse should assess for any history of asthma or respiratory problems with
this client and notify the health care provider (HCP).

(Option 1)  A headache is a common occurrence with hypertension.  The nurse may administer an analgesic as
needed.

(Option 2)  This is the first dose of propranolol that the client has received.  It may take several days of
treatment for the blood pressure to reduce to a more normal reading.

(Option 3)  A reduction in heart rate is expected with a beta-blocker.  The nurse should continue to monitor it
for further reduction.

Educational objective:
The nurse should be concerned about the presence of wheezing in a client taking a nonselective beta-blocker
like propranolol.  Wheezing may indicate bronchoconstriction or bronchospasm.  The nurse should assess for
any history of asthma or other respiratory problems and report to the HCP.

The nurse would expect to hear a murmur with an atrial septal defect.  This defect is an abnormal opening
between the right and left atria, allowing blood from the higher pressure left atrium to flow into the lower
pressure right atrium.  The back-and-forth flow of blood between the 2 chambers causes a vibration that is heard
as a murmur on auscultation.  ASD has a characteristic systolic murmur with a fixed split second heart
sound.  Some clients may also have a diastolic murmur.

(Option 1)  Muffled heart tones are not typical in ASD.  Muffled heart tones that are heard postsurgical
intervention are concerning for cardiac tamponade.

(Option 3)  Atrial and ventricular septal defects are acyanotic congenital heart defects because the blood from
the high pressure left side (oxygenated blood) goes to the low pressure right side.

(Option 4)  Weak lower and strong upper extremity pulses are present in coarctation of the aorta.

Educational objective:
In a child with atrial septal defect, the nurse would expect to hear a heart murmur on auscultation of heart
sounds.

Theophylline has narrow therapeutic index and plasma concentrations >20 mcg/mL (111 µmol/L) are
associated with theophylline drug toxicity.  Toxicity can be acute or chronic.  Conditions associated with
chronic toxicity include advanced age (>60), drug interactions (eg, alcohol, macrolide and quinolone
antibiotics), and liver disease.  Acute toxicity is associated with intentional or accidental overdose.

Symptoms of toxicity usually manifest as central nervous system stimulation (eg, headache, insomnia,
seizures), gastrointestinal disturbances (eg, nausea, vomiting), and cardiac toxicity (eg, arrhythmia).
(Option 1)  Alteration in color perception and visual changes are commonly seen with digoxin toxicity.

(Option 2)  Gum hypertrophy is seen with phenytoin toxicity.

(Option 3)  Hyperthermia and tinnitus are often seen with aspirin overdose.

Educational objective:
Theophylline plasma concentrations >20 mcg/mL (111 µmol/L) are associated with theophylline drug toxicity. 
Seizures (central nervous system stimulation) and cardiac arrhythmias are the most serious and lethal
consequences.

Medications commonly prescribed for a client with an open fracture include:

 Cefazolin (Ancef), a bone-penetrating cephalosporin antibiotic that is active against skin flora
(Staphylococcus aureus); it is given prophylactically before and after surgery to prevent infection
(Option 1)

 Cyclobenzaprine (Flexeril), a central and peripheral muscle relaxant given to treat pain associated with
muscle spasm; carisoprodol (Soma) or methocarbamol (Robaxin) can also be prescribed

 Tetanus and diphtheria toxoid, an immunization given prophylactically to prevent infection


(Clostridium tetani) if immunizations are not up to date (>10 years), unavailable, or unknown (Option
4)

 Ketorolac (Toradol), a nonsteroidal anti-inflammatory drug given to decrease inflammation and pain

 Opioids (eg, morphine, hydrocodone [Vicodin]), given for analgesia (Option 3)

Educational objective:
Medications commonly prescribed for a client with an open fracture to prevent infection and treat pain and
muscle spasm include cefazolin (Ancef), tetanus toxoid, ketorolac (Toradol), opioids, and cyclobenzaprine
(Flexeril).

Rheumatoid arthritis (RA) is a chronic, systemic, inflammatory, autoimmune condition of unknown origin
that has periods of exacerbation and remission.  The body's immune system attacks the lining of the joints,
leading to bone erosion and joint deformity.  Although there is no cure for the disease, early diagnosis and
appropriate treatment can help limit localized joint destruction and systemic organ damage.

Characteristic features of RA include the following:

 Symmetrical pain and swelling that initially affects the small joints of the hands and feet
 Morning joint stiffness that lasts from 60 minutes to several hours
 Elevated ESR and rheumatoid factor levels

(Option 1)  Asymmetrical pain in the weight bearing joints is characteristic of osteoarthritis.  Crepitus,
especially over the knee joints, is also present in osteoarthritis.

(Option 2)  Low back pain and stiffness, worse in the morning and improving as the day progresses, is
characteristic of ankylosing spondylitis.  RA typically does not involve the spine, except the cervical spine.
(Option 3)  Pain, swelling, and redness of one or more extremity joints (typically the great toe) are
characteristic of acute gout attack.

Educational objective:
RA is a chronic, systemic, inflammatory, autoimmune disease of unknown origin.  Early localized articular
symptoms include bilateral, symmetrical pain and swelling that initially affects the small joints of the wrists,
hands, and feet and morning joint stiffness that lasts at least an hour.

Aortic stenosis is the narrowing of the orifice between the left ventricle and aorta.  Many clients with aortic
stenosis are asymptomatic.  Symptoms usually develop with exertion as the left ventricle cannot pump enough
blood to meet the body's demands due to aortic obstruction (stenosis).  These include dyspnea, angina, and, in
severe cases, syncope (reduced blood flow to the brain).  Clients usually do not experience symptoms at rest.

(Options 2 and 4)  This client already developed syncope and angina (exertional chest pain) and is at high risk
for sudden death with exertion.

(Option 3)  The client should restrict activity.  The incidence of sudden death is high in this population, and it is
therefore prudent to decrease the strain on the heart while awaiting surgery.

Educational objective:
Clients with severe aortic stenosis are at risk for developing syncope and sudden death with exertion.  The left
ventricle cannot push enough blood into the aorta to meet the body's demands due to the valve stenosis.

Hypothyroidism is a thyroid disorder characterized by thyroid hormone deficit (low T3 and T4).  TSH is
elevated due to compensatory increase from pituitary.  Hypothyroidism affects almost every body system and is
predominately associated with a slow metabolic rate.  Some common manifestations include the following:

1. Decreased gut motility leading to constipation


2. Cool and pale skin due to decreased blood flow; hyperkeratosis results in dry and rough skin
3. Brittle nails and hair; hair loss due to poor blood supply
4. Bradycardia from low metabolic state
5. Joint pains and muscle aches are common
6. Clients can develop dementia and depression due to mental slowing
7. Cold intolerance characteristic
8. Modest weight gain

(Options 5 and 6)  Weight loss; heat intolerance; shakiness; diarrhea; and warm, moist skin are symptoms
associated with hyperthyroidism or an increased metabolic rate.

Educational objective:
Signs and symptoms of hypothyroidism (a thyroid hormone deficit) are associated with a low metabolic rate. 
Weight gain, constipation, dry skin, hair loss, cold intolerance, bradycardia, mental slowing (dementia and
depression), and anemia are some of the most common manifestations.

Raynaud phenomenon is a vasospastic disorder resulting in an episodic vascular response related to cold
temperatures or emotional stress.  It most commonly affects women age 15-40.  Vasospasms induce a
characteristic color change in the appendages (eg, fingers, toes, ears, nose).  When vasoconstriction occurs, the
affected appendage initially turns white from decreased perfusion, followed by a bluish-purple appearance due
to cyanosis.  Clients usually report numbness and coldness during this stage.  When blood flow is subsequently
restored, the affected area becomes reddened and clients experience throbbing or aching pain, swelling, and
tingling.  Acute vasospasms are treated by immersing the hands in warm water.
Client teaching regarding prevention of vasospasms includes:

 Wear gloves when handling cold objects (Option 5).


 Dress in warm layers, particularly in cold weather.
 Avoid extremes and abrupt changes in temperature.
 Avoid vasoconstricting drugs (eg, cocaine, amphetamines, ergotamine, pseudoephedrine).
 Avoid excessive caffeine intake (Option 1).
 Refrain from use of tobacco products (Option 4).
 Implement stress management strategies (eg, yoga, tai chi) (Option 3).

If conservative management is unsuccessful, clients may be prescribed calcium channel blockers to relax
arteriole smooth muscle and prevent recurrent episodes.

(Option 2)  Cold water will cause vasoconstriction and worsen the condition.

Educational objective:
Raynaud phenomenon is a vasospastic disorder triggered by exposure to cold or stress.  Key elements of client
teaching include management of acute attacks, avoidance of vasoconstrictive substances (eg, tobacco, cocaine,
caffeine), stress reduction, and appropriate clothing (eg, gloves, warm layers).

Endotracheal suctioning is performed to maintain a patent airway if a client cannot mobilize secretions
independently.  Inserting a catheter into the airway compromises the sterility of the lower airway and increases
the risk for infection.  Suctioning removes oxygen in addition to secretions, placing the client at risk for
hypoxemia.  High suction levels or the contact of the catheter with the trachea can cause trauma, such as
barotrauma, damage to tracheal mucosa, and microatelectasis.  In order to decrease the occurrence of these
complications:

 Use strict sterile technique throughout suctioning process.


 Pre-oxygenate with 100% oxygen (hyperoxygenation) for 3-4 breaths.
 Aspirate during withdrawal of catheter only, limiting each suction pass to 10 seconds.
 Allow client 4-5 recovery breaths between suction passes to replenish oxygen.

(Option 2)  Instilling 5-10 mL of sterile normal saline solution (NSS) is thought to help loosen thick secretions
and stimulate cough.  Although saline lavage is a common practice in some facilities, the installation of NSS
into the airway prior to suctioning is not recommended.  It can dislodge bacteria, causing increased bacterial
colonization, and can stimulate excessive coughing.

Educational objective:
Endotracheal suctioning increases risks for pulmonary infection, hypoxemia, microatelectasis, and mucosal
tissue damage.  To decrease these suctioning-associated risks, use sterile technique, preoxygenate, apply suction
only while withdrawing catheter, and limit aspiration time.  Avoid the use of saline lavage and frequent suction
passes without adequate rest between.

Intussusception is a process in which one part of the intestine prolapses and then telescopes into another part. 
It is one of the most frequent causes of intestinal obstruction during infancy.  Initially, the telescoping is
intermittent, resulting in periodic pain in association with the legs drawn up toward the abdomen.  Pain is
severe, progressive, and associated with inconsolable crying.  Ongoing obstruction can compromise circulation,
causing mucosal ischemia, occult bleeding, and, if untreated, grossly bloody "currant jelly" stools (mixture of
blood and mucus).
A contrast enema is used for diagnostic purposes and often reduces the intussusceptions.  An air enema is
considered safer than a barium enema.

(Option 1)  Human recombinant erythropoietin (epoetin alfa [Epogen, Procrit]) stimulates bone marrow to form
red blood cells and is used to combat the effects of chemotherapy (due to bone marrow suppression) and/or
kidney disease (erythropoietin is secreted by the kidneys).  Human recombinant erythropoietin is not indicated
in this client.

(Option 3)  Constipation during infancy usually can be corrected by increasing fluids or adding 2 ounces of
pear or apple juice to the daily diet.  In addition, eliminating constipating foods and increasing high-fiber foods
can help.  In this client, it is more important to treat the intussusception as there is no evidence of constipation.

(Option 4)  A hemoccult test is performed typically when occult (hidden) blood is suspected due to a dark and
tarry stool.  Blood is evident in intussusception, and so the priority in this client is to treat the cause of the
bloody mucus stool.

Educational objective:
Intussusception (the intestine telescoping into itself) causes intermittent cramping and progressive abdominal
pain, inconsolable crying, and currant jelly stool (from blood or mucus).  It is often treated successfully with an
air enema.

Atopic dermatitis, also known as eczema, is a chronic skin disorder characterized by pruritus, erythema, and
dry skin.  In infants, red, crusted, scaly lesions may also be present.  It is commonly first diagnosed before age
1 year.  The exact cause is unknown, although it is associated with an impaired skin barrier that allows
penetration of allergens, leading to an immune response.

The primary goals of management are to alleviate pruritus and keep the skin hydrated to prevent scratching. 
Scratching leads to the formation of new lesions and predisposes to secondary infections.  Important measures
to prevent scratching include cutting and filing nails short, placing gloves or cotton stockings over the hands,
not wearing rough fabrics or woolen clothing, and applying moisturizer.  These measures would have an
immediate effect in preventing scratching.

(Option 1)  A room humidifier may improve skin hydration and comfort in clients with excessively dry skin. 
However, comfort measures are not as crucial as immediate prevention of scratching (eg, gloves or cotton
stockings placed over the hands).

(Option 2)  Having an infant with severe atopic dermatitis may be a source of anxiety or stress for parents. 
Although it may be beneficial to explore the psychosocial effects on the family, prevention of scratching is a
higher priority as it can lead to secondary infection.

(Option 3)  Many clients with atopic dermatitis are also diagnosed with food sensitivities that aggravate the
condition and require a hypoallergenic diet.  However, nutritional education is a lower priority than infection
prevention.

Educational objective:
Atopic dermatitis (eczema) is a chronic skin disorder manifested in infants by pruritus, dry skin, and red,
crusted, scaly lesions.  The priority management is to prevent scratching as this would promote formation of
new lesions and predispose to secondary infections.

Codeine is an opioid drug prescribed as an analgesic to treat mild to moderate pain and as an antitussive to
suppress the cough reflex.  Although the antitussive dose (10-20 mg orally every 4-6 hours) is lower than the
analgesic dose, clients can still experience the common adverse effects (eg, constipation, nausea, vomiting,
orthostatic hypotension, dizziness) associated with the drug.

Codeine decreases gastric motility, resulting in constipation.  Increasing fluid intake and fiber in the diet and
taking laxatives are effective measures to prevent constipation (Option 2).

Changing position slowly is effective in preventing the orthostatic hypotension associated with codeine,
especially in the elderly (Option 4).

Taking the medication with food is effective in preventing the gastrointestinal irritation (eg, nausea, vomiting)
associated with codeine (Option 5).

(Options 1 and 3)  These statements are inaccurate as photosensitivity, insomnia, palpitations, and anxiety are
not adverse effects associated with codeine.

Educational objective:
The common adverse effects of codeine, an opioid drug, include constipation, nausea, vomiting, orthostatic
hypotension, and dizziness.  Interventions to help prevent them include increasing fluid intake and bulk in the
diet, laxatives, taking the medication with food, and changing position slowly.

Receptive aphasia refers to impairment or loss of language comprehension (ie, speech, reading) that is caused
by a neurological condition (eg, stroke, traumatic brain injury).  The terms "aphasia" and "dysphasia" can be
used interchangeably as both refer to impaired communication; however, "aphasia" is more commonly used.

When assisting a client with receptive aphasia to complete activities of daily living, the nurse should avoid
completing tasks for the client and should instead encourage independence using appropriate communication
techniques.  Appropriate interventions to aid communication include:

 Ask short, simple, "yes" or "no" questions (Option 1).


 Use gestures or pictures (eg, communication board) to demonstrate activities (Option 4).
 Remain patient and calm, allowing the client time to understand each instruction (Option 3).

(Option 2)  Clients with aphasia often become frustrated due to inability to communicate effectively. 
Frustration does not result from the nurse's care, so reassigning the client to a different care provider is not an
effective solution.

(Option 5)  Eye contact is important in all communication, but raising the voice will not help.  Speaking loudly
will not improve comprehension and may increase anxiety and confusion.

Educational objective:
Receptive aphasia refers to impairment or loss of language comprehension.  Appropriate interventions to aid
communication include asking short, simple, "yes" or "no" questions; using hand gestures or pictures to
demonstrate activities; and patiently allowing the client time to understand each instruction.

Cranial nerve IX (glossopharyngeal) is involved in the gag reflex, ability to swallow, phonation, and taste. 
Postoperative partial laryngectomy clients will need to undergo evaluation by a speech pathologist to evaluate
their ability to swallow safely to prevent aspiration.  Clients are taught the supraglottic swallow, a technique
that allows them to have voluntary control over closing the vocal cords to protect themselves from aspiration. 
Clients are instructed to:

1. Inhale deeply
2. Hold breath tightly to close the vocal cords
3. Place food in mouth and swallow while continuing to hold breath
4. Cough to dispel remaining food from vocal cords
5. Swallow a second time before breathing

(Option 1)  This would be considered "passing the buck."  The nurse should try to address the client's concerns
before calling the health care provider.

(Option 2)  Cranial nerve VIII (vestibulocochlear) affects hearing and equilibrium, not swallowing.

(Option 3)  The speech pathologist conducts a swallowing assessment early on to evaluate a client's ability to
swallow safely.  This consult is not done at discharge.

Educational objective:

Clients who undergo a partial laryngectomy are at increased risk for aspiration.  As a result, they are taught a
swallowing technique (supraglottic swallow) to decrease this risk.

The first phone call the nurse should return is to the client with acute sinusitis prescribed azithromycin 3 days
ago and now reporting hives.  Hives can be a manifestation of hypersensitivity to the macrolide antibiotic
azithromycin.  Anaphylaxis is a potential complication, and the drug should be discontinued immediately. 
Anaphylaxis poses the greatest threat to survival, so this is the priority call.

(Option 2)  Narcotic refills cannot be prescribed on the telephone and a new prescription is necessary; this is
not the priority call.

(Option 3)  A low-grade temperature, myalgia, headache, congestion, pain, redness, and itching at the injection
site are common side effects within 24 hours after receiving the influenza vaccine.  Clients often believe they
have the flu because the manifestations are similar; this is not the priority call.

(Option 4)  Palpitations are a common, expected side effect after use of a short-acting beta-agonist metered-
dose inhaler.  The nurse will assess the client's respiratory status and ask how often the client uses the rescue
inhaler; this is not the priority call.

Educational objective:
The same frameworks that guide nurses to prioritize nursing care can guide them to prioritize returning client
phone calls as well.  These include Maslow’s hierarchy of needs; and airway, breathing, cardiac status,
circulation, and vital signs (ABC plus V).

Meningitis is an inflammation of the meninges covering the brain and spinal cord.  The key clinical
manifestations of bacterial meningitis include fever, severe headache, nausea/vomiting, and nuchal rigidity. 
Other symptoms include photophobia, altered mental status, and other signs of increased intracranial
pressure (ICP).

In a hypotensive client with sepsis, the priority of care is fluid resuscitation to increase the client's blood
pressure (Option 2).

In addition to IV fluid administration, interventions and prescriptions for a client with sepsis and meningitis
may include:

 Administer vasopressors.
 Obtain relevant labs and blood cultures prior to administering antibiotics.
 Administer empiric antibiotics, preferably within 30 minutes of admission (Option 1).  This client will
continue to decline without antibiotic therapy.
 Prior to a lumbar puncture (LP), obtain a head CT scan as increased ICP or mass lesions may
contraindicate a LP due to the risk of brain herniation (Option 4).
 Assist with a LP for cerebrospinal fluid (CSF) examination and cultures (Option 3).  CSF is usually
purulent and turbid in clients with bacterial meningitis.  CSF cultures will allow for targeted antibiotic
therapy.

Educational objective:
For bacterial meningitis with sepsis, fluid resuscitation is the priority.  Blood cultures should be drawn before
starting antibiotics.  After a head CT scan is performed to rule out increased intracranial pressure and mass
lesions, cerebrospinal fluid cultures should be drawn via lumbar puncture.

Carpal tunnel syndrome (CTS) is caused by compression of a median nerve within the carpal tunnel at the
wrist.  Any swelling in the canal puts pressure on the nerve and produces pain and paresthesia in the median
nerve distribution (first 3½ digits).  These symptoms are often worse at night when the wrists are flexed during
sleep.  The most commonly used conservative treatment is wrist splinting, particularly at nighttime.  Splinting
of the wrist prevents excessive flexion or extension, which could narrow the carpal tunnel.

(Option 1)  Caffeine and tobacco products do not affect CTS.

(Options 2 and 3)  Repetitive hand exercises and elastic compression would make the symptoms worse by
narrowing the carpal tunnel.

Educational objective:
Carpal tunnel syndrome (CTS) is caused by compression of a median nerve at the wrist.  Nighttime wrist
splinting is most beneficial.

A normal blood glucose range for an infant is 40-60 mg/dL (2.2-3.3 mmol/L) within the first 24 hours after
delivery.  A blood glucose level <40 mg/dL (2.2 mmol/L) indicates hypoglycemia.  Symptoms of hypoglycemia
include jitters, cyanosis, tremors, pallor, poor feeding, retractions, lethargy, low oxygen saturation, and
seizures.  This infant with borderline-low glucose level is symptomatic and should be assessed first.

(Option 1)  A normal respiratory rate for an infant is 30–60/min.  This infant is currently stable.

(Option 3)  It is normal to auscultate crackles in an infant during the first hour of life.  This is because fluid is
still being pushed out of and absorbed by the lungs.  This infant is currently stable.

(Option 4)  A normal temperature range for an infant is 97.7–99.7 F (36.5–37.6 C).  This infant is currently
stable.

Educational objective:
The nurse should monitor infants for hypoglycemia by assessing for symptoms and monitoring the blood
glucose level.  A blood glucose level <40 mg/dL (2.2 mmol/L) indicates hypoglycemia and should be treated
immediately by feeding or administering a glucose bolus.

Colonoscopy evaluates colonic mucosa.  Therefore, clients should follow instructions to keep the colon clean
with no stool left for better visualization during the procedure.  These instructions include:

1. Clear liquid diet the day before


2. Nothing by mouth 8–12 hours prior to the examination
3. The health care provider prescribes a bowel-cleansing agent such as a cathartic, enema, or polyethylene
glycol (GoLYTELY) the day before the test.  The type of prep depends on the health care provider's
preference and client health status.

(Option 2)  Healthy clients screened for colon disease do not require antibiotics prior to the procedure.

(Option 3)  The instructions prior to a nuclear gastric emptying scan include teaching the client to avoid
smoking the day of the examination as delay of gastric emptying occurs with tobacco use.  Smoking cessation
per se has no role in colonoscopy, but it is good for general health.

Educational objective:
Instructions for clients scheduled for a colonoscopy include a clear liquid diet the day before the procedure,
avoiding any food or liquids (nothing by mouth) 8–12 hours prior to the examination, and taking the bowel-
cleansing agent as prescribed.

The nurse should first call the client with tingling in the right foot.  Musculoskeletal injuries and immobilization
devices (cast) can cause neurologic or vascular damage to the extremity distal to the injury.  Paresthesia (eg,
numbness, tingling) is an early sign of neurovascular impairment (nerve ischemia).  It would be important for
the client to report to the HCP for immediate evaluation.  This is the most urgent call to return.

(Option 1)  Nausea is an expected side effect of the synthetic opioid pain reliever, oxycodone.  The nurse can
instruct this client to take this medication with food, which may help alleviate the nausea.  This is not the most
urgent call.

(Option 3)  Clients with diabetes are usually able to take the prescribed insulin dose when ill, and some clients
may need a higher dose.  Illness is a physiologic stressor and can increase blood glucose level.  On the other
hand, if the oral intake is low, blood sugars can be low and insulin may need to be reduced.  The best step is to
instruct this client to check glucose level and repeat every 4 hours and to report glucose levels above or below
the target range to the HCP for specific orders.  This is not the most urgent call.

(Option 4)  Amitriptyline (Elavil), a tricyclic antidepressant drug, can be prescribed for difficulty sleeping due
to chronic pain of fibromyalgia, but it may not be effective in all clients.  This is not the most urgent call.

Educational objective:
Musculoskeletal injuries and immobilization devices (cast) can cause neurologic or vascular damage to the
extremity distal to the injury.  Paresthesia (eg, numbness, tingling) is an early sign of neurovascular impairment
(nerve ischemia).

The new graduate RN should be given RN-level responsibilities caring for stable clients who do not require
specialized knowledge and performing skills taught in nursing school.

Multiple sclerosis is a chronic relapsing and remitting nerve disorder caused by patchy demyelination of nerve
fibers in the brain and spinal cord.  Ataxia is an expected finding.  The client is relatively stable and appropriate
for management by a new graduate (Option 1).

A client being discharged after an acute stroke is presumably stable.  This task (eg, calling report) is an RN-
level responsibility (Option 2).

Initial teaching must be done by the RN, not an LPN.  Providing client teaching for common medications is
within the skill and knowledge set of a new graduate RN (Option 5).
(Choice 3)  Guillain-Barré syndrome results in bilateral, ascending paralysis.  The ascent can be rapid and it is
unknown at what level of the client's body the ascension will stop.  This client is unstable.  A key concern is the
paralysis ascending to the diaphragm and causing respiratory failure.

(Choice 4)  The GCS score provides objective monitoring of acute head injury.  This is an unstable client near
the stage of a coma, which is a GCS score of <8.  Urgent intubation is anticipated due to the risk of rapid loss of
airway.  The acuity and uncertainty of the nursing interventions in this situation require an experienced RN.

Educational objective:
New graduate RNs should be assigned to stable clients who do not require specialized skill or knowledge. 
Appropriate clients include those being discharged or requiring new teaching.

he nurse-client relationship is the basis of quality nursing treatment approaches in mental health.  It should have
clear boundaries that allow for the client to examine feelings and treatment issues.  The nurse's needs are clearly
separated from the client's needs.  However, roles can become blurred when transference and
countertransference are not recognized.  The act of a client unconsciously displacing (transferring) feelings
and behaviors related to a person in the client's past onto the nurse is known as transference.  The nurse
unknowingly displacing feelings and behaviors about someone in the nurse's past onto the client is known as
countertransference.  These phenomena disrupt the therapeutic nurse-client relationship.

(Options 1, 2, and 4)  These client statements do not represent transference.

Educational objective:
It is important for the nurse to recognize transference in order to maintain a therapeutic nurse-client
relationship.

A colostomy is a surgical procedure that creates an opening (stoma) in the abdominal wall for the passage of
stool to bypass an obstructed or diseased portion of the colon.  Stool drains through the intestinal stoma into a
pouch device secured to the skin.  Colostomies can be performed on any part of the colon (ascending,
transverse, descending, and sigmoid).  Depending on the location of the colostomy, characteristics of the stool
will vary, with the stool becoming more solid as it passes through the colon.

Proper care of the stoma and pouch appliance that should be taught to the client or caregiver includes the
following:

 Ensure sufficient fluid intake (at least 3,000 mL/day unless contraindicated) to prevent dehydration;
identify times to increase fluid requirements (hot weather, increased perspiration, diarrhea) (Option 4).
 Identify and eliminate foods that cause gas and odor (broccoli, cauliflower, dried beans, brussels
sprouts) (Option 1).
 Empty the pouch when it becomes one-third full to prevent leaks due to increasing pouch weight
(Option 2).

(Option 3)  Stool produced in the ascending and transverse colon is semiliquid, which eliminates the need for
irrigation.  Irrigation to promote a bowel regimen may be useful for descending or sigmoid colostomies as the
stool is more formed.

Educational objective:
The stool changes from liquid to more solid as it passes through the colon.  Proper care of the ostomy and
pouching device in clients with a colostomy includes ensuring sufficient fluid intake, preventing gas and odor,
and changing the pouching system when it becomes one-third full to prevent leaks.
Torsades de pointes (ie, "twisting of the points") is a polymorphic ventricular tachycardia characterized by
QRS complexes that change size and shape in a characteristic twisting pattern.

Torsades de pointes is usually due to a prolonged QT interval (more than half the RR interval), which is the
result of electrolyte imbalances, especially hypomagnesemia, or some medications.  The first-line treatment is
IV magnesium (Option 3).  Treatment may also include defibrillation and discontinuation of any QT-
prolonging medications.

(Option 1)  Adenosine is an antiarrhythmic used to treat supraventricular tachycardia.

(Option 2)  Dopamine is a vasopressor used to treat symptomatic hypotension.

(Option 4)  Metoprolol is a beta blocker used for heart rate control in tachyarrhythmias.

Educational objective:
Torsades de pointes is usually due to a prolonged QT interval, which is the result of electrolyte imbalances,
especially hypomagnesemia, or some medications.  First-line treatment is magnesium IV.  Treatment may also
include defibrillation and discontinuation of QT-prolonging medications.

Mannitol (Osmitrol) is an osmotic diuretic used to treat cerebral edema (increased intracranial pressure) and
acute glaucoma.  When administered, mannitol causes an increase in plasma oncotic pressure (similar to excess
glucose) that draws free water from the extravascular space into the intravascular space, creating a volume
expansion.  This fluid, along with the drug, is excreted through the kidneys, thereby reducing cerebral edema
and intracranial pressure.  However, if a higher dose of mannitol is given or it accumulates (as in kidney
disease), fluid overload that may cause life-threatening pulmonary edema results.  An early sensitive indicator
of fluid overload is new onset of crackles auscultated in the lungs.  To prevent these complications, clients
require frequent monitoring of serum osmolarity, input and output, serum electrolytes, and kidney function.

(Option 2)  Urine output would be expected to increase from the diuretic effect of mannitol.  This is not a
complication.

(Option 3)  Glasgow Coma Scale scores range from 3-15.  Improved mental status (orientation, alertness) is a
desired effect of treatment.

(Option 4)  The presence of crackles is a more sensitive sign of fluid overload than pedal edema.  Furthermore,
in a bedridden client, the assessment should take place at a dependent part of the body, usually the sacral area.

Educational objective:
Mannitol is an osmotic diuretic used to treat cerebral edema and acute glaucoma.  Normal kidney function and
adequate urine output are crucial while administering this medication as mannitol accumulation can result in
significant volume expansion, dilutional hyponatremia, and pulmonary edema.

Acute care for diverticulitis focuses on allowing the colon to rest and the inflammation to resolve.  This
includes the following:

 IV antibiotic therapy – to cover the gram-negative and anaerobic organisms that reside in the colon and
contribute to diverticulitis; these commonly include metronidazole (Flagyl) plus
trimethoprim/sulfamethoxazole (TMZ) (Bactrim or Bactrim DS; Septra) or ciprofloxacin (Cipro)
(Option 1)
 NPO status – more acute cases require complete rest of the bowel (NPO status); less severe cases may
be handled at home, and clients may tolerate a low-fiber or clear liquid diet (Option 3)
 NG suction – in severe cases of abdominal distention, nausea, or vomiting (Option 2)
 IV fluids – prevent dehydration
 Bed rest

(Option 4)  Any procedure or treatment that increases intraabdominal pressure (lifting, straining, coughing,
bending), increases peristalsis (laxative, enema), or could lead to perforation or rupture of the inflamed
diverticula should be avoided during the acute disease process.  A barium enema may be used after treatment
with antibiotics and the inflammation is resolved.  Diagnostic examinations, such as abdominal x-rays or CT
scans, may be used without risking rupture.

Educational objective:
Management of acute diverticulitis focuses on bowel rest (NPO status, NG suction, bed rest), and drug
therapy (IV antibiotics, analgesics).  Any procedure or treatment that increases intraabdominal pressure or may
cause rupture of the inflamed diverticula should be avoided.

An ischemic stroke is a loss of brain tissue perfusion due to blockage in blood flow.  Elevated blood pressure is
common and permitted after a stroke and may be a compensatory mechanism to maintain cerebral perfusion
distal to the area of blockage.  This permissive hypertension usually autocorrects within 24-48 hours and does
not require treatment unless the hypertension is extreme (systolic blood pressure >220 mm Hg or diastolic blood
pressure >120 mm Hg) or contraindicated due to the presence of another illness requiring strict blood pressure
control (eg, active ischemic coronary disease, heart failure, aortic dissection).  A blood pressure of 178/95 mm
Hg should be monitored, along with the client's other vital signs and status (Option 2).

(Option 1)  The elevated blood pressure may be a protective measure to ensure tissue perfusion; therefore, the
antihypertensive drug should not be given unless indicated by the prescription parameters.

(Options 3 and 4)  The as-needed prescription is appropriate in this case, so there is no reason to question it and
call the health care provider.

Educational objective:
Elevated blood pressure in the presence of an acute ischemic stroke is an expected finding and may be a
protective measure to maintain tissue perfusion.

Asystole represents the total absence of ventricular electrical activity in the heart.  No ventricular contraction
occurs.  The client is pulseless, apneic, and unresponsive.  Asystole is usually a result of advanced cardiac
disease, end-stage heart failure, or a severe electrical conduction system disturbance.  Treatment consists of
cardiopulmonary resuscitation (CPR) and initiation of advanced cardiac life support measures, including
administration of epinephrine and/or vasopressin, placement of an advanced airway, and treatment of any
reversible causes.

(Option 2)  Clients in complete heart block will have electrical activity present on the electrocardiogram
(ECG).  Atrial and ventricular rhythms are unrelated to each other.

(Option 3)  Lead connections should be checked in asystole, but the nurse has already assessed the absence of
heart tones and unresponsiveness in the client.

(Option 4)  Ventricular fibrillation is characterized by chaotic electrical activity on the ECG.

Educational objective:
Asystole is characterized on the ECG by complete absence of ventricular electrical activity.  The client will be
pulseless, apneic, and unresponsive.  Treatment consists of CPR, advanced cardiac life support measures, and
treatment of any reversible causes.

The average age of menopause in the United States is 50-52.  Major health risks of menopause include
osteoporosis and heart disease.  Bisphosphonates, such as alendronate (Fosamax), risedronate (Actonel), or
ibandronate (Boniva), decrease bone resorption so that loss of bone density is minimized.  They must be
consumed in the morning, on an empty stomach, with at least 30 minutes before other drugs.  The medication is
taken with a full glass of water and the client must remain upright for at least 30 minutes to aid absorption
and prevent esophageal irritation (Option 4).

Adequate sources (both food and supplements) of calcium and vitamin D are required to build bone mass
(Option 1).

HRT can improve bone mass and prevent osteoporosis but is associated with increased risk of thrombotic
complications (deep vein thrombosis, stroke, myocardial infarction) and some cancers (breast, uterine). 
Therefore, it is used only in clients who have disabling hot flashes.  Unilateral leg swelling is a classic symptom
of venous thromboembolism (Option 3).

(Option 2)  Anemia in older adults is usually not related to lack of iron intake, especially once menstruation has
stopped.  Excessive iron intake can lead to iron overload, and the risk of excess iron tends to be higher with
aging.

(Option 5)  Postmenopausal bleeding or abnormal premenopausal bleeding is the most common symptom of
endometrial cancer and requires follow-up.

Educational objective:
A postmenopausal woman (usually after age 51) is at risk for osteoporosis and heart disease.  Clients should
remain upright after taking a bisphosphonate and consume calcium and vitamin D for bone health.  Clotting
disorder is a risk with HRT.  Intermittent vaginal spotting after menopause can be a sign of endometrial cancer.

This client with pneumonia and hyperactive airway disease with increased shortness of breath and new-onset
high-pitched wheezing is at increased risk for problems in oxygenation and airway management and should
be assessed first.  Clients with symptomatic reactive airway disease can receive inhaled and/or nebulized beta
agonists as needed.  It is a priority to assess baseline lung sounds and breathing for comparison with subsequent
findings and to determine respiratory status and the need for a bronchodilator treatment.

(Option 1)  This client with cellulitis pain received hydromorphone (Dilaudid) IV one hour ago, and it is too
soon to be remedicated at this time (ie, frequency should be every 2-4 hours).  The nurse needs to reassess pain,
but the client with respiratory problems takes priority.

(Option 2)  Shortness of breath with activity is expected in a client with chronic kidney disease and decreased
hemoglobin (normal 13.2-17.3 g/dL [132-173 g/L] for males, 11.7-15.5 g/dL [117-155 g/L] for females ) and
hematocrit (normal 39%-50% [0.39-0.50] for males, 35%-47% [0.35-0.47] for females) that is most likely
associated with decreased production of the hormone, erythropoietin.  This does not require immediate
attention.

(Option 3)  Dilutional hyponatremia (serum sodium <135 mEq/L [135 mmol/L]) is expected in this client with
heart failure and bilateral pleural effusions due to excess fluid.  The nurse needs to assess mental status, lung
sounds, and respiratory status, validate that the informed consent is signed, and make sure that the client
understands the procedure.  Hyponatremia can cause change in level of consciousness (with sodium <120-125
mEq/L [120-125 mmol/L]), but sodium level of 133 mEq/L (133 mmol/L) is only borderline low and does not
require immediate attention.
Educational objective:
Information communicated in the nurse-to-nurse change-of-shift hand-off report helps the oncoming nurse
identify client problems and potential complications.  The ABC (airway, breathing, circulation) and Maslow's
Hierarchy of Needs frameworks can guide the nurse to properly prioritize nursing care.

Clients with hypertension should be instructed not to take potentially high-risk over-the-counter (OTC)
medications such as high-sodium antacids, appetite suppressants, and cold and sinus preparations.

It is appropriate to ask a client with hypertension about taking OTC cold medications as many cold and sinus
medications contain phenylephrine or pseudoephedrine.  These sympathomimetic decongestants activate
alpha-1 adrenergic receptors, producing vasoconstriction.  The resulting decreased nasal blood flow relieves
nasal congestion.  These agents have both oral and topical forms.  With systemic absorption, these agents can
cause dangerous hypertensive crisis.

(Option 2)  Taking extra vitamin C may offer some protection for the immune system, but it does not cause an
increase in blood pressure.

(Option 3)  Exposure to young children increases the risk for contracting a contagious respiratory illness, but it
does not directly increase blood pressure.

(Option 4)  A flu shot would not offer protection against the flu within a week and does not cause an increase in
blood pressure.

Educational objective:
Clients with hypertension should be instructed not to take potentially high-risk over-the-counter medications,
including high-sodium antacids, appetite suppressants, and cold and sinus preparations, as they can increase
blood pressure.

Angina is chest pain due to myocardial ischemia.  A client with chronic stable angina experiences intermittent
chest pain relieved with rest or administration of nitroglycerin.  The priority action for acute angina is
administration of rapid-acting (1-3 minutes) sublingual nitroglycerin to restore cardiac perfusion. 
Nitroglycerin is a vasodilator that decreases cardiac workload (decreasing oxygen consumption), reduces
preload, and increases myocardial perfusion.  Onset and duration of action of nitroglycerin varies with route of
administration.

(Option 1)  Morphine sulfate relieves pain and has a mild vasodilator effect that decreases cardiac workload. 
Morphine is given if nitroglycerin does not relieve chest pain.

(Option 3)  Transdermal nitroglycerin patches have a delayed onset of action (40-60 minutes) and are not
effective in the treatment of acute anginal pain.  If a patch is accidentally removed, a new one may be applied
after the nurse first administers sublingual nitroglycerin.

(Option 4)  A client with known stable angina is expected to have exertional chest pain if the pain is not
prevented with nitroglycerin (eg, transdermal patch, prophylactic sublingual dose).  Restoration of myocardial
perfusion with sublingual nitroglycerin should not be delayed to obtain an electrocardiogram (ECG).  The nurse
should consider obtaining an ECG and implementing emergency measures if the pain does not resolve after 3
doses of sublingual nitroglycerin (unstable angina).

Educational objective:
Acute stable angina is managed with nitroglycerin, which causes vasodilation and restores myocardial
perfusion.  Sublingual nitroglycerin has a rapid onset and is used to treat acute angina by increasing myocardial
perfusion; transdermal patches have a delayed onset and are used prophylactically.
An ECG should be performed immediately on all adult clients with chest pain; all chest pain should be
considered cardiac until proven otherwise.  After the initial ECG, the client with chest pain will need to be
placed on a cardiac monitor and assessed by the health care provider before the other 3 clients.

(Option 2)  This client will need a prescription renewal.  Glargine (Lantus) is given once a day, typically in the
evening, as basal insulin.  The consequence of late administration is hyperglycemia.  A single temporary rise in
glucose will not have a significant negative impact.  The damage to vessels in a diabetic client comes from
long-term uncontrolled diabetes.  The other clients are a higher priority.

(Option 3)  This client may have a deep vein thrombosis and will probably require anticoagulant therapy. 
However, this client is hemodynamically stable without evidence of active pulmonary embolism and can safely
wait to be seen after the higher-risk client with chest pain.

(Option 4)  This client may have acute diverticulitis and should be seen urgently, but after the client with chest
pain.  Prioritization should be based on which client is most ill and not on advanced age.

Educational objective:
Chest pain in an adult, regardless of age, is a priority.  It is important to not make assumptions based on client
age, race, or nationality.

A cystoscopy is a procedure that uses a flexible fiber-optic scope inserted through the urethra into the urinary
bladder with the client in the lithotomy position.

Complications associated with cystoscopy include urinary retention, hemorrhage, and infection.  Therefore,
clients are instructed to notify the health care provider (HCP) immediately if they have bright red blood when
urinating, blood clots, inability to urinate, fever >100.4 F (38 C) and chills, or abdominal pain unrelieved by
analgesia.  These conditions necessitate evaluation by the HCP and may require antibiotic therapy or the
insertion of a urinary catheter to irrigate the bladder, remove clots, or drain the bladder (Option 4).

(Options 1 and 3)  Pink-tinged urine, frequency, and dysuria are expected for up to 48 hours following a
cystoscopy.  Clients are instructed to increase fluids, drink 4-6 glasses of water daily to help dilute the urine,
and avoid alcohol and caffeine for 24-48 hours as these can irritate the bladder.

(Option 2)  Abdominal discomfort and bladder spasms may occur for up to 48 hours following the procedure. 
Clients are taught to take a mild analgesic (eg, acetaminophen, ibuprofen) and a warm tub/sitz bath (except with
recurrent urinary tract infections) for pain relief.

Educational objective:
Clients can expect pink-tinged urine, frequency, dysuria, and abdominal discomfort for up to 48 hours after
cystoscopy.  They are instructed to increase fluid intake, avoid alcohol and caffeine, take a mild analgesic and
tub/sitz bath to relieve discomfort, and notify the HCP immediately of inability to void, gross hematuria, blood
clots, fever, chills, or severe pain.

To prevent hip flexion contractures in clients with above-the-knee amputation, the residual limb should not be
elevated, especially after 24 hours.  Instead, edema should be managed using a figure eight compression
bandage.  The bandage should be worn at all times until the residual limb is healed, and care should be taken
not to wrap it too tightly.  Hip flexion contractures can also be avoided by placing the client in prone position
with hip in extension for 30 minutes 3 or 4 times a day.

(Option 2)  Following total hip replacement, hip dislocation is prevented by using an abductor pillow to
maintain the hip in a straight and neutral position.  The nurse should also teach the client not to bend at the hip
more than 90 degrees or cross the legs or ankles.
(Option 3)  Buck traction immobilizes hip and femur fractures.  A boot or traction tape is applied to the
affected extremity and a prescribed weight pulls the limb into traction.  The client is typically placed in supine
position with the foot of the bed raised to maintain countertraction.

(Option 4)  After a new cast is placed, the nurse should elevate the client's limb above the heart for the first 48
hours to increase venous return and decrease edema in the affected extremity.  However, the extremity should
not be elevated if compartment syndrome develops.

Educational objective:
Care of the client with above-the-knee amputation includes placement in prone position for 30 minutes 3 or 4
times a day and using a figure eight compression bandage to decrease edema.  The client's residual limb should
not be elevated as this will promote flexion contractures.

Oral mucositis, inflammation or ulceration of the oral mucosa, results from chemotherapy or radiation therapy. 
Oral hygiene practices that minimize oral mucositis and promote comfort include the following:

 Cleansing the mouth with normal saline after meals and at bedtime to promote oral health
 Use of a soft-bristle toothbrush to decrease gum irritation
 Application of prescribed viscous lidocaine HCl (Xylocaine) to alleviate oral pain
 Use of water-soluble lubricating agents to moisten mouth tissues that may become dry due to therapy
 Avoidance of hot liquids and spicy/acidic foods, which can cause oral discomfort

(Option 5)  Clients with mucositis should avoid antiseptic mouthwashes with alcohol as they are irritating to
mucous membranes.

(Option 6)  Administration of palifermin (Kepivance), a recombinant human keratinocyte growth factor,
prevents oral mucositis in clients diagnosed with hematologic malignancies.  However, it does not help with
pain.  Viscous lidocaine HCl (Xylocaine) alleviates the oral pain caused by mucositis.

Educational objective:
Measures to minimize oral mucositis from chemoradiotherapy include rinsing the mouth with normal saline,
brushing with a soft-bristle toothbrush, using a water-soluble lubricating agent, avoidance of hot liquids and
spicy/acidic foods, and application of prescribed viscous lidocaine.

A reddened area on the sacrum puts the client at risk for skin breakdown.  The nurse should first perform an
assessment on the client's skin to see if there are any other reddened areas or skin breakdown present.  This
should be compared to previous assessments or serve as a baseline assessment of skin integrity.  The Braden
Scale, a tool for predicting pressure sore risk, would be appropriate to use as part of the assessment.

(Option 1)  After the nurse has performed a skin assessment, it may be appropriate to direct the UAP to apply a
protective foam dressing to the area.

(Option 2)  Documentation should occur after the client has been assessed thoroughly and received care.

(Option 3)  After assessing the client, the nurse can decide whether to notify the HCP.

Educational objective:
When the nurse receives report of a change in client condition from the UAP, the nurse should reassess the
client before completing other interventions.
Unexpected and projectile vomiting without nausea can be a sign of increased ICP, especially in the client
with a history of increased ICP.  The unexpected vomiting is related to pressure changes in the cranium.  The
vomiting can be associated with headache and gets worse with lowered head position.  The most appropriate
action is to obtain a full set of vital signs and contact the HCP immediately.

(Option 1)  Documentation is important, but it is not the priority action.

(Option 2)  The head of the bed should be raised, not lowered, for clients with suspected increased ICP. 
Raising the head of the bed to 30 degrees helps to drain the cerebrospinal fluid via the valve system without
lowering the cerebral blood pressure.

(Option 4)  The vomiting is caused not by nausea but by pressure changes in the cranium.  Anti-nausea
medications are often not effective.  Decreasing intracranial pressure will help the vomiting.

Educational objective:
Notify the HCP of signs/symptoms of increased ICP, including unexpected vomiting.  The vomiting is often
projectile, associated with headache, and gets worse with lowering the head position.

The pursed-lip breathing technique helps to decrease shortness of breath by preventing airway collapse,
promoting carbon dioxide elimination, and reducing air trapping in clients with chronic obstructive
pulmonary disease (COPD).  Clients with COPD are taught to use this technique when experiencing dyspnea as
it increases ventilation and decreases work of breathing.  Regular practice (eg, 5–10 minutes 4 times daily)
enables the client to do pursed lip breathing when short of breath, without conscious effect.  Clients are taught
the following steps:

1. Relax the neck and shoulders


2. Inhale for 2 seconds through the nose with the mouth closed (Option 4)
3. Exhale for 4 seconds through pursed lips.  If unable to exhale for this long, exhale twice as long as
inhaling (Option 2).

(Option 1)  Exhalation through pursed lips is done for 4 seconds, not 2 seconds, or twice as long as inhalation.

(Option 3)  Inhalation is done through the nose, not the mouth.

(Option 5)  Inhalation is done for 2 seconds, not 4 seconds.

Educational objective:
The pursed-lip breathing technique helps to decrease shortness of breath by preventing airway collapse,
promoting carbon dioxide elimination, and reducing air trapping in clients with chronic obstructive pulmonary
disease.  Clients are taught to relax the shoulders and neck, inhale through the nose for 2 seconds with the
mouth closed, and exhale through pursed lips for 4 seconds (or twice as long as inhalation).

The major adverse effects of sulfonylurea medications (eg, glyburide, glipizide, glimepiride) are
hypoglycemia and weight gain.  Weight gain should be addressed.  Clients taking glyburide should be taught
to use sunscreen and protective clothing as serious sunburns can occur.

(Option 1)  Clients taking sulfonylureas should avoid alcohol as it lowers blood glucose and can lead to severe
hypoglycemia.
(Option 2)  Hypoglycemia (serum glucose <70 mg/dL [3.9 mmol/L]) is a major side effect of sulfonylurea
medications.  A fasting blood glucose <60 mg/dL (3.3 mmol/L) indicates moderate to severe hypoglycemia and
the medication needs to be reassessed.

(Option 3)  Even foods labeled "diabetic", "sugar free," or "sugarless" may contain carbohydrates such as
honey, brown sugar, and corn syrup, all of which can elevate blood sugar.

Educational objective:
The major adverse effects of sulfonylurea medications are hypoglycemia and weight gain.  Alcohol must be
avoided while taking these medications due to the risk of severe hypoglycemia.  Glyburide can also make
clients sunburn easily.

The priority when administering 2 IV medications concurrently is to determine drug compatibility. 


Incompatible drugs given through the same IV line will deteriorate or form a precipitate.  This change is
visualized through either a color change, a clouding of the solution, or the presence of particles.  If 2 or more
drugs are not compatible, the nurse may consider inserting a second IV or consulting the pharmacist and the
health care provider to determine the safest and most beneficial plan for the client.

(Option 1)  Assessing the IV site for complications (eg, infiltration, phlebitis) should always be performed
before giving any IV medication.  This will be completed after determining drug compatibility.

(Option 2)  Verification using 2 client identifiers pertains to the "right client" in the "6 rights" of medication
administration.  Drug compatibility should be determined prior to entering the client's room and verifying
identity.

(Option 4)  Hand hygiene is a standard precaution taken before any type of client interaction to prevent
contamination and infection; hand washing will be completed after checking for drug compatibility.

Educational objective:
Checking for drug compatibility is a priority before administering 2 IV medications concurrently in the same IV
site.  Incompatible drugs will deteriorate or form a precipitate that is visible as a color change, cloudiness, or
particulates.

Mechanical prosthetic valves are more durable than biological valves but require long-term anticoagulation
therapy due to the increased risk of thromboembolism.  The client should be taught ways to reduce the risk of
bleeding.

Teaching topics for clients on anticoagulants:

 Take medication at the same time daily


 Depending on medication, report for periodic blood tests to assess therapeutic effect
 Avoid any action that may cause trauma/injury and lead to bleeding (eg, contact sports, vigorous teeth
brushing, use of a razor blade) (Option 4)
 Avoid aspirin and nonsteroidal anti-inflammatory drugs (NSAIDs)
 Limit alcohol consumption
 Avoid changing eating habits frequently (eg, dramatically increasing intake of foods high in vitamin K
such as kale, spinach, broccoli, greens) (Option 2) and do not take vitamin K supplements
 Consult with health care provider before beginning or discontinuing any medication or dietary/herbal
supplement (eg, Ginkgo biloba and ginseng affect blood clotting and may increase bleeding risk)
(Option 1)
 Wear a medical alert bracelet indicating what anticoagulant is being taken
(Option 3)  Early in the recovery period, care of the incision site typically includes washing with soap and
water and patting it dry.  Ointments (eg, vitamin E) may be applied after the incision has healed.

Educational objective:
Clients who are on anticoagulants should avoid aspirin, NSAIDS, and other over-the-counter or herbal products
(eg, Ginkgo biloba) that can increase bleeding risk.  They should also avoid behaviors that increase the risk of
clotting (eg, eating excess green leafy vegetables).

A chemical burn to the eye is an emergency.  Alkali burns (concrete, drain cleaners containing lye) are
particularly concerning as they will denude the protein and continue to penetrate until the substance is
completely removed.  Copious irrigation with water (at home) or normal saline/lactated Ringer's solution is
started immediately.  If the client cannot open the eye, another person should help open the eyelid.  The
irrigation should continue in the ambulance and in the emergency department with a special irrigating device
that looks like a large contact lens.  The pH of the eye is obtained prior to irrigation and irrigations continue
until the eye pH is 7.0-7.5.  Irrigation can last up to 60 minutes.

(Option 1)  Topical anesthetic eye drops may be instilled prior to eye irrigation as eye burns are very painful. 
Systemic analgesia is not a priority.

(Option 3)  Additional steps after copious irrigation may include covering the eye with an eye patch and use of
eye drops to prevent eye muscle spasms.

(Option 4)  The Snellen eye chart is commonly used to determine visual acuity.  However, eye irrigation to
remove the substance is essential and is never delayed to determine acuity first.

Educational objective:
The priority with caustic substances (especially alkaline) in the eyes is copious eye irrigation with water or
normal saline/lactated Ringer's.

VAP is the second most common health care-associated infection (HAI) in the United States and is associated
with increased mortality, hospital cost, and length of stay.

Because it is a nosocomial infection, signs and symptoms associated with VAP usually present within ≥2-3 days
after initiation of mechanical ventilation (MV).  Characteristic clinical manifestations of VAP include purulent
sputum, positive sputum culture, leukocytosis (12,000 mm3), elevated temperature (>100.4 F [38 C]), and
new or progressive pulmonary infiltrates suggestive of pneumonia on chest x-ray.

(Option 1)  Blood-tinged sputum may occur but is not the best indicator of VAP.

(Option 2)  Positive blood cultures may identify the microorganism causing the infection but are not the best
indicator of VAP.  Positive blood cultures could be from another source of infection.

(Option 4)  Rhonchi and crackles are adventitious lung sounds associated with pneumonia but can be present in
pulmonary edema or just from increased mucous secretions.  They are not the best indicator of VAP.

Educational objective:
VAP is an HAI that usually occurs within ≥2-3 days after the initiation of mechanical ventilation. 
Characteristic manifestations of VAP include purulent secretions, positive sputum culture, leukocytosis,
elevated temperature, and new or progressive pulmonary infiltrates on chest x-ray.
Hepatic encephalopathy (HE), a potentially reversible disturbance in central nervous system function, results
when the liver fails to detoxify the body of ammonia.  Clients with cirrhosis or other forms of liver disease are
at risk for developing HE.  Clinical findings include:

 Cognitive deficits – Confusion and disorientation, impaired thinking and judgment (eg, inability to
perform basic math), loss of meaningful conversation
 Mental status changes – Sleep disturbances and progressively altered level of consciousness, leading to
coma if not effectively treated (Option 2)
 Motor alterations – Asterixis (ie, flapping hand tremor when arms and hands are extended),
hyperreflexia, apraxia (ie, inability to draw simple figures) (Option 3)
 Fetor hepaticus (ie, musty, sweet odor to the breath) (Option 1)

(Option 4)  Constipation is a risk factor for HE.  The lactulose goal in clients with cirrhosis or HE is to have 2-3
soft bowel movements a day.

(Option 5)  Constricted pupils are seen with use of opioids.  Pupillary changes are not significant in HE.

Educational objective:
Hepatic encephalopathy, a complication of liver disease, results from the accumulation of ammonia in the
bloodstream.  Clinical findings include changes in level of consciousness, asterixis, and fetor hepaticus.

Elevation promotes mobilization of fluids (eg, blood, lymph drainage, edema) back into circulation.

Elevating a limb with deep vein thrombosis promotes venous return and reduces edema.  The nurse should
elevate the affected extremity without applying direct pressure at the thrombus site (Option 4).

Cellulitis is characterized by an edematous rash from subcutaneous tissue inflammation.  The nurse should
elevate the extremity to promote lymphatic drainage of edema (Option 5).

(Option 1)  For a client with an above-the-knee amputation, the affected limb is elevated by raising the foot of
the bed for the first 24 hours only, to reduce edema.  After 24 hours, the client is placed in a prone position
twice daily to promote stretching of the flexor muscles and prevent contractures.

(Option 2)  For a client with a hip fracture, the affected extremity is kept abducted using splints or pillows
between the legs; elevation is contraindicated due to the risk of affecting alignment.

(Option 3)  To prevent bleeding (eg, insertion site, retroperitoneal, hematoma) following percutaneous coronary
intervention, the client should lie flat with the affected hip extended (straight) while sheaths remain in place and
for a few hours after sheath removal.

Educational objective:
Therapeutic positioning must be evaluated carefully by the nurse.  Elevating an extremity can reduce edema,
promote comfort, and increase venous return.  Elevation is inappropriate for clients with hip fractures, recent
percutaneous coronary intervention, or above-the-knee amputation more than 24 hours ago.

Erythropoiesis-stimulating agents (ESAs) (eg, epoetin alfa [Epogen, Procrit], darbepoetin alfa [Aranesp]) are
used to treat chronic anemia related to chronic kidney disease (CKD) or bone marrow suppression
(chemotherapy).  ESAs are synthetic forms of the naturally occurring hormone erythropoietin, which
stimulates the production of red blood cells (ie, erythropoiesis) by the bone marrow.  Erythropoietin is
normally produced by the kidneys, so clients with CKD may develop anemia due to decreased erythropoietin
production.
ESAs are administered on a regular schedule (eg, once weekly) and may be self-administered subcutaneously at
home or IV during hemodialysis as needed.  A total of 2-3 months is required to reach a target hemoglobin
level.  ESAs are held if hemoglobin exceeds 11 g/dL (110 g/L) due to an increased risk of thrombotic events
(eg, myocardial infarction, stroke) (Option 3).  ESAs increase blood pressure and are contraindicated in clients
with uncontrolled hypertension.

(Options 1, 2, and 4)  Creatinine and blood urea nitrogen are expected to be elevated in clients with CKD. 
Laboratory results for creatinine, platelet, and blood urea nitrogen do not indicate the effectiveness of epoetin
alfa.

Educational objective:
Erythropoiesis-stimulating agents (eg, epoetin alfa, darbepoetin alfa) treat chronic anemia by stimulating red
blood cell production.  Hemoglobin level should not exceed 11 g/dL (110 g/L) due to an increased risk of
thrombotic events (eg, myocardial infarction, stroke).

If heart sounds are difficult to auscultate, the nurse can ask the client to either sit up and lean forward (best
for aortic and pulmonic areas) or lie down on the left side (best for the mitral area).  These positions move the
heart closer to the chest wall.

(Option 2)  Exhaling and holding the breath aid in auscultating heart sounds that are difficult to hear.  Inhaling
introduces more air into the lung, which may muffle heart sounds.

(Option 3)  Raising the arm over the head helps to flatten the breast tissue, making any lumps more
pronounced.  It does not help with auscultation.

(Option 4)  The diaphragm of the stethoscope is best for auscultating higher-pitched sounds (S1 and S2).  The
bell is better for lower-pitched sounds (eg, extra heart sounds [S3, S4]).

Educational objective:
The nurse should assess normal (S1, S2) and any adventitious (eg, S3, S4, murmurs) heart sounds at each of the
valve areas (aortic, pulmonic, tricuspid, mitral) and Erb's point in a Z pattern.  Listening while the client is
sitting up and leaning forward or lying on the left side improves auscultation.

Enteral tube feeding is the preferred route for providing nutrition to clients who cannot intake oral nutrition
(eg, dysphagia, prolonged intubation).  The nurse assesses tube feeding tolerance by monitoring for signs of
potential complications:

 Diarrhea: May occur if tube feeding formula is too concentrated (hyperosmolar) or administered too
rapidly (Option 2).  The presence of hyperosmolar feedings in the intestines causes the osmotic
movement of water into the intestinal lumen, resulting in diarrhea.
 Fluid overload: Manifested as rapid weight gain and peripheral edema; due to excess water flushes or
too-dilute (hypo-osmolar) formula (Option 3)
 Nausea and vomiting: Due to delayed gastric emptying or rapid administration (Option 1)

(Option 4)  A low volume of gastric residuals suggests that the client has adequate gastric emptying and is
tolerating tube feedings.

(Option 5)  A serum blood glucose of 110 mg/dL (6.1 mmol/L) is an expected finding in a client receiving
enteral nutrition.  The recommended target serum glucose range for clients receiving nutritional support is 140-
180 mg/dL (7.8-10.0 mmol/L).  Hyperglycemia (>180 mg/dL [10.0 mmol/L]) would require intervention (eg,
sliding scale insulin, alternate tube feeding formula).
Educational objective:
The nurse assesses for complications of tube feeding by monitoring bowel movements (eg, diarrhea), fluid
status (eg, rapid weight gain), and the presence of delayed gastric emptying (eg, high gastric residuals,
nausea/vomiting).

Varicose veins are tortuous, distended veins.  The condition is usually accompanied by discomfort (eg, heavy
feeling, aching, pruritus).  It occurs frequently in clients with a family history, certain chronic conditions (eg,
heart disease, obesity), or jobs that require prolonged sitting, standing or heavy lifting.  Over time, increased
pressure on the legs leads to weakening and dilation of healthy veins.

To promote improved venous return and prevent further complications (eg, rupture of the vein, venous stasis
ulcer), the nurse should encourage the 3 Es:  elevation (Option 4), exercise (Option 5), and elastic
compression hose (Option 2).

Elevating the legs uses gravity to promote venous return.  Low-impact exercise (eg, walking, swimming) helps
muscles pump blood back to the heart more effectively.  Graduated elastic compression hose help maintain
venous tone, preventing the backward flow of blood.

In addition, weight reduction improves mobility and places less body pressure on the legs, improving venous
return (Option 1).

(Option 3)  Prolonged sitting may be as damaging for varicose veins as prolonged standing, as venous return is
decreased due to lack of movement.  The client who is required to sit at a desk should flex the ankles
periodically, elevate the legs when able, and get up and walk whenever possible.

Educational objective:
Varicose veins are tortuous, distended veins that occur frequently in clients with a family history, certain
chronic conditions, or jobs requiring prolonged sitting, standing, or heavy lifting.  To improve venous return,
the client should follow the 3 Es:  elevation, exercise, and elastic compression hose, and should maintain an
appropriate weight.

Acute pericarditis is inflammation of the pericardium, the double-walled sac that surrounds the heart. 
Pericardial inflammation can progress to pericardial effusion (fluid buildup between the pericardial layers)
followed by cardiac tamponade.  In cardiac tamponade, a potentially fatal complication, the heart is
compressed by fluid buildup and prevented from pumping effectively (decreased atrioventricular filling and
contractility).  The nurse should monitor the client with pericarditis for signs of cardiac tamponade (eg,
narrowed pulse pressure, jugular venous distension, muffled heart tones) (Option 2).

(Option 1)  Pericarditis is characterized by chest and neck pain that increases with inspiration and coughing. 
Supine positioning also aggravates pain, but sitting up and leaning forward relieves it.  These are expected
findings.

(Option 3)  A hallmark sign of pericarditis is pericardial friction rub, a creaky, grating sound on cardiac
auscultation caused by friction between inflamed pericardial layers.  This is an expected finding.

(Option 4)  ST-segment elevation across multiple ECG leads (rather than in specific leads, as seen with
myocardial infarction) is common in pericarditis.  The nurse does not need to report this expected finding.

Educational objective:
Clinical manifestations of pericarditis include pericardial friction rub; chest and neck pain worse with supine
position, inspiration, and coughing; and ST-segment elevation.  The nurse should monitor the client with
pericarditis for signs of cardiac tamponade (eg, narrowed pulse pressure, jugular venous distension, muffled
heart tones).

Fetal tachycardia is defined as a baseline heart rate above 160 beats per minute.  Tachycardia can be an early
indicator of fetal hypoxia and acidosis.  Other common causes include infection, maternal fever, maternal
dehydration, maternal hypotension, and drug side effects.  Maternal temperature should be taken to assess for
fever, and blood pressure should be assessed to rule out hypotension (Options 1 and 5).  Certain medications
can lead to fetal tachycardia (eg, terbutaline, bronchodilators, decongestants), and the nurse should review the
medication administration record to determine whether potential causative medications were administered
recently (Option 4).

(Option 2)  Reassessment at a later time results in an inappropriate delay in care.  Fetal tachycardia is
considered an abnormal finding requiring timely intervention.

(Option 3)  Maternal dehydration, hypovolemia, and subsequent hypotension are possible causes of fetal
tachycardia.  The IV fluid rate may need to be increased (not decreased) if hypotension is identified.

Educational objective:
Fetal tachycardia can be an indicator of early fetal hypoxia and acidosis.  Potential etiologies should be
addressed to identify the cause and determine appropriate interventions.  Other common causes include
maternal fever, maternal dehydration, maternal hypotension, and drug side effects.

Torsades de pointes (ie, "twisting of the points") is a polymorphic ventricular tachycardia characterized by
QRS complexes that change size and shape in a characteristic twisting pattern.  Torsades de pointes may be
the result of a prolonged QT interval (normal 0.34-0.43 sec or less than half the RR interval), usually due to
medications or electrolyte imbalances.

The nurse should review the client's medical record for any condition or medication that may prolong the QT
interval and precipitate another episode of torsades de pointes, including:

 Antiarrhythmics (eg, sotalol, amiodarone, ibutilide, dofetilide) (Option 1)


 Macrolide antibiotics (eg, erythromycin, azithromycin)
 Electrolyte abnormalities:  Hypokalemia and hypomagnesemia

Clients on anticoagulants (eg, apixaban, rivaroxaban, edoxaban) or with a bleeding disorder (eg, hemophilia)
should not undergo acupuncture (thin needle insertion) without first consulting their health care provider as it
increases the risk for bleeding (Option 1).

(Option 2)  The use of garlic supplements has been shown to slightly reduce cholesterol levels, and there is no
known interaction with statin medications (eg, atorvastatin).

Sleep hygiene refers to a group of practices that promote regular, restful sleep.  Components of sleep hygiene
include:

 Keep a consistent sleep schedule, even on nonworking days.


 Avoid daytime naps; if needed, they should be brief (less than 20-30 min).
 Go to bed early enough to get at least 7 hours of sleep.
 Get out of bed if sleep does not occur after 20 minutes (Option 3).
 Engage in regular, relaxing activities (eg, warm bath, reading) before bedtime.
 Sleep in a cool, quiet, dark room (Option 4).
 Avoid brain-stimulating substances or activities at least 4 hours before bedtime (eg, caffeine, computer
usage, exercise) (Option 1).
 Reduce fluid intake before bedtime to prevent nocturia.
 Use sleeping pills cautiously or avoid them as they may affect daytime functioning, and rebound
insomnia may occur on withdrawal.

(Option 2)  Alcohol may help to induce sleepiness at bedtime; however, it can cause early awakening and
fragmented sleep.

(Option 5)  Clients should not watch television at bedtime as it is stimulating and produces ambient light.  The
bed should be used only for sleep and sex.

Educational objective:
Practices to promote sleep hygiene include establishing a regular sleep routine; sleeping in a cool, quiet,
comfortable environment; avoiding caffeine and alcohol and reducing fluid intake before bedtime; and limiting
stimulating light (eg, computer, television) before bedtime.  A client unable to sleep after 20 minutes should get
out of bed.

Priority assessment of the pregnant client during the third trimester emphasizes early identification of
complications and differentiation from normal physiologic changes and discomforts of pregnancy.

Leukorrhea (ie, whitish, mucoid vaginal discharge) increases dramatically during pregnancy.  However,
copious, clear vaginal discharge that is thin or watery could indicate leaking of amniotic fluid, especially in
the third trimester.  Assessing for rupture of membranes should be a priority (Option 1).

Frequent urination throughout pregnancy may be caused by uterine enlargement, hormonal influences,
increased blood volume, and changes in glomerular filtration rate.  However, dysuria, cloudy urine, or flank
pain should not be present and may indicate infection (Option 2).

Headache, right upper quadrant pain, and visual changes could indicate preeclampsia and need priority
assessment (Option 4).

(Option 3)  During pregnancy, total blood volume increases by 30%-50%, and estrogen contributes to increased
vascularity of the mucous membranes.  These physiologic changes cause capillary engorgement and hyperemia,
which may lead to nasal stuffiness and a sense of fullness in the ears.

(Option 5)  Colostrum, a precursor to breast milk, is yellow-orange in color and may be seen leaking from the
nipples during the second and third trimesters.

Educational objective:
Common physiologic changes in pregnancy include nasal stuffiness, ear fullness, and colostrum secretion. 
Findings warranting further investigation and evaluation include dysuria; flank pain; headache with blurred
vision; and copious amounts of watery, clear vaginal discharge.

An asthma exacerbation occurs when a stimulus (eg, allergen [smoke, stress, illness) triggers acute
inflammation and bronchoconstriction, causing shortness of breath and wheezing.

Fear of not being able to breathe can cause severe anxiety, which may further exacerbate asthma symptoms
(eg, hyperventilation).  The nurse should reduce environmental stimuli and encourage coping mechanisms
(eg, breathing exercises).  If family members' actions are overstimulating an anxious client, the nurse should
provide education about the importance of a calm environment and attempt to redirect the family member to
assist in the client's care as able (eg, holding the client's hand, instructing on breathing techniques) (Option 3).

disseminated herpes zoster. Herpes zoster (shingles) is a viral infection that occurs as a result of the reactivation of the
varicella-zoster virus in individuals who previously had chickenpox.  Airborne and contact precautions are necessary
when lesions are fluid-filled and disseminated as the fluid from the blisters carries a high viral load.- N95

Permanent pacemakers consist of a generator that is implanted subcutaneously in the chest and lead wires that
terminate in the heart.  Infection of the incision site can easily travel down the pacemaker lead wires into the heart,
causing myocarditis and/or endocarditis.  Infection may disrupt pacemaker function and result in failure to sense or
pace that causes decreased cardiac output and life-threatening arrhythmias.  Signs and symptoms of pacemaker
malfunction (eg, hypotension, bradycardia, dizziness) and infection (eg, redness, fever, purulent drainage) should be
assessed immediately (Option 3).

Singed facial hair may indicate a smoke inhalation injury from close proximity to a fire.  Inhaled smoke
causes injury to the airway and lung tissue, which may result in life-threatening pulmonary or tracheal
edema.  The nurse should assess for any indications of inhalation injury (eg, singed facial hair, hoarse voice,
burned clothing around the chest and neck) and prepare for emergent intubation to protect the airway.

(Option 1)  A child who has a headache after a head trauma may have a concussion and will require a
neurologic examination.  This client is alert enough to verbalize pain and will likely be discharged with
instructions to the parents to observe for changes in neurologic status.

(Option 2)  A client who is unable to raise an injured arm above the head may have a rotator cuff tear.  This
client will require joint rest, application of ice or heat, and analgesia with nonsteroidal anti-inflammatory drugs
but is not the priority.

(Option 4)  An elderly client with severe diarrhea is at risk for dehydration.  The client may require IV fluids
and further workup, but this treatment may be delayed until treatment of higher-priority clients.

Educational objective:
Smoke inhalation injuries may cause life-threatening pulmonary or tracheal edema.  The nurse should assess for
any indications of inhalation injury (eg, singed facial hair, hoarse voice, burned clothing around the chest and
neck) and prepare for emergent intubation to protect the airway.

Infants and toddlers should sit in a rear-facing car seat until 2 years of age or the child exceeds the car seat's maximum
allowable height/weight.

Head trauma may cause a rise in intracranial pressure (ICP).  Elevated ICP is a life-threatening condition that
decreases cerebral blood flow, risking brain ischemia, infarction, or herniation.  Late manifestations of
severely increased ICP, impending brainstem herniation, and possible brain death include changes in vital
signs (eg, Cushing triad) and pupil response.

Cushing triad is a neurologic emergency characterized by bradycardia, irregular respirations, and


hypertension with a widening pulse pressure (Option 1).  The body attempts to increase perfusion to the
brain by increasing blood pressure, which causes systolic hypertension with a widening pulse pressure.  Other
late signs may include motor abnormalities (eg, posturing) and cranial nerve deficits (eg, loss of cough and gag
reflexes).

(Option 2)  Although irregular respirations are present in Cushing triad, increased ICP causes hyperthermia
due to injury to the hypothalamus.
(Option 3)  Fixed, dilated pupils are seen late in neurologic decline due to increased ICP compressing
oculomotor nerve III.  Otorrhea (cerebrospinal fluid leaking from the ear) indicates possible skull fracture but is
not directly related to increased ICP.

Bullying, harassment, or disparaging behaviors between coworkers in the workplace, or horizontal


violence, is an issue in nursing that results in job dissatisfaction, decreased productivity, anxiety, burnout, and
decreased quality of care.  Nurses must be aware of horizontal violence and work to eliminate disparaging
practices in the work environment.

Horizontal violence behaviors may be verbal (eg, belittling, gossiping, using a hostile tone of voice) and/or
nonverbal (eg, ignoring, eye-rolling).  Anonymous reporting (eg, suggestion boxes, tip lines) may encourage
staff participation and identify aggressors (Option 2).  Current trends and behaviors related to
professionalism should be analyzed to work toward identifying solutions.  All staff should receive education
regarding the importance of a positive work culture and the facility's expectations for behavior and
consequences of horizontal violence (Options 1 and 3).  Addressing factors that increase stress in the
workplace (eg, increased workload, high client-staff ratios, unfair scheduling) can reduce horizontal violence.

The vast majority of horizontal violence is in response to perceived or actual inequality in workplace dynamics
(eg, manager gives raises to personal friends) or feeling professionally threatened (eg, new nurses being hired
preferentially over experienced nurses).

(Option 4)  Although gender and age diversity play a role in overall collaboration, diversification is not known
to decrease workplace hostility and horizontal violence.

Educational objective:
Horizontal violence (eg, harassment, bullying, disparaging behaviors) is common in the health care environment
and often occurs between nurses.  Nurses should analyze their workplace culture, create anonymous reporting
systems, and provide staff education to remediate factors contributing to horizontal violence.

Barium, a contrast medium, aids in the visualization of tumors, obstructions, polyps, and other abnormalities. 
Barium can be administered rectally (ie, enema) to fill the lower gastrointestinal tract and facilitate clear x-ray
images of the large intestine.  After the procedure, clients should increase fluid intake and consume foods high
in fiber to facilitate removal of the barium.

Retention of barium can cause fecal impaction or bowel obstruction, resulting in severe complications such
as bowel perforation and peritonitis.  Reports of constipation should be assessed further as intervention (eg,
laxatives, suppositories) may be needed to help evacuate the barium and prevent complications.

(Option 2)  A bowel obstruction causes gas and fluid to accumulate, stretching the lumen.  Flatulence is an
expected finding after the obstruction is cleared and the bowels are decompressed.

(Option 3)  Clients with irritable bowel syndrome may experience diarrhea, constipation, or both.  This client
may need fluid and electrolyte replacement from loss through stools, but it is not a priority.

(Option 4)  Bloody diarrhea is an expected finding in clients with ulcerative colitis; fewer than 4 stools a day
indicates mild disease.  Although this client should be assessed, the client with a potential bowel obstruction
related to barium is higher priority.

Mechanical ventilator alarms (eg, high- or low-pressure limit) alert the nurse to potential problems caused by
a change in the client's condition, a problem with the artificial airway (eg, endotracheal or tracheostomy tube),
and/or a problem with the ventilator.  Any condition that abnormally decreases resistance in the tubing or
airway can trigger the low-pressure limit alarm.  When this alarm sounds, the nurse should assess for
conditions that decrease airway resistance, causing air to flow freely without the expected resistance caused
by physiologic lung compliance, such as:

 Loss of airway:  Intentional or accidental removal of endotracheal tube (Option 1)


 Tubing disconnection:  Disconnection of tubing at either client-end or ventilator-end of circuit (Option
5)
 Cuff leak in artificial airway:  Deflation or displacement of the endotracheal or tracheostomy tube cuff
(Option 3)

(Options 2 and 4)  Any condition that increases resistance can trigger the ventilator high-pressure limit alarm
(eg, bronchospasm, secretions, pneumothorax, kinked tubing, coughing or gagging).

Educational objective:
When the mechanical ventilator low-pressure limit alarm sounds, the nurse should assess for conditions that
decrease resistance in the airway or tubing (eg, loss of airway, tubing disconnection, cuff leak in artificial
airway).

At birth, a newborn will have enough iron (received during the last trimester of pregnancy) to last until
approximately age 4 months.  After this age, formula-fed infants usually receive adequate iron intake from
iron-fortified formula, whereas breastfed infants may require supplementation until they begin eating iron-
rich foods.

Oral iron supplements should be given on an empty stomach between meals for best absorption (Option 5). 
If gastric irritation occurs, iron may be given with meals; however, this will decrease absorption.  If the child is
old enough, the supplements should be offered with citrus fruit juice as vitamin C will increase absorption. 
Milk products and antacids also decrease the absorption of oral iron and should be avoided for 2 hours
following administration (Option 3).

After a colostomy, the stoma should be beefy red and edematous but will begin to shrink over the course of a
few days as inflammation subsides (Option 5).  There should be no mucocutaneous separation (eg,
separation of the stoma from the abdominal wall), unusual bleeding (eg, moderate to large amounts of blood in
the ostomy pouch), or signs of inadequate circulation, including stoma ischemia (eg, pale, dusky) and necrosis
(eg, dark red, purple, black).  Appliances should be resized during the first several weeks to ensure proper fit,
preventing skin breakdown (eg, excoriation) due to stool coming into contact with the skin (Option 1).

Within 24 hours of surgery, the client should demonstrate signs of returning gastrointestinal motility,
including resolution of nausea, active bowel sounds, and flatus (Option 3).  Nausea and absent bowel sounds
may indicate postoperative ileus and should be reported to the health care provider (Option 4).

Clients should change the pouch according to the manufacturer's instructions (every 5-10 days) and when the
skin surrounding the stoma is irritated (eg, burning).  The nurse should also assess the client with a new ostomy
for body image disturbance and ineffective coping (eg, client unwilling to care for the ostomy) (Option 2).

Educational objective:
Careful assessment of clients with new ostomies should include the stoma site (eg, perfusion, approximation to
the skin), gastrointestinal function (eg, bowel sounds, flatus, stool), and self-care and body image.  Appliances
must be properly fitted to prevent skin breakdown (eg, excoriation).

Failure to sense occurs when a pacemaker fails to recognize the intrinsic electrical activity of the heart and fires at
inappropriate times during the cardiac cycle.  If the pacemaker fires during myocardial repolarization (eg, within the T
wave), life-threatening arrhythmias (eg, ventricular tachycardia, ventricular fibrillation) may occur (Option 3).  Failure to
sense is treated by increasing or lowering the sensitivity setting until the pacer senses appropriately.

In a cerebrovascular accident (CVA), blood flow in the brain is compromised due to either bleeding or
occlusion of a blood vessel.  After a CVA, mental status may continue to decline, especially within the first 24-
48 hours.  The nurse should immediately notify the health care provider of decorticate (flexion) posturing,
which is characterized by arms rigidly flexed at the elbow, hands raised to the chest, and legs extended (Option
4).  This posturing suggests worsening cerebral impairment (eg, increased intracranial pressure) that may be
reversible with proper interventions.

(Option 1)  Hemiparesis and hemiplegia are expected findings in a client with a CVA.

(Option 2)  An impaired gag reflex may indicate an impaired airway; however, a patent airway has already
been established in an intubated and mechanically ventilated client.  An impaired gag reflex can occur after
intubation if sedation was required for placement of the endotracheal tube.

(Option 3)  Bladder dysfunction (eg, retention, incontinence) can be an expected finding in a client who has
experienced a CVA, depending on the level of neurological impairment.

Educational objective:
Mental status may continue to decline in the first 24-48 hours following a cerebrovascular accident.  Decorticate
(flexion) posturing (arms rigidly flexed at the elbow, hands raised to the chest, and legs extended) is a sign of
worsening neurological function that should be reported immediately to the health care provider.

Tumor lysis syndrome (TLS), an oncologic emergency, occurs when cancer treatment successfully kills
cancer cells, resulting in release of intracellular components (eg, potassium, phosphate, nucleic acids).  Clients
with TLS develop significant imbalances of serum electrolytes and metabolites.

TLS may result in the following life-threatening conditions:

 Hyperkalemia (eg, >5.0 mEq/L [5.0 mmol/L]) may progress to lethal dysrhythmias (eg, ventricular
fibrillation)
 Large amounts of nucleic acids (normally converted to uric acid and excreted by the kidneys)
overwhelm the kidneys, leading to hyperuricemia and acute kidney injury from uric acid crystal
formation
 Hyperphosphatemia (eg, >4.4 mg/dL [>1.42 mmol/L]) can cause acute kidney injury and
dysrhythmias

TLS is best prevented by aggressive hydration and prophylactic allopurinol for hyperuricemia.

(Option 1)  Superior vena cava (SVC) syndrome (eg, dyspnea, facial and upper body edema, engorged upper
body blood vessels) is an oncologic emergency caused by SVC compression (eg, tumors).  Radiation or
chemotherapy may relieve SVC compression.

(Option 2)  Syndrome of inappropriate antidiuretic hormone (SIADH) (eg, edema, dilutional hyponatremia)
often occurs with central nervous system involvement of cancer.  Effective cancer treatment corrects SIADH.

(Option 4)  Neutropenia from cancer treatment puts clients at high risk for infection and sepsis (eg,
hypotension, elevated lactic acid).  Fluid resuscitation should be promptly initiated, blood cultures drawn, and
IV antibiotics started.
Educational objective:
Tumor lysis syndrome, an oncologic emergency, occurs when cancer treatment successfully kills cancer cells,
resulting in release of intracellular components into the bloodstream (eg, hyperkalemia, hyperphosphatemia). 
Released nucleic acids degrade into uric acid and cause hyperuricemia, leading to possible kidney injury.

Rabies is caused by a virus present in the saliva of an infected animal (eg, bat, dog) and can be transmitted to a
human through a bite, a scratch, or mucous membrane contact.  Rabies affects the central nervous system, and
can cause viral encephalitis with eventual death from cardiovascular and respiratory collapse if untreated. 
Clients with actual or suspected rabies exposure should receive rabies postexposure prophylaxis, including:

 Immediate wound care:  Aggressive scrubbing and cleaning with povidone-iodine solution or soap
and water to decrease the viral count and the rabies transmission risk (Option 4)
 Administration of tetanus toxoid vaccine (if the client is not current with immunizations):  Tetanus is
associated with a high mortality rate and can be transmitted through animal bites (Option 2)
 Administration of the human rabies immunoglobulin:  Provides passive immunity and is injected into
the proximal wound area (Option 3)
 Administration of the human rabies vaccine:  Provides active immunity and is administered
intramuscularly on the day of exposure and again on days 3, 7, and 14 postexposure (Option 1)

Educational objective:
The rabies virus affects the central nervous system and is transmitted by the saliva of infected animals (eg, bat,
dog) usually via a bite or scratch.  Postexposure prophylaxis includes immediate wound care with povidone-
iodine or soap and water; vaccines for tetanus and rabies, or rabies immunoglobulin, may be given afterward.

Septic arthritis (infectious arthritis) is acute joint inflammation due to an infection.  Pathogens may enter the
joint from the bloodstream (eg, current infection elsewhere in the body), direct penetration (eg, intraarticular
injection), or infected adjacent tissue (eg, osteomyelitis).  Septic arthritis can lead to irreversible joint damage
if not treated promptly.

Clinical manifestations of septic arthritis include:

 Severe, pulsating pain, usually with sudden onset and exacerbated by movement
 Erythema, warmth, effusion (ie, excess synovial fluid)
 Limited range of motion due to swelling in the joint
 Systemic immune response to the joint infection (eg, fever) (may not be present in elderly or
immunocompromised clients)

The goal of treatment is to limit joint destruction and promote pain relief.  Management may include aspirating
synovial fluid; immobilizing the joint; restricting weight bearing; and administering antibiotics, analgesics, and
antipyretics.

(Option 5)  Numbness in the lower extremity related to spinal nerve compression can be associated with
arthritic diseases (eg, spinal stenosis) but is not characteristic of septic arthritis.

Educational objective:
Septic arthritis can lead to irreversible joint damage if not treated promptly.  Characteristic manifestations
include severe pain of sudden onset, erythema, warmth, swelling, limited range of motion, and fever.

Verapamil is a calcium channel blocker sometimes used for the prevention of migraines.  Calcium channel
blockers may decrease neurovascular inflammation, thereby reducing the occurrence of migraines.  Because
verapamil affects the cardiac system, the pulse rate should be checked prior to administration due to possible
bradycardia (Option 2).  The medication should be held, and the client's health care provider contacted, if the
heart rate is <60/min.  Clients should also have periodic blood pressure evaluations to ensure that hypotension
is not occurring.

Intake of grapefruit (including grapefruit juice) should be avoided, as it can increase serum levels of verapamil
by reducing hepatic clearance of the drug (Option 1).  Increasing fluids and fiber helps to prevent constipation,
a common side effect of most calcium channel blockers, particularly verapamil (Option 4).

(Option 3)  Verapamil is taken daily to prevent migraines and does not provide relief during an acute
migraine episode.  The most common treatment for acute migraine symptoms is administration of nonsteroidal
anti-inflammatory drugs (eg, naproxen) or triptan medications (eg, sumatriptan).

Educational objective:
Clients taking verapamil, a calcium channel blocker, for the prevention of migraines should take the medication
daily, check their pulse prior to administration, avoid grapefruit, and increase their intake of fluids and fiber to
prevent constipation.

Cardiac glycosides (digoxin [Lanoxin]) improve cardiac output and efficiency in clients with heart failure by
increasing cardiac contractility and decreasing heart rate.  A client experiencing weight gain and orthopnea
is likely experiencing a heart failure exacerbation.  The nurse should assess the client's entire medication
regimen to determine why digoxin may not be working effectively (eg, possible drug interactions).

Sucralfate (Carafate) is used to coat and protect the mucosal lining in clients with ulcers; therefore, drug
absorption will be altered.  Sucralfate should be taken at least 2 hours after digoxin administration, as taking
these medications at the same time can result in decreased digoxin absorption (Option 3).

(Option 1)  Exercise is an important part of maintaining cardiac health.  A client with heart failure should
exercise as tolerated with frequent periods of rest.  However, this is not the priority assessment for a client with
a heart failure exacerbation.

(Option 2)  Checking the heart rate prior to taking digoxin is recommended to assess for signs of digoxin
toxicity (eg, bradycardia).  However, this is not the priority teaching for a client with a heart failure
exacerbation.

(Option 4)  Information about the client's last visit to the primary care clinic might be helpful but would not be
beneficial to the assessment of symptoms.

Educational objective:
Cardiac glycosides (digoxin [Lanoxin]) improve cardiac output and efficiency in clients with heart failure. 
Sucralfate (Carafate) taken at the same time as digoxin can decrease absorption of the latter, thereby increasing
symptoms of heart failure.  Clients should take sucralfate at least 2 hours after digoxin.

Cyclosporine is an immunosuppressant prescribed to manage rheumatoid arthritis (RA) and psoriasis, and to
prevent transplant rejection.  This medication inhibits the normal immune response by interfering with T cell
response, which slows the progression of certain autoimmune diseases.  Clients taking cyclosporine have an
increased risk for infection and are instructed to avoid large crowds (eg, concerts, movie theaters) and known
sick contacts (Option 1).  It can take 1-2 months for the full effect of therapy and relief of symptoms from
autoimmune disease (eg, joint stiffness in RA, psoriasis symptoms) to occur.

This medication is for long-term use, and it is therefore important to monitor clients for adverse effects.  The
incidence of secondary malignancies (eg, skin cancer, lymphoma) is increased in these clients.
(Option 2)  Cyclosporine can cause increased hair growth (ie, hirsutism).  Waxing and hair removal creams are
safe for use.  Gingival hyperplasia is also common and clients require education on proper oral care.

(Option 3)  Cyclosporine is associated with hypertension and nephrotoxicity.

(Option 4)  Grapefruit juice can increase serum levels of cyclosporine and is avoided during therapy.

Educational objective:
Cyclosporine is an immunosuppressant.  Clients are at increased risk for infection and secondary cancers (eg,
skin cancer, lymphoma).  Consuming grapefruit increases the risk for drug toxicity.  Gingival hyperplasia,
hirsutism, uncontrolled hypertension, and kidney toxicity are other common adverse effects.

Stevens-Johnson syndrome is an immune-mediated reaction triggered by certain classes of medications (eg,


sulfonamide antibiotics, allopurinol, anticonvulsants [eg, phenytoin]).  Initial symptoms are nonspecific and
flu-like (eg, fever, fatigue) and are followed by blistered lesions and skin detachment on the face, trunk, and
palms (Option 4).

Stevens-Johnson syndrome can cause fatal complications (eg, sepsis, multiple organ dysfunction) and requires
immediate hospitalization and follow-up by the health care provider.  Treatment includes prompt
discontinuation of the causative medication and initiation of supportive care (eg, fluids, nutrition, wound care).

(Option 1)  Capsaicin cream (Zostrix) is a topical analgesic.  Capsaicin, a component of hot peppers (eg,
cayenne, jalapeño), can cause a burning sensation; therefore, clients are taught to wash their hands after
application to avoid getting the cream in their eyes.

(Option 2)  Carbidopa-levodopa (Sinemet) is an anti-Parkinson medication.  Orthostatic hypotension is a


common side effect.

(Option 3)  Hydroxyzine (Atarax) is a commonly used, first-generation antihistamine similar to


diphenhydramine (Benadryl) and chlorpheniramine.  Anticholinergic side effects (eg, urinary retention, dry
mouth, constipation, blurred vision) are common.  This client needs to be called second.

Educational objective:
Stevens-Johnson syndrome is an immune-mediated reaction triggered by certain classes of drugs (eg,
sulfonamide antibiotics, allopurinol, anticonvulsants).  It is characterized by blistered lesions on the face, trunk,
and palms and may be fatal if left untreated.

Ectopic pregnancy can be life-threatening if diagnosis and treatment are delayed.  A growing embryo
implanting anywhere outside the uterus (eg, fallopian tube, abdominal cavity) results in an ectopic pregnancy. 
Rupture of an ectopic pregnancy can occur if the embryo outgrows its environment, resulting in hemorrhage. 
Any woman with amenorrhea (ie, delayed or absent menstrual period), pelvic or abdominal pain, and/or
subsequent vaginal bleeding/spotting should be evaluated promptly for the possibility of ectopic pregnancy.

(Option 1)  Severe, intractable nausea, and vomiting can result in dehydration and electrolyte imbalances (ie,
hyperemesis gravidarum).  Although this client should be assessed for hyperemesis gravidarum, the condition is
not immediately life-threatening.

(Option 2)  Irregular Braxton Hicks contractions (ie, false labor) are not uncommon for a client at 37 weeks
gestation (ie, term gestation).  These contractions are more common in multiparous women or women with an
overdistended uterus (eg, twin gestation, polyhydramnios).  True labor is indicated by cervical change resulting
from regular contractions over time.
(Option 4)  Quickening, or a woman's first perception of fetal movement, usually occurs at 18-20 weeks
gestation in primigravida clients.  However, this timeframe may be influenced by increased maternal BMI or an
anteriorly placed placenta.

Educational objective:
Ectopic pregnancy can result in rupture and life-threatening hemorrhage if not promptly identified.  Any woman
with amenorrhea, pelvic or abdominal pain, and/or subsequent vaginal bleeding/spotting should be evaluated
promptly for the possibility of ectopic pregnancy.

Atypical presentation of a myocardial infarction involves associated symptoms (eg, sweating, nausea, dyspnea) with no
chest pain.  Women, older adults, and clients with diabetes or neuropathy are more likely to have an atypical
presentation.

Rapid fluid infusions can cause hypervolemia as excess fluid accumulates within the extracellular space,
especially in clients with heart failure or kidney disease.  The infusion should be stopped and the client
assessed for pulmonary edema (eg, dyspnea, lung crackles) and other signs of fluid overload (eg, peripheral
edema, jugular venous distension) (Option 2).  If signs are present, the health care provider should be notified
as fluid overload can cause respiratory and cardiovascular compromise.

(Option 1)  Itching is common post surgery from side effects of narcotics (generalized) and the wound healing
process (localized).  Generalized itching may indicate an allergic drug reaction and should be assessed.  This
client with localized itching may need ice packs or diphenhydramine but would not be a priority.  This client
would be seen second.

(Option 3)  An angry client waiting for discharge should be addressed before the situation escalates.  However,
clients with physiologic needs take priority.

(Option 4)  The nurse should notify the health care provider of this client's potassium level and prescribed IV
fluids.  If IV fluids are still required, the prescription should be changed to normal saline (without potassium
chloride).  This client is not a priority as the potassium level is still within normal limits (3.5-5.0 mEq/L [3.5-5.0
mmol/L]).  The value is critical if potassium level is >5.5 mEq/L (5.5 mmol/L).

Educational objective:
Fluid overload (ie, dyspnea, lung crackles, peripheral edema, jugular venous distension) can occur with
increased infusion rates and should be addressed promptly to prevent respiratory or cardiovascular compromise.

Frostbite occurs when vasoconstriction restricts blood flow, intracellular fluid freezes, and cell membranes
rupture; tissue may appear pale, waxy, blue, or mottled.  Clients with peripheral vascular problems (eg,
advanced age, diabetes, smoking) are at a higher risk for developing frostbite.  A warm water bath (eg, 98.6-
102.2 F [37-39 C]) is administered to thaw and reestablish as much circulation to viable tissue as possible. 
Subsequent edema and/or superficial blistering may develop as the damaged tissue is rewarmed.  Blisters are
opened to reduce pressure and sterile dressings are applied.  The client will need analgesics as the rewarming
process is very painful.

(Options 1 and 3)  Manual friction (eg, massage, ambulation) applied to tissues affected by frostbite is
contraindicated as it may further damage the tissues.

(Option 2)  Comfort care (eg, warm blankets) may be provided after emergent interventions to salvage the
client's lower extremities.  However, the nurse should consult with the health care provider before providing
food or drink to the client.
Educational objective:
Tissue damaged by frostbite may appear pale, waxy, blue, or mottled due to frozen intracellular fluid.  Affected
extremities are thawed in a warm water bath (98.6-102.2 F [37-39 C]), and analgesics are administered.  Manual
friction (eg, massage, ambulation) is contraindicated as it may further damage the tissue.

This child age <1 with fever, lethargy, and vomiting likely has sepsis or meningitis.  Clients with suspected
meningitis need to be seen immediately and require close monitoring (eg, level of consciousness, vital signs),
isolation, spinal fluid cultures, and antibiotics (Option 1).

Signs of meningitis in an infant include:

 Fever or hypothermia
 Poor feeding, vomiting
 Altered level of consciousness (eg, restlessness, irritability, lethargy)
 Increased intracranial pressure (bulging fontanelle [late sign], opisthotonic positioning [arching of the
back with hyperextension of the neck])

(Option 2)  Occasional barking cough without stridor is a sign of mild viral laryngotracheobronchitis (ie,
croup).  This client is stable and lower priority than a client with meningitis; however, the nurse should monitor
for signs of deterioration (eg, stridor, lethargy, tachypnea) and have oxygen and emergency respiratory
equipment available.

(Option 3)  Generalized edema and cola- or tea-colored urine (hematuria) indicates acute postinfectious
glomerulonephritis, which occurs after infection with group A beta-hemolytic Streptococcus and is not
emergent.  Treatment is supportive (eg, bed rest, sodium restriction, diuretics) and may be delayed.

(Option 4)  Painful urination could signify a urinary tract infection and has less risk of becoming emergent than
the other clients' conditions.  The teenager's withdrawn behavior warrants further assessment (eg, for suspected
abuse or depression), but this can be delayed.

Educational objective:
Bacterial meningitis is inflammation of the meninges, which causes cerebral edema and may be fatal.  Clients
with fever and signs of increased intracranial pressure need close monitoring, isolation, spinal fluid cultures,
and antibiotics.

Each of the 12 cranial nerves has a sensory function, a motor function, or both.  Cranial nerve II, the optic
nerve, allows the brain to sense what the eye sees.  The client with an impaired cranial nerve II may have
altered visual acuity or visual fields.  To ensure that the client understands interventions, the nurse should
verbally explain all procedures in detail (Option 4).

(Option 1)  Chewing is affected by cranial nerve V (trigeminal).

(Option 2)  Although raising the head of the client's bed to prevent aspiration is an appropriate nursing action, it
is not necessary with an alteration with cranial nerve II.  Swallowing is affected by cranial nerves IX
(glossopharyngeal) and X (vagus).

(Option 3)  Using a pen and paper to ask the client questions would be appropriate if cranial nerve VIII
(vestibulocochlear or acoustic) were impaired, causing a hearing deficit.
Educational objective:
Clients with impairment of cranial nerve II have altered visual acuity or visual fields.  To ensure that the client
understands interventions, the nurse should verbally explain all procedures in detail.

A craniotomy involves incision into the cranium and is indicated for elevated intracranial pressure or removal
of tumors, blood, or abscesses.  Postoperatively, clients are at risk for developing a cerebrospinal fluid (CSF)
leak from an intraoperative dural injury, which increases the risk for meningitis.

Excessive drainage from a craniotomy incision (eg, saturated dressing, >50 mL per shift into the drain) or from
the nose or ear suggests a possible CSF leak requiring immediate notification of the health care provider (HCP)
(Option 3).  Interventions focus on decreasing strain on the dural tear to encourage closure and include
bedrest, lumbar drain placement, and surgical intervention.

(Option 1)  The incision should not be re-dressed until the HCP can evaluate the wound and drainage.

(Option 2)  The nurse should mark the drainage edges at least once per shift for comparison.  However, a
saturated dressing indicates a possible CSF leak.

(Option 4)  Specific client positioning postcraniotomy is usually prescribed by the HCP.  The head of the bed is
elevated approximately 30 degrees to facilitate venous drainage from the head and prevent increased
intracranial pressure; if the bed is flat, the client should not lie on the operative side.  Repositioning may be
indicated but is not the most appropriate action at this time.

Educational objective:
Clients postcraniotomy are at risk for developing a cerebrospinal fluid leak.  Incisional edema or clear drainage
from the incision, nose, or ear is reported immediately to the health care provider for evaluation and
intervention to decrease the risk of meningitis.

Severe anxiety impairs the ability to attend to stimuli in the environment other than the anxiety-producing
event or factor.  Physiological responses to anxiety include hyperventilation, palpitations, shortness of breath,
and diaphoresis.  Behavioral responses (eg, fixation on specific details, pacing) serve as coping mechanisms to
manage anxiety.  Unrelieved anxiety may become severe and escalate to a panic attack.

The nurse should ensure safety and support the client with severe anxiety to prevent injury and escalation. 
The client is unable to attend to details, so the nurse should communicate in a calm, accepting manner; answer
questions directly; and use simple statements (Option 4).

(Option 1)  Clients with severe anxiety are unable to attend to their own safety or needs.  The nurse should not
leave the client alone at this time.

(Option 2)  Anxiety is "contagious," so nurses' anxiety can exacerbate clients' anxiety (ie, reciprocal anxiety). 
Nurses should accept their own feelings about the situation and remain calm without transferring their
frustrations to clients.

(Option 3)  Clients should be allowed to engage in coping behaviors (eg, repetitive acts, pacing) during
episodes of severe anxiety as long as they do not risk harm.  These behaviors relieve tension and prevent
escalation.

Educational objective:
When caring for a client experiencing severe anxiety, nurses should provide a calm presence, reassure clients of
safety, use simple statements, and answer questions directly.  Nurses should not leave the client alone, interfere
with coping behaviors, or transfer their own frustrations to the client.
Selective serotonin reuptake inhibitors (SSRIs) (eg, escitalopram [Lexapro]), which are commonly
prescribed antidepressants, usually take 1-4 weeks from the first dose to improve depression symptoms.  If a
client experiences no improvement after 2 months, re-evaluation is necessary (Option 2).

Noncompliance is common with SSRIs due to intolerance of side effects (eg, nausea, weight gain, sexual
dysfunction).  The nurse should first assess if the client is taking the medication as prescribed (Option 3). 
Clients may require education on symptom management (eg, taking with food for nausea, nutritional education
to manage weight).  If the client is compliant but the medication has not relieved depressive symptoms, the
health care provider may change the prescribed dose or medication.

(Option 1)  Assessing for stressors is important when a client is taking an SSRI.  However, this can be asked
later in the assessment as the priority is to determine compliance.

(Option 4)  The nurse should assess the client's medication compliance before discussing a change in the
prescription with the health care provider.

Educational objective:
Selective serotonin reuptake inhibitors (eg, escitalopram, sertraline, fluoxetine) take about 1-4 weeks from the
first dose to improve depression symptoms.  If the medication is ineffective, the nurse should determine client
compliance prior to notifying the health care provider.

Chronic pancreatitis is an inflammatory disease that causes the tissue of the pancreas to become fibrotic,
impairing pancreatic endocrine and exocrine functions.  Chronic pancreatitis is most commonly caused by
alcohol abuse, but may also result from biliary tract disease (eg, cholelithiasis), autoimmune processes, or cystic
fibrosis.  Lifestyle modification is a key component of treatment and includes cessation of alcohol and smoking
as well as dietary modifications.

Clients with pancreatitis often cannot secrete lipase in sufficient quantities to digest consumed fats.  Therefore,
clients should follow a low-fat diet, with the degree of fat restriction based on the severity of disease.  Due to
lack of endogenous lipase, oral supplementation of pancreatic enzymes is often required before meals.  To
avoid exacerbating gastric discomfort, the client should avoid spicy and gas-forming foods.

Low-fat food choices include lean meats (eg, fish, chicken), nonfat dairy products, vegetables/fruits prepared
without added fat, and low-fat carbohydrates (eg, green peas) (Option 1).

(Options 2, 3, and 4)  Dairy-containing foods (eg, macaroni and cheese, creamed soup), baked goods (eg,
biscuits, cornbread, croissants), and some meats (eg, roast beef) are high in fat.  Refried beans also contribute to
gas formation and promote bloating.  Salsas and spicy foods should be avoided.

Educational objective:
Clients with pancreatitis often cannot secrete lipase in sufficient quantities to digest consumed fats, requiring
the client to follow a low-fat diet.  Low-fat food choices include lean meats, non-fat dairy products,
vegetables/fruits prepared without added fat, and low-fat carbohydrates.

Rituximab (Rituxan) is a monoclonal antibody (end in -mab) that affects the lymphocytes.  It is commonly
prescribed to treat certain forms of cancer (eg, lymphoma) and autoimmune diseases (eg, lupus).  Like many
monoclonal antibodies, rituximab can produce a powerful immune response (eg, bronchospasm, dyspnea,
tachypnea, hypotension, angioedema) (Option 3).  The nurse should closely monitor the client during and after
the infusion.  If life-threatening symptoms develop, the nurse should stop the infusion and immediately notify
the health care provider.  The symptoms will be treated (eg, corticosteroids) and, when resolved, the infusion is
usually restarted at a slower rate.
(Options 1, 2, and 4)  In many clients, monoclonal antibody therapies, like many oncology pharmaceuticals,
invoke flu-like responses (eg, fever, chills, diarrhea, nausea, vomiting).  Clients are often pretreated with
acetaminophen and diphenhydramine in anticipation of these reactions.  Clients' symptoms are treated as needed
(eg, antiemetics, antidiarrheals).

Educational objective:
Rituximab can produce a powerful immune response (eg, bronchospasm, dyspnea, tachypnea, hypotension,
angioedema).  Clients should be closely monitored during and after the infusion.

Pressure injuries are staged from 1 to 4 to classify the degree of tissue damage and determine the most
appropriate and effective wound treatments.

 Stage 1:  Intact skin with nonblanchable redness


 Stage 2:  Partial-thickness skin loss (abrasion, blister, or shallow crater) involving the dermis or
epidermis; the wound bed is red or pink and may be shiny or dry
 Stage 3:  Full-thickness skin loss; subcutaneous fat is visible but not tendon, muscle, or bone; tunneling
may be present
 Stage 4:  Full-thickness skin loss with visible tendon, muscle, or bone; slough or eschar (scabbing, dead
tissue) may be present; undermining and tunneling may be present
 Pressure injuries are described as "unstageable" if the base is covered by necrotic tissue or eschar

In ventricular trigeminy, premature ventricular contractions (PVCs) occur every third heartbeat. 
Myocardial injury (eg, myocardial infarction) predisposes the client to ectopy (eg, PVCs), which increases
the client's risk for lethal dysrhythmias (eg, ventricular tachycardia).  PVCs are caused and/or exacerbated
by hypoxia, electrolyte imbalances, emotional stress, stimulants, fever, and exercise.

 This client's morning laboratory results show hypokalemia (potassium <3.5 mEq/L [3.5 mmol/L]);
therefore, the priority is treatment of the underlying cause of the ectopy by administering the prescribed
potassium replacement (Option 1).  Health care providers (HCPs) often prescribe electrolyte
replacement algorithms to clients at risk for electrolyte imbalances (eg, myocardial injury, receiving
diuretics) unless a contraindication exists (eg, serum creatinine >1.5 mg/dL [133 µmol/L], anuric,
weight <99.2 lb [45 kg]).

 (Option 2)  Amiodarone is an antiarrhythmic medication with a long duration of action (ie, 13-107
days).  An acute drop in the drug level is not likely the cause of the ectopy.  The nurse should administer
amiodarone as prescribed after initiating the potassium replacement.

 (Option 3)  Correcting the electrolyte imbalance should resolve the client's ectopy, preventing the need
for defibrillation.

 (Option 4)  The HCP should be notified; however, the nurse should first assess the client and initiate
potassium replacement.

 Educational objective:
Myocardial injury can predispose a client to premature ventricular contractions (PVCs), placing the
client at risk for lethal dysrhythmias (eg, ventricular tachycardia).  PVCs are caused and/or exacerbated
by hypoxia, electrolyte imbalances, emotional stress, stimulants, fever, and exercise.  Treatment of the
underlying cause is the priority.

A client who undergoes percutaneous coronary intervention (PCI) and intracoronary stent placement
using the femoral approach is at increased risk for retroperitoneal hemorrhage.  Administration of
antithrombotic drugs before, during, and after PCI can exacerbate potentially life-threatening bleeding
from the femoral artery.  Hypotension, back pain, flank ecchymosis (Grey-Turner sign), hematoma
formation, and diminished distal pulses can be early signs of bleeding into the retroperitoneal space
and require immediate intervention/evaluation (eg, notify health care provider, serial CBCs, abdominal
CT).

 (Option 1)  During a heart transplant, the donor heart is cut off from the autonomic nervous system
(denervated), which alters the heart rate during rest and exercise after the transplant.  The transplanted
heart is expected to be tachycardic (eg, 90-110/min).

 (Option 3)  Infective endocarditis is often associated with cardiac valve disease and requires long-term
antibiotic therapy (4-6 weeks).  Characteristic manifestations include fever, myalgia, chills, joint pain,
anorexia, and petechiae.

 (Option 4)  Some clients notice swelling in the leg used for donor venous graft (interruption of blood
flow).  Elevating the leg and wearing compression stockings can help decrease symptoms.

Educational objective:
Percutaneous coronary intervention via the femoral approach places the client at increased risk for
retroperitoneal hemorrhage, which is exacerbated by anticoagulants.  Back pain, hypotension, flank
ecchymosis (Grey-Turner sign), hematoma formation, and diminished distal pulses can be early signs of
bleeding into the retroperitoneal space and require immediate intervention.

Sickle cell crisis occurs when inadequate oxygenation or hydration exacerbates sickling and causes red blood
cells (RBCs) to clump together in the capillaries (vasoocclusion).  Vasoocclusion causes severe ischemic pain,
hypoxia, and possible organ dysfunction if left untreated.

Adequate oxygenation and hydration may reverse the acute sickling response.  In the sickled state, RBCs
cannot carry enough oxygen from the lungs to the tissues, even with supplemental oxygen.  The priority
intervention is the administration of IV fluids to reduce blood viscosity and restore perfusion to the areas
previously affected by vasoocclusion (Option 1).  Only after IV rehydration reverses vasoocclusion can
nonsickled RBCs effectively carry supplemental oxygen to the tissues (Option 2).

(Option 3)  Bed rest improves oxygen use and reduces energy consumption during sickle cell crisis but does not
directly resolve vasoocclusion.

(Option 4)  Blood transfusions provide the client with nonsickled RBCs, increasing the oxygen-carrying
capacity of the blood.  However, this therapy is generally reserved for clients with sickle cell disease who do
not respond to rehydration with IV fluids.

Educational objective:
Sickle cell crisis results from vasoocclusion of sickled red blood cells in the microcirculation, resulting in
severe ischemic pain.  The administration of IV fluids reduces blood viscosity and restores perfusion to the
areas previously affected by vasoocclusion.

Bruising behind the ear (eg, Battle sign) following head trauma may indicate a basilar skull fracture (Option
3).  Because of their close proximity to the brainstem, basilar skull fractures pose a risk of serious intracranial
injury, which is the most common cause of traumatic death in children.  Other signs include blood behind the
tympanic membrane, periorbital hematomas (ie, raccoon eyes), and cerebrospinal fluid leakage from the nose or
ears.  This client requires cervical spine immobilization, close neurologic monitoring, and support of airway,
breathing, and circulation.
(Option 1)  Vomiting with oral intake may indicate infection (viral or bacterial).  Most serious abdominal
processes (eg, obstruction, intussusception, appendicitis) also have abdominal pain.  This client may require IV
fluids and antiemetics but is not a priority.

(Option 2)  Tympanostomy tubes placed for recurrent otitis media may fall out of the ear canal.  This child
should be evaluated for the presence of infection and the need for possible tube reinsertion, but this is a
common occurrence and can wait to be addressed.

(Option 4)  Increasing pain at skeletal pin sites after surgical fracture repair may indicate infection or
displacement of the pins.  Pin displacement may compromise blood flow to the leg.  The nurse should assess the
neurovascular status of the limb, but this does not take priority over a basilar skull fracture.

Educational objective:
A client with signs of basilar skull fracture (eg, periorbital hematomas, bruising behind the ear, leakage of
cerebrospinal fluid) requires immediate cervical spine immobilization, neurologic assessment, and airway,
breathing, and circulation support.

Total parenteral nutrition (TPN) may be prescribed for clients with dysfunction of the gastrointestinal tract
(eg, short bowel).  Glucose (dextrose) is a primary component of TPN solutions; therefore, the nurse should
monitor blood glucose and assess for symptoms of hyperglycemia (eg, polydipsia, polyuria, headaches,
blurred vision).  A urine output of 4,800 mL during a shift may indicate hyperglycemia (Option 3). 
Symptomatic clients should be assessed and treated immediately as hyperglycemia can lead to seizures, coma,
or death.

(Option 1)  A client experiencing a transient ischemic attack has stroke-like symptoms that later resolve.  Such
clients are often placed on prophylactic antithrombotic treatment (eg, aspirin, clopidogrel) to prevent future
strokes.  This client requires scheduled medication but is not an immediate priority.

(Option 2)  A client with a subdural hemorrhage should be frequently assessed for neurological changes and
early symptoms of increased intracranial pressure (eg, headache, nausea).  A verbal request for breakfast
indicates that the client is stable and therefore does not require immediate attention.

(Option 4)  Tissue plasminogen activator is administered to clients with ischemic strokes to dissolve clots in the
brain.  A Glasgow Coma Scale change from 9 to 13 demonstrates improving neurological status.

Educational objective:
Total parenteral nutrition (intravenous nutrition) is high in glucose, which places the client at risk for
hyperglycemia.  Signs and symptoms of hyperglycemia include polydipsia, polyuria, headaches, and blurred
vision.

During mass casualty events, the goal is to do the greatest good for the greatest number of people.  Clients
are triaged using various systems (eg, Simple Triage and Rapid Transport/Treatment [START]; Sort, Assess,
Life-saving interventions, Treatment/Transport [SALT]) and placed into 4 categories:

 Immediate (red tag):  Life-threatening injuries with good prognoses once treated (eg, airway
obstruction, open fractures, second- or third-degree burns covering 15%-40% body surface area)
 Delayed (yellow tag):  Injuries requiring treatment within hours (eg, stable abdominal wounds, soft
tissue injuries)
 Minimal (green tag):  Injuries requiring treatment within a few days (eg, minor burns or fractures, small
lacerations)
 Expectant (black tag):  Extensive injuries, poor prognosis regardless of treatment
Immediate medical care of the client with an open fracture would likely result in a good prognosis (Option
3).

(Option 1)  Delayed treatment is appropriate for the client with partial-thickness burns to a small portion of the
body (eg, hands).

(Option 2)  Depending on its size and depth, a laceration would require minimal or delayed treatment.

(Option 4)  A large, open head wound with a Glasgow Coma Scale score of 3 has a poor prognosis regardless
of treatment; death is expected.

Educational objective:
Disaster triage ranks the likelihood of survival with treatment, not necessarily the severity of injury.  Clients
with significant alteration in airway, breathing, and circulation who are likely to survive with timely
intervention are the first priority.

Urosepsis is a type of bloodstream infection that originates from the urinary tract.  The initial treatment of
sepsis focuses on the management or prevention of septic shock, mainly by administering boluses of isotonic IV
fluids (fluid resuscitation) and IV broad-spectrum antibiotics (Option 1).  Blood and urine cultures are
obtained, ideally before the first dose of antibiotics (Option 4).  Continuous vital sign and cardiac telemetry
monitoring are initiated as hyperkalemia and sepsis cause cardiovascular disturbances (eg, dysrhythmias and
hypotension, respectively) (Option 3).

(Option 2)  Chronic kidney disease impairs the excretion of excess potassium and can potentiate hyperkalemia,
which can lead to life-threatening arrhythmias (eg, ventricular fibrillation).  ACE inhibitors (eg, lisinopril,
ramipril) or angiotensin II receptor blockers (eg, valsartan, losartan, irbesartan) can be used to manage
hypertension secondary to renal disease; however, these drugs can worsen hyperkalemia.

(Option 5)  Clients with chronic kidney disease and elevated creatinine are unable to excrete the iodinated
contrast administered for CT scans.  Toxic effects from the contrast can occur; therefore, this prescription
should be clarified before the scan.

Educational objective:
A uroseptic client with chronic kidney disease and hyperkalemia should be treated with IV isotonic fluid
boluses and IV broad-spectrum antibiotics.  Blood and urine cultures should be obtained.  The nurse would
question the administration of iodinated contrast to a client with significant kidney disease.  ACE inhibitors and
angiotensin II receptor blockers should be avoided in clients with hyperkalemia.

Preeclampsia is a multisystem disorder that occurs after the 20th week of pregnancy.  Preeclampsia is defined
as new-onset hypertension (≥140/90 mm Hg) after 20 weeks gestation plus proteinuria or signs of end-organ
damage.  Proteinuria diagnostic of preeclampsia is defined as ≥300 mg/24 hr (0.3 g/day) urine collection,
protein/creatinine ratio ≥0.3, or dipstick of ≥1+ (Option 2).

Cerebral symptoms, such as headache and visual changes, are potential manifestations of preeclampsia and
reasons for concern (Option 4).  With severely elevated blood pressure, there is risk of cerebral edema,
hemorrhage, and stroke.  It is important to prevent the progression of preeclampsia to eclampsia (new-onset
seizure in the presence of preeclampsia) to tonic-clonic seizures.

Edema occurs in many normal pregnant women as well as those with preeclampsia.  Although edema is not part
of the diagnostic criteria for preeclampsia, it is a common manifestation of the disease process (Option 1).

(Option 3)  Frequent urination is common in pregnancy and is not associated with preeclampsia.
(Option 5)  Pregnancy causes an intravascular volume expansion larger than the rise in red blood cells,
resulting in hemodilution.  Inadequate iron stores or intake can also play a role.  Anemia is defined as
hemoglobin <11 g/dL (110 g/L) in the first and third trimesters and <10.5 g/dL (105 g/L) in the second
trimester.  Anemia is not a specific indicator or criterion for preeclampsia.

Educational objective:
Preeclampsia is defined as new-onset hypertension (≥140/90 mm Hg) plus proteinuria and/or signs of end-organ
damage after 20 weeks gestation.  Although edema is not a diagnostic criterion for preeclampsia, it is a common
manifestation of the disease process.

Amphotericin B is an antifungal medication used to treat systemic fungal infections.  It is commonly


associated with severe adverse effects, including hypotension, fever, chills, and nephrotoxicity.  Due to the
similarity between the adverse effects of amphotericin B and the symptoms of a blood transfusion reaction,
the nurse's best action is to complete the blood transfusion and allow one hour of observation before initiating
amphotericin B (Option 4).  This allows the nurse to distinguish between transfusion-related reactions and
adverse effects from amphotericin B.

(Options 1 and 2)  Although starting a peripheral IV line or using the unused lumen of the peripherally inserted
central catheter line would prevent mixing the drug with the blood products, it would not help distinguish the
onset of potentially fatal sequela from either component.

(Option 3)  Transfusions should not be interrupted after initiation except in cases of transfusion-related
reactions or fluid overload.  In addition, interrupting and restarting transfusions increases the risk for
infection.  Blood products should be transfused within 4 hours of removal from refrigeration.

Educational objective:
At least one hour should be allowed between completion of a blood transfusion and administration of
amphotericin B.  The adverse effects of a transfusion-related reaction and an adverse reaction from
amphotericin B are similar, and the observation time allows the nurse to distinguish the triggering event if
symptoms develop.

Clients who have had a stroke can experience cognitive dysfunction (eg, confusion), neglect on one side,
deficits in spatial perception, and paralysis (hemiplegia), all of which increase the risk for injury (eg, falls). 
The nurse should see this client first to ensure that safety precautions (eg, bed alarm, nonslip socks) are in
place to prevent injury (Option 1).  The least restrictive measures (eg, orienting the client, room assignment
near the nursing station) should be implemented prior to initiating restraints.

(Option 2)  Albuterol is a beta-adrenergic agonist that stimulates the sympathetic nervous system to cause
bronchodilation and relieve asthma symptoms.  Expected side effects include tachycardia, palpitations, and
tremors.

(Option 3)  The nurse should administer the prescribed dose of insulin for a client with a blood glucose of 290
mg/dL (16.1 mmol/L).  However, a client with a high risk of injury is the priority.

(Option 4)  Anesthetics and sedating analgesics may exacerbate symptoms of obstructive sleep apnea. 
Although oxygen saturation is below 95%, the client is stable on room air.

Educational objective:
Clients who have suffered a stroke can experience cognitive dysfunction (eg, confusion) and hemiplegia,
resulting in a high risk for injury (eg, falls).  The nurse should ensure that safety precautions (eg, bed alarm,
nonslip socks) are in place.
Foods highest & lowest in potassium
Foods highest in potassium Foods lowest in potassium
Cruciferous vegetables, legumes, potatoes, green Asparagus, green beans, corn, cucumbers, okra,
Vegetables
leafy vegetables, tomatoes, raw carrots onions, green peas, green peppers
Dried fruits, melons, avocados, oranges, mangos, Apple products, grapes, grapefruits, peaches,
Fruits
nectarines, strawberries, bananas pears, pineapples
Dairy Milk & milk products Cottage cheese
Protein Most fish & shellfish, most beef products, pork Chicken, turkey, shrimp, tuna, eggs
Grains Whole grains, granola, bran Breads & pastas made with white flour, rice

Clients with end-stage renal disease are unable to excrete potassium; therefore, the nurse should teach them to
choose foods low in potassium to maintain normal serum potassium levels (3.5-5.0 mEq/L [3.5-5.0 mmol/L]). 
Grilled chicken sandwich on white bread and applesauce are low in potassium (Option 2).

(Options 1, 3, and 4)  Legumes (eg, black beans), tomatoes, melons (eg, cantaloupe), beef, whole grains,
carrots, chocolate, fish (eg, salmon), potatoes, and strawberries are all high in potassium.

Educational objective:
The kidneys' ability to excrete potassium is compromised in clients with end-stage renal disease.  These clients
should avoid foods high in potassium (eg, green leafy and cruciferous vegetables; legumes; melons; bananas;
strawberries; milk and milk products; most beef, fish, and shellfish; and whole grains) to maintain normal serum
potassium levels (3.5-5.0 mEq/L [3.5-5.0 mmol/L]).

Thrombolytic therapy (tissue plasminogen activator [tPA]) is used to dissolve blood clots and restore
perfusion in clients with ischemic stroke.  The nurse assesses for contraindications to tPA due to the risk of
hemorrhage.

tPA must be administered within a 3- to 4.5-hour window from onset of symptoms for full effectiveness
(Option 4).

(Option 1)  Recent major surgery (within the past 14 days) is a contraindication as tPA dissolves all clots in the
body and may therefore disrupt the surgical site.  Gallbladder surgery 2 months prior is outside the window of
contraindication.

(Option 2)  Loss of the gag reflex and other major functions would most likely make the client a candidate for
thrombolytics due to proof of deficits from stroke.

(Option 3)  Clients with thrombocytopenia (platelet count <100,000/mm3 [100 × 109/L]) and/or coagulation
disorders should not receive tPA as these conditions further increase the risk for bleeding.  Other
contraindications include hemorrhagic stroke, uncontrolled hypertension, and stroke or head trauma within the
past 3 months.

Educational objective:
Thrombolytic therapy (tissue plasminogen activator) is used to treat and dissolve blood clots in clients with
ischemic stroke.  This therapy should be administered within 3-4.5 hours of when the client was last "normal." 
Contraindications include thrombocytopenia (platelet count <100,000/mm3 [100 × 109/L]), coagulation
disorders, and major surgery within the past 14 days.

Lithium is a mood stabilizer most often used to treat bipolar affective disorders.  Lithium has a very narrow
therapeutic index (0.6-1.2 mEq/L [0.6-1.2 mmol/L]) that should be closely monitored; it also has the potential
for many drug interactions.  Several medications can cause increased lithium levels, including thiazide
diuretics (eg, hydrochlorothiazide), nonsteroidal anti-inflammatory drugs, and antidepressants.  Thiazide
diuretics have demonstrated the greatest potential to increase lithium concentrations, with a possible 25%-
40% increase in concentrations (Option 2).  The nurse should assess the client for signs and symptoms of
lithium toxicity and report the findings to the health care provider.

The client should bring all prescription and over-the-counter medications to each office visit to perform a
medication reconciliation and reduce the risks associated with polypharmacy.

(Options 1 and 4)  Acetaminophen (Tylenol) and sulfa antibiotics (eg, sulfadiazine) do not interact with lithium
and are safe for the client to take.

(Option 3)  Lithium levels are not affected by antidiabetic medications such as metformin; however, lithium
has been known to increase serum glucose levels.  This may necessitate a dose adjustment of the antidiabetic
medications.  The client's blood glucose should be monitored, but this effect is not the most concerning at this
time.

Educational objective:
Lithium is a mood stabilizer used to treat bipolar affective disorders.  Medications such as thiazide diuretics,
nonsteroidal anti-inflammatory drugs, and antidepressants can cause elevated lithium levels, which increases the
risk of toxicity.

Nitroglycerin is an antianginal medication that causes potent vasodilation (coronary and systemic) and is
used in the treatment of acute coronary syndrome (eg, unstable angina, myocardial infarction).  IV nitroglycerin
administration requires continuous cardiac monitoring and frequent blood pressure assessment (eg, every 15
minutes for the first hour).  Headache is an expected side effect from vasodilation of cranial vessels and should
decrease with continuing nitroglycerin therapy.  As long as the client does not have severe hypotension (eg,
systemic blood pressure <90 mm Hg), the finding can be documented and the headache can be treated with
aspirin or acetaminophen.

(Options 1 and 4)  If the headache becomes severe or persistent despite acetaminophen, the health care
provider (HCP) may temporarily decrease the dose.  The nurse should not arbitrarily stop the infusion or
decrease the rate.

(Option 3)  Nitroglycerin therapy can precipitate increased intracranial pressure (ICP).  Additional signs of
increased ICP (eg, decreased level of consciousness, vomiting, Cushing triad) should be reported to the HCP.  A
CT scan of the head is not indicated at this time.

Educational objective:
Nitroglycerin is a potent vasodilator used to treat acute coronary syndrome.  Headache is an expected side effect
that decreases with continued therapy and can be treated with aspirin or acetaminophen.  A headache is not
concerning unless it is severe or persistent and/or accompanied by severe hypotension or signs of increased
intracranial pressure.

Sterile procedures (eg, urinary catheter insertion, tracheostomy suctioning) require the nurse to wear sterile
gloves.  Insertion of catheters into the body can introduce infectious microorganisms; correctly donning sterile
gloves decreases infection risk.  The nondominant hand is used to apply the glove of the dominant hand
first.  Using the dominant hand to apply the second glove improves dexterity and decreases the risk of
contamination of the gloved dominant hand.

The correct procedure for donning sterile gloves includes:

1. Perform hand hygiene, and remove the outer glove package (Option 2).
2. Place the inner glove package on a clean, dry surface.  Open the inner package by carefully folding back
the edges (Option 1).
3. Use the nondominant hand to grasp the cuff of the dominant glove.  Touch only the inside surface of the
glove (Option 6).
4. Pull on the dominant hand glove (Option 4).
5. Place the fingers of the gloved dominant hand under the cuff of the nondominant glove.  Keep the
gloved thumb pulled away to prevent contact with the skin of the nondominant hand (Option 3).
6. Pull on the nondominant hand glove (Option 5).

Educational objective:
Correctly donning sterile gloves decreases infection risk by preventing the introduction of infectious
microorganisms into the body.  The nurse first applies the glove of the dominant hand, and then uses the
dominant hand to pull on the glove of the nondominant hand.

An ST-segment elevation myocardial infarction (STEMI) occurs when at least one of the coronary arteries is
completely occluded.  The ST segment is the portion of the ECG between the QRS complex and the T wave. 
Prompt treatment (eg, percutaneous coronary intervention, thrombolytics) is needed to restore myocardial
oxygen supply and limit myocardial damage.

(Option 1)  Atrial fibrillation is characterized by an irregularly irregular rhythm with P waves replaced by
fibrillatory waves, resulting in ineffective atrial contraction.  Clients are at increased risk for clot formation
(long-term), which can cause a stroke or pulmonary embolism; however, signs of cardiac injury take priority.

(Option 2)  First-degree heart block is characterized by a prolonged PR interval.  Clients are usually
asymptomatic and do not require immediate assessment.

(Option 4)  Premature ventricular contractions are early contractions of the ventricles that originate from an
ectopic focus and result in a wide, distorted QRS complex.  They are usually not harmful and can occur as a
response to stimulants (eg, caffeine, nicotine, alcohol) or electrolyte imbalances.

Educational objective:
An ST-segment elevation myocardial infarction occurs when ≥1 of the coronary arteries that supply blood to the
myocardium are occluded; it is characterized by elevation of the ST segment (located between the QRS
complex and the T wave).  Prompt treatment (eg, percutaneous coronary intervention, thrombolytics) restores
myocardial oxygen and limits myocardial injury.

A low-pressure limit alarm on the ventilator is triggered when the amount of positive pressure necessary to
deliver a breath to the client is decreased.  A decrease in resistance to airflow occurs due to complications that
arise in the client (eg, loss of airway), artificial airway (eg, cuff leak), and/or ventilator system (eg, tubing
disconnect).  All of these conditions impair airway and ventilation; therefore, addressing this alarm is the
highest priority (Option 2). 

(Option 1)  Heparin is used to treat thromboembolic conditions (eg, pulmonary embolism); however, a brief
interruption is not life-threatening.  A distal occlusion alarm indicates that there is an obstruction to the flow of
medication that occurs distal to the infusion pump (eg, between the infusion pump and the client).  Causes
include infiltration, clotting, displaced IV catheter, kinked tubing, and/or client positioning.

(Option 3)  The client with a low respiratory rate alarm should be assessed second.  A low respiratory rate
alarm may be caused by client factors (eg, sedation, shallow breathing) or equipment error (eg, malposition of
ECG leads).
(Option 4)  An occlusion alarm on the enteral feeding pump requires intervention, however; this is not a
priority action.  Possible causes include kinked tubing, food or medication blockage, or client positioning.

Educational objective:
A low-pressure limit alarm on a ventilator indicates that decreased pressure is required to deliver a breath.  A
decrease in resistance to airflow occurs due to complications that arise in the client (eg, loss of airway), artificial
airway (eg, cuff leak), and/or ventilator system (eg, tubing disconnect).

Children often develop respiratory distress and bradycardia before going into cardiac arrest.  When the collapse
of a child with apnea and a pulse is witnessed, emergency services should be contacted and then rescue
breathing should be implemented.  If, after rescuing breathing is performed for 2 minutes, the pulse remains
≤60/min and there are signs of poor perfusion (eg, skin pallor), then compressions should be initiated.  In
infants and children (1 year to puberty), a heart rate ≤60/min with signs of poor perfusion is treated as
pulseless.

(Option 2)  If the heart rate increases to >60/min with signs of adequate perfusion, the nurse would continue
with only rescue breathing.  The correct rescue breathing rate for children is 1 breath every 3-5 seconds or 12-
20 breaths per minute.

(Option 3)  Abdominal thrusts are performed during the Heimlich maneuver to aid a choking victim.  They are
powerful upward squeezes to the victim's diaphragm to expel an object from the trachea.  There is no indication
that this child was choking.

(Option 4)  If the child was initially found to be in a witnessed cardiac arrest, the nurse would retrieve the
defibrillator prior to starting CPR.

Educational objective:
Infants and children with a heart rate ≤60/min with signs of poor perfusion are treated as pulseless.  If the child
is apneic with a pulse ≤60/min, rescue breaths should be implemented.  If pulse remains ≤60 with signs of
inadequate perfusion after rescue breaths, compressions should be initiated.

Venous thromboembolism (VTE) occurs when a thrombus (eg, deep vein thrombosis) forms and embolizes
into the bloodstream (eg, pulmonary embolism).  Hospitalized clients tend to have multiple risk factors for
VTE, including venous stasis from prolonged immobility and endothelial damage from surgeries or IV catheter
placement.

VTE prophylaxis should be implemented in all hospitalized clients.  Measures include:

 Administration of anticoagulants (eg, enoxaparin), usually prescribed in clients with a moderate or high
risk of VTE (eg, postsurgical) unless contraindicated (eg, active bleeding) (Option 1)
 Application of compression devices or antiembolism stockings to limit venous stasis (Option 2)
 Frequent ambulation, 4-6 times daily as tolerated, to improve circulation and promote venous return
(Option 4)
 Foot and leg exercises (eg, extend and flex the feet and knees) to promote venous return by activating
calf muscles (Option 5)

(Option 3)  Elevating the legs while in bed promotes venous return by gravity.  However, the nurse should
ensure that any pillows used to elevate the legs do not place pressure directly behind the knees, as pressure on
the posterior knees compresses leg veins.  Clients should also avoid crossing the legs to prevent pressure on the
back of the knees.
Educational objective:
Hospitalized clients have many risk factors for venous thromboembolism (VTE), including immobility and
damage to the endothelium from surgeries or IV catheters.  VTE prophylaxis measures include anticoagulation,
ambulation, leg exercises, compression devices, and prevention of pressure behind the knees (eg, crossing legs).

Clients with preeclampsia are at risk for developing preeclampsia-associated seizure activity (eg, eclampsia) as
a result of increased central nervous system irritability.  The presence of neurologic manifestations (eg,
hyperreflexia, clonus) may indicate worsening preeclampsia and can precede seizure activity (Option 4).  This
client is at the most immediate risk of harm and is the priority to report to the health care provider.

To assess for clonus, the nurse firmly dorsiflexes the foot with 1 hand while supporting the leg and ankle with
the other hand.  The abnormal finding of positive clonus is identified when rhythmic, jerking "beats" of the
foot are present as the foot is released and allowed to fall back into plantar flexion.

(Option 1)  Clients with gestational diabetes mellitus are more susceptible to infection (eg, urinary tract
infection, vaginal yeast infection).  Although the client's report of dysuria may indicate a urinary tract infection,
the assessment findings do not indicate immediate risk.

(Option 2)  Hyperemesis gravidarum usually affects clients in the first trimester and is characterized by severe
nausea and vomiting that can lead to dehydration, hypotension, electrolyte imbalances, and nutritional deficits. 
This client should be assessed for further symptoms of hypotension (eg, dizziness, blurry vision) before
notifying the health care provider.

(Option 3)  Oligohydramnios indicates low amniotic fluid, which may lead to umbilical cord compression and
fetal compromise.  However, a reactive nonstress test is a reassuring finding.

Educational objective:
Hyperreflexia and clonus are abnormal findings that may indicate worsening preeclampsia and impending
seizure activity.

Petechiae are reddish or purple pinpoints on the skin that occur due to bleeding from capillaries.  Petechiae
usually occur due to blood vessel injury or bleeding disorders (eg, thrombocytopenia, disseminated
intravascular coagulation).  Petechiae and similar skin conditions are often challenging to detect in dark-skinned
clients as dark pigmentation makes it difficult to assess skin color changes.  In dark-skinned clients, petechiae
can best be assessed in the conjunctivae of the eyes and the buccal mucosae.

(Option 2)  The nail bed of the finger is the best location to assess dark-skinned clients for cyanosis, a blue
discoloration that may occur with hypoxemia (ie, decreased blood oxygen).  Petechiae generally do not occur in
the nail bed.

(Option 3)  The palms of the hands and soles of the feet are ideal locations for assessing other skin color
changes that may occur in dark-skinned clients, such as jaundice (ie, yellowing of the skin due to increased
bilirubin in the blood).  However, these are not ideal locations to assess for petechiae in a dark-skinned client.

(Option 4)  Over the sacrum and behind the heels are common locations for pressure injury formation; skin
here typically appears dark, especially in dark-skinned clients.

Educational objective:
Skin assessment of dark-skinned clients can be challenging as dark pigmentation makes it difficult to detect
color changes.  To best assess for petechiae in a dark-skinned client, the nurse should observe the buccal
mucosae or conjunctivae.
Emphysema is characterized by loss of elasticity in the lungs due to permanently enlarged, "floppy" alveoli. 
The lungs become hyperinflated (eg, hyperresonance on percussion, prolonged expiration, barrel chest).  The
client has activity intolerance and anxiety due to inability to meet oxygen demands during exertion.

Arteriovenous (AV) graft placement involves surgical connection of an artery and a vein using a synthetic
material to graft a hemodialysis access site.  Postoperative infection of an AV graft may cause thrombosis,
graft failure, or systemic infection.  Fever in a postoperative client may indicate infection of the graft site,
which warrants immediate notification of the health care provider (HCP); this client may require antibiotics and
surgical removal of the graft (Option 4).

(Option 1)  A small amount of rectal bleeding and abdominal cramping is expected following a colonoscopy as
the bowel contracts to expel the air inserted during the procedure.  Following a colonoscopy, clients should
notify the HCP of severe abdominal pain, distension, and excessive bleeding, which may indicate bowel
perforation.

(Option 2)  Following surgery, constipation can occur due to decreased ambulation and narcotic pain
medications.  The client may require a stool softener to reduce straining.

(Option 3)  Anesthesia and opioid analgesics may cause postoperative urinary retention for up to 3 days
following surgeries, especially abdominal or pelvic surgeries.  This client should be instructed on measures to
aid voiding (eg, standing) and may need to come to the clinic for bladder ultrasound or straight catheterization.

Educational objective:
Postoperative infection of an arteriovenous graft may result in thrombosis, graft failure, or systemic infection. 
Clients with signs of infection (eg, low-grade fever) require immediate follow-up.

Potassium chloride (KCl) is commonly prescribed to correct or prevent hypokalemia.  The normal range for
serum potassium is 3.5-5.0 mEq/L (3.5-5.0 mmol/L).  Potassium is commonly lost through diarrhea, vomiting,
and diuretic use.

Clients receiving KCl IV should have periodic cardiac monitoring during therapy, as changes in potassium
levels can cause cardiac rhythm disturbances and rapid infusion can cause cardiac arrest (Option 4).

Potassium is a vesicant; therefore, the IV insertion site should be monitored frequently for extravasation to
prevent tissue necrosis (Option 2).  The maximum infusion rate of KCl through a peripheral line is 10
mEq/hr (10 mmol/hr) and the maximum concentration is 40 mEq/L (40 mmol/L) (Option 5).  Higher rates and
concentrations require a central venous catheter.

Renal function should be assessed as clients with impaired renal function are unable to excrete potassium
and other electrolytes effectively, potentially leading to toxicity.  To assess renal function, the nurse should
monitor renal function laboratory results (eg, blood urea nitrogen, creatinine) and urine output (Option 3).

(Option 1)  KCl is never administered by IV push or as a fluid bolus.  KCl is always diluted and given via
infusion pump.

Educational objective:
Potassium chloride is administered to correct or prevent hypokalemia.  When infused peripherally, the rate
should be ≤10 mEq/hr (10 mmol/hr), the IV site should be frequently assessed for extravasation, and renal
function (eg, urine output) should be monitored to prevent toxicity.  Periodic cardiac monitoring is
recommended during therapy.
(MI) A client comes to the emergency department with crushing substernal chest pain. The nurse should rapidly
assess (eg, vital signs, heart and lung sounds, pain) and intervene (eg, 2 large-bore IV lines, morphine, oxygen,
nitroglycerin, aspirin) for the client with acute chest pain.  Upright positioning improves ventilation and reduces
pressure on the heart.  The nurse should obtain a 12-lead ECG, chest x-ray, and blood work (eg, cardiac
markers), and place the client on continuous cardiac monitoring.

A pregnant nurse does not have a high risk for contracting methicillin-resistant Staphylococcus aureus (MRSA)
if appropriate infection precautions are used (Option 2).  The nurse should carefully follow contact precautions,
including wearing gloves and gown and performing strict hand hygiene.  Even if the pregnant nurse were to
contract MRSA, there are few known harmful effects to the fetus.

TORCH infections (Toxoplasmosis, Other [parvovirus B19/varicella-zoster virus], Rubella, Cytomegalovirus,


Herpes simplex virus) can cause fetal abnormalities, and clients with these infections should not be assigned
to pregnant health care workers.

(Option 1)  Clients receiving brachytherapy have radioactive implants placed in a body cavity.  To safely care
for these clients, nurses limit/cluster client time and keep a distance of at least 6 ft (1.8 m) unless wearing lead
shielding for direct care.  Pregnant health care workers should not care for these clients if possible as fetal
radiation exposure is teratogenic.

(Option 3)  Herpes zoster (ie, shingles, varicella-zoster virus infection) is a TORCH infection, and pregnant
health care workers should avoid caring for these clients.

(Option 4)  Zika virus may be transmitted through mosquito bites, infected body fluids, and sexual contact. 
Using standard precautions should provide protection; however, because Zika is known to cause birth defects,
pregnant health care workers should not care for a client exposed to it if at all possible.

Educational objective:
Pregnant health care workers can safely care for clients with methicillin-resistant Staphylococcus aureus by
using contact precautions.  Clients receiving radioactive therapy or with infections known to be teratogenic
should not be assigned to pregnant health care workers.

A contraction stress test (CST) evaluates fetal well-being under stress by identifying uteroplacental
insufficiency.  Uterine blood flow is decreased during uterine contractions, which stresses the fetus during
labor.  Contractions are stimulated using either oxytocin administration or nipple stimulation.  A fetal tracing
is evaluated until 3 uterine contractions, each lasting 40-60 seconds, are captured within 10 minutes.  A negative
test has no late or variable decelerations and is associated with good fetal outcomes (Option 2).  A positive test
includes late decelerations with ≥50% contractions.  A suspicious or equivocal test includes variable or
prolonged decelerations or late decelerations with <50% contractions.

A CST may be combined with a nonstress test to further evaluate fetal well-being.  A negative CST with a
reactive nonstress test provides strong reassurance of fetal well-being.

(Options 1 and 4)  These fetal tracings show late decelerations, indicating uteroplacental insufficiency.  They
indicate a positive test and an at-risk fetus.

(Option 3)  This fetal tracing has variable decelerations, indicating umbilical cord compression.  This is a
suspicious or equivocal test and follow-up testing is indicated.

Educational objective:
A contraction stress test evaluates fetal well-being during uterine contractions.  Contractions are stimulated by
oxytocin administration or nipple stimulation.  A negative test has no late or variable decelerations and is
associated with good fetal outcomes.  Late or variable decelerations indicate a need for further evaluation.

Respiratory syncytial virus (RSV) is a common cause of respiratory tract infection and bronchiolitis in infants
and children, occurring primarily during the winter.  It affects the ciliated cells of the respiratory tract, causing
bronchiolar swelling and excessive mucus production.  RSV in infants causes rhinorrhea, fever, cough,
lethargy, irritability, and poor feeding.  Severe RSV infection also causes tachypnea, dyspnea, and poor air
exchange.  Interventions are supportive, including:

 Providing supplemental oxygen and suctioning to support oxygen exchange and clear the airway
(Option 5)
 Elevating the head of the bed to improve diaphragmatic expansion and promote secretion clearance
(Option 3)
 Administering antipyretics to reduce fever and provide comfort (Option 1)
 Initiating IV fluids to correct dehydration due to fever, tachypnea, or poor oral intake (Option 2)

RSV is transmitted via direct contact with respiratory secretions.  Contact isolation is required, and droplet
precautions are added if within 3 ft (0.91 m) of the client, depending on facility policy (Option 4).

Palivizumab (Synagis), a monoclonal antibody, is administered intramuscularly once monthly during the
winter and spring to prevent RSV in children at high risk for contracting the infection (eg, prematurity,
chronic lung disease).

Educational objective:
Respiratory syncytial virus is a common cause of respiratory tract infection and bronchiolitis in infants and
children.  Nursing management includes respiratory support (ie, supplemental oxygen, elevation of the head of
the bed, airway suctioning) and administration of antipyretics and IV fluids.  Contact and droplet isolation are
required.

Polypharmacy and the physiologic changes associated with aging place older adults at an increased risk of
adverse drug events.  Decreased renal and hepatic function causes increased drug half-life and impaired drug
clearance, potentially resulting in toxicity and adverse events.

Clients may see different health care providers and receive multiple prescriptions for different health problems
(polypharmacy).  Clients should be encouraged to bring all medications (ie, prescription, over-the-counter
[OTC], herbal supplements) they take regularly and occasionally to each appointment so that potential drug
interactions can be evaluated (Option 3).

(Option 1)  Keeping a list of all medications and their dosages is a good idea to help organize the client's
medications.  However, the client may not remember all the medications and may not regularly update the list.

(Option 2)  Getting all medications from the same pharmacy is preferable.  The pharmacist can monitor for
possible interactions from prescription drugs, but many clients do not report the use of OTC medications or
herbal supplements to the pharmacist.

(Option 4)  A pill organizer helps the client remember to take medications at the appropriate times.  By
ensuring drugs are taken at prescribed intervals, some interactions can be avoided.  However, this may not take
into account herbal supplements and OTC drugs taken as needed.
Educational objective:
To reduce the risk for drug interactions, the nurse should encourage clients to bring all medications (ie,
prescription, over-the-counter, herbal supplements) to each appointment.

Percutaneous coronary intervention (PCI) with stent placement is performed to improve coronary artery
patency and increase cardiac perfusion.  After the use of contrast media to identify the occlusion, a balloon
and stent are inserted via a catheter through a large artery (eg, femoral artery) and threaded up to the blocked
coronary artery.  The balloon expands the stent against the arterial wall, compressing plaque and improving
patency.  The stent remains in the client after the balloon and catheter are removed.

Potential complications of PCI include thrombosis, stent occlusion, bleeding/hematoma, and limb ischemia. 
The nurse should immediately notify the health care provider of postprocedure angina, which indicates
possible thrombosis or stent occlusion; necessary prescriptions (eg, nitroglycerin, second PCI) should be
obtained and promptly initiated (Option 3).

(Option 1)  Neurovascular assessments (eg, circulation, sensation) of the affected extremity should be
compared with those of the unaffected extremity and the client's baseline; this client's 1+ pulses are not a
concern as they are bilateral, not unilateral.  Most clients with diabetes and coronary artery disease may also
have baseline peripheral artery disease.

(Option 2)  A small amount of bleeding/ecchymosis is expected at the access site due to anticoagulation
therapy, which is initiated prior to PCI.  The nurse should assess for signs of hematoma formation and
retroperitoneal hemorrhage (eg, ecchymosis, flank/back pain).

(Option 4)  Increased blood glucose needs to be treated but is not a priority over stent occlusion.

Educational objective:
Acute complications from stent placement include thrombosis, stent occlusion, bleeding/hemorrhage, and limb
ischemia.  The nurse should immediately notify the health care provider to evaluate postprocedure angina, back
pain (retroperitoneal hemorrhage), or poor pulses.

Dabigatran (Pradaxa) is a thrombin inhibitor anticoagulant often prescribed to prevent thrombotic events in
clients with atrial fibrillation, pulmonary embolism, and deep vein thrombosis.  Clients taking dabigatran are at
increased risk for bleeding and hemorrhage.  Clients with signs of abnormal bleeding (eg, bruising; blood in
the urine, sputum, vomitus, or stool; epistaxis; heavy menstrual bleeding [menorrhagia]) should be prioritized
as prompt intervention and treatment may be required.

(Option 1)  Missing a dose of phenytoin (Dilantin), an antiseizure medication, could precipitate seizure
activity.  The client should be instructed to take the medication as prescribed with a small sip of water;
however, this client does not take priority over one with active bleeding.

(Option 3)  Gastrointestinal upset is a common side effect of many antibiotics, including metronidazole
(Flagyl).  Abdominal discomfort may be relieved by taking the medication with food or a glass of milk.

(Option 4)  This client requires a refill of insulin to prevent hyperglycemic episodes but is not a priority over a
client with active bleeding.  Glargine is long-acting insulin that works for 24 hours.

Educational objective:
Dabigatran (Pradaxa) is an anticoagulant medication that works by inhibiting thrombin.  Clients taking
dabigatran are at increased risk for bleeding.  Signs of abnormal bleeding or hemorrhage should be addressed
promptly as acute medical intervention may be required.
Human papillomavirus (HPV) is a common sexually transmitted infection (STI) that is often asymptomatic
and may resolve spontaneously in young, healthy people.  However, certain HPV strains can persist, resulting in
genital warts.  Genital warts can be treated (eg, topical podophyllin, cryotherapy, laser surgery) but may recur at
any time (Option 1).  High-risk HPV strains (types 16 and 18) increase risk of cervical, oral, and genital
cancers (Option 4).

The HPV vaccine helps prevent HPV infection and is most effective if taken before becoming sexually active. 
However, current guidelines suggest that even teens and young adults (age ≤26) who have already become
sexually active may benefit from HPV vaccination (Option 3).

(Option 2)  The majority of clinical organizations recommend that cervical cancer screening (Pap testing) be
initiated at age 21, regardless of sexual history.  In women age <21, HPV infection rarely progresses to
malignancy.  Overdiagnosis and treatment of potentially benign HPV infections can lead to negative
reproductive outcomes in the future (eg, pregnancy loss, preterm birth).

(Option 5)  Barrier methods (eg, condoms) can reduce the risk of HPV transmission.  However, abstinence is
the only definitive way to eliminate the risk of contracting STIs.

Educational objective:
Human papillomavirus (HPV) increases the risk of genital warts and cervical cancer.  Teens and young adults
should be vaccinated with the HPV vaccine, and women should receive cervical cancer screening (eg, Pap tests)
beginning at age 21.  Barrier methods (eg, condoms) can reduce HPV transmission but do not eliminate risk.

The client is experiencing a second-degree type 2 atrioventricular (AV) block (Mobitz II), which is
characterized by more P waves than QRS complexes.  The PR intervals are consistent or constant, reflecting
regular conduction of electrical impulses through the AV node, but dropped QRS beats randomly occur as
ventricular conduction is blocked.

A second-degree type 2 AV block can rapidly deteriorate to complete heart block (third-degree AV block),
which is life-threatening.  The nurse should quickly obtain a transcutaneous pacemaker, assess the client for
symptoms (eg, bradycardia, hypotension, syncope), and be prepared to pace the client if symptoms occur
(Option 2).  If the client is asymptomatic, the pacemaker is kept nearby in case the rhythm deteriorates, and the
health care provider is alerted.

(Option 1)  A common complication following myocardial infarction is the development of new arrhythmias. 
Although a second-degree type 2 AV block is not completely unexpected in this client, it indicates a concerning
situation that requires assessment and monitoring.

(Option 3)  Adenosine is used to treat supraventricular tachycardia.  Adenosine creates a transient heart block,
which then allows the heart to resume normal sinus rhythm.  It is never given for bradyarrhythmias.

(Option 4)  Medications should be reviewed as drug toxicity (eg, beta blockers, digoxin) can cause this type of
block.  However, this can be done after other interventions.

Educational objective:
Second-degree type 2 atrioventricular heart blocks are characterized by consistent PR intervals and dropped
QRS complexes.  Clients should have temporary pacing immediately available as rapid deterioration to
complete heart block can occur.

You might also like